Вы находитесь на странице: 1из 255

OSCE guide

Third edition
Table of contents

Table of contents ............................................................................................................................. 2


History taking Medicine ............................................................................................................... 6
General review:............................................................................................................................ 7
Notes .......................................................................................................................................... 12
Chest pain ACUTE ................................................................................................................. 14
Chest pain CHRONIC ............................................................................................................ 16
Headache.................................................................................................................................... 18
Blood results / Macrocytic Anemia / B12 Deficiency................................................................. 21
Difficulty swallowing ................................................................................................................ 23
Elevated liver enzymes .............................................................................................................. 24
Drinking / Alcohol ..................................................................................................................... 26
Fever / Tired .............................................................................................................................. 28
Diarrhea ACUTE .................................................................................................................... 29
Diarrhea CHRONIC ............................................................................................................... 30
ASTHMA .................................................................................................................................. 32
COPD management ................................................................................................................... 35
Ankle swelling Bilateral ......................................................................................................... 36
Ankle swelling Unilateral ....................................................................................................... 37
Congestive heart failure CHF ................................................................................................. 38
Heart racing ............................................................................................................................... 40
Fall ............................................................................................................................................. 42
Peripheral vascular disease: ....................................................................................................... 44
Urinary symptoms: .................................................................................................................... 45
Anuria ........................................................................................................................................ 46
Hematuria .................................................................................................................................. 47
Renal stones ............................................................................................................................... 48
Incontinence............................................................................................................................... 50
Lump Neck Swelling .............................................................................................................. 51
Lump Breast............................................................................................................................ 52
Dizziness.................................................................................................................................... 55
INR Counselling ..................................................................................................................... 57
Patient is receiving blood counsel for adverse reactions ........................................................ 59
Counselling Ventilator............................................................................................................ 61
Ethical questions ........................................................................................................................ 62
HIV post-test counselling .......................................................................................................... 63
Lung Nodule .............................................................................................................................. 64
High Creatinine.......................................................................................................................... 66
Impotence / Erectile Dysfunction .............................................................................................. 67
Rheumatology History Taking................................................................................................ 68
Multiple Sclerosis ...................................................................................................................... 69
Obesity....................................................................................................................................... 70
Epilepsy Counselling ................................................................................................................. 71
Medical note .............................................................................................................................. 73
Pre-diabetes Counselling ........................................................................................................ 74
Emergency Medicine..................................................................................................................... 75
Emergency Room ...................................................................................................................... 76
Trauma ....................................................................................................................................... 77

OSCE-guide-III.doc Page 2 of 255


Unconscious Patient Neuro..................................................................................................... 81
Unconscious Patient Diabetic ................................................................................................. 82
Unconscious Seizing Patient DT / Epilepsy / Brain tumour / ............................................ 84
Heart Attack Chest Pain (MI or Heart Block) ........................................................................ 86
Case 1: Chest pain with initial normal ECG.............................................................................. 87
Case 2: Chest pain with STEMI ................................................................................................ 89
Case 3: Chest pain v fibrillation / v tachy............................................................................... 90
Case 4: Chest pain v fibrillation intoxicated patient ............................................................ 90
Heart Block................................................................................................................................ 91
Headache.................................................................................................................................... 92
Acute Abdominal Pain............................................................................................................... 93
Upper GIT bleeding ................................................................................................................... 98
Lower GIT bleeding................................................................................................................... 99
ECG ......................................................................................................................................... 100
Phone calls ............................................................................................................................... 103
Physical Examination .................................................................................................................. 106
Medical Physical Exam ........................................................................................................... 107
Abdominal examination:.......................................................................................................... 109
Liver Examination ................................................................................................................... 111
Nasal bleeding  Hematological Examination....................................................................... 112
Chest Examination................................................................................................................... 114
Pneumonia ............................................................................................................................... 115
Cardiac Examination Essential HTN case ............................................................................ 117
Secondary Hypertension .......................................................................................................... 119
Hypertension............................................................................................................................ 120
SOB shortness of breathe...................................................................................................... 121
DVT ......................................................................................................................................... 122
Peripheral Arterial Disease Examination................................................................................. 125
Diabetic Foot ........................................................................................................................... 126
Neurological Examination ....................................................................................................... 127
Cranial Nerves Examination.................................................................................................... 129
Tremors.................................................................................................................................... 131
Thyroid Exam .......................................................................................................................... 132
Dermatomes ............................................................................................................................. 133
Neck Examination.................................................................................................................... 134
Carpal Tunnel Syndrome ......................................................................................................... 135
Hand Laceration / Wrist Laceration......................................................................................... 136
Back Pain ................................................................................................................................. 137
Acute Back Pain....................................................................................................................... 138
Chronic Back Pain ................................................................................................................... 139
Back Joint Examination ........................................................................................................... 140
Ankle Twist ............................................................................................................................. 141
Shoulder Joint .......................................................................................................................... 142
Elbow....................................................................................................................................... 143
Hip Joint................................................................................................................................... 144
Knee Joint ................................................................................................................................ 145
Obstetrics and Gynecology.......................................................................................................... 146
History taking OB-GYN ....................................................................................................... 147
OB/GYN cases......................................................................................................................... 147
MGOS history questions:......................................................................................................... 148
History of pregnant lady third trimester................................................................................ 149

OSCE-guide-III.doc Page 3 of 255


Vaginal Discharge.................................................................................................................... 150
Vaginal Bleeding Non-Pregnant / Not-Known Pregnant...................................................... 151
Vaginal Bleeding Pregnant / Ante-Partum Hemorrhage....................................................... 152
Abnormal Uterine Bleeding (AUB)......................................................................................... 153
Amenorrhea ............................................................................................................................. 155
Infertility .................................................................................................................................. 156
Counselling pre-eclampsia.................................................................................................... 157
Caesarean Section Counselling wants to have CS............................................................. 159
Caesarean Section Counselling does not want to have CS ................................................ 161
OCPs / Contraception Counselling .......................................................................................... 163
HRT counselling...................................................................................................................... 166
Needle Stick Counselling HIV ............................................................................................. 168
Counselling PAP smear ........................................................................................................ 170
Antenatal Counselling.............................................................................................................. 172
Endometriosis .......................................................................................................................... 174
Woman wanting an abortion.................................................................................................... 175
Osteoporosis Counselling / OR / Short Case ........................................................................ 176
Pediatrics ..................................................................................................................................... 177
History taking Pediatrics....................................................................................................... 178
Jaundice ................................................................................................................................... 182
IUGR........................................................................................................................................ 184
Crying Baby............................................................................................................................. 185
Chronic Cough Asthma ........................................................................................................ 186
Anemia..................................................................................................................................... 188
Vomiting .................................................................................................................................. 189
Diarrhea ................................................................................................................................... 191
Mother worried about her child weight ................................................................................... 193
Fever ........................................................................................................................................ 195
Runny Nose / Flu / URTI......................................................................................................... 196
Rash ......................................................................................................................................... 197
Delayed Speech........................................................................................................................ 198
Seizing child counselling ......................................................................................................... 199
ADHD counselling .................................................................................................................. 200
Vaccination counselling........................................................................................................... 202
Child with DM counselling...................................................................................................... 204
Bed wetting counselling / Nocturnal Enuresis......................................................................... 205
Breast feeding counselling....................................................................................................... 207
Psychiatry .................................................................................................................................... 209
Mental status exam the psychiatry interview........................................................................ 210
DSM-IV-TR............................................................................................................................. 213
History taking Psychiatry...................................................................................................... 214
Psychosis.................................................................................................................................. 216
Schizotypal personality disorder.............................................................................................. 218
Panic attack.............................................................................................................................. 219
Tiredness OR weight loss ........................................................................................................ 221
Sleep / fatigue notes................................................................................................................. 222
Insomnia .................................................................................................................................. 223
Domestic Violence Spouse Abuse ........................................................................................ 224
Child Abuse ............................................................................................................................. 226
Domestic abuser....................................................................................................................... 228
Depression ............................................................................................................................... 230

OSCE-guide-III.doc Page 4 of 255


Depression management / counselling..................................................................................... 230
Dysthymia................................................................................................................................ 231
Premenstrual Dysphoric Disorder (PMDD)............................................................................. 232
Abdominal Pain / Headache .................................................................................................... 233
Somatoform disorders DD ....................................................................................................... 234
Drug seeker .............................................................................................................................. 236
Lithium discontinuity............................................................................................................... 238
Manic patient ........................................................................................................................... 240
Suicidal attempt ....................................................................................................................... 241
Eating disorder......................................................................................................................... 243
Mini-mental status exam:......................................................................................................... 245
Dementia.................................................................................................................................. 246
Delirium................................................................................................................................... 248
Case 1: Dad has not been himself / not sleeping well.............................................................. 248
Case 2: DT ............................................................................................................................... 249
Smoking Cessation counselling ............................................................................................ 250
Refusal to treatment counselling........................................................................................... 251
Truth telling ............................................................................................................................. 252
Organ Donation........................................................................................................................ 253
OCD......................................................................................................................................... 254
NOTES .................................................................................................................................... 255

OSCE-guide-III.doc Page 5 of 255


History taking Medicine

History taking Medicine

OSCE-guide-III.doc Page 6 of 255


History taking Medicine

General review:

 Introduction:
 Chief complaint
 History of present illness
 Past medical history
 Family history
 Social history

Introduction

Chief complaint

TIME: Os Cf D
Analysis of
Character: PQRST
CC

 Chronic diarrhea: dehydration
HPI Impact  Anemia: fatigue
 Cancer: metastasis
Constitutional symptoms
Red flags
Risk factors
Differential
diagnosis

A Allergy
M Medications
PMH P PMH: diseases (DM, HTN, heart attack, stroke, cancer)
L LMP / Last tetanus shot
E Events: hospitalization / surgery

Any long term disease


FH
Any specific disease

How do you support yourself financially?


SH With whom do you live?
SAD

OSCE-guide-III.doc Page 7 of 255


History taking Medicine

Introduction:
- Knock the door
- Go to the examiner  give stickers  use alcohol rub (disinfective)
- Stand at the edge of the chair
- Good evening Mr , I am Dr , I am the physician in charge today,
o < 18 years: use first name
o > 18 years: use Mr / Mrs / Ms
- I understand that you are here because of ,
Examples:
- History Taking: Good evening Mr , I am Dr , I am the physician on duty now, and I
understand that you are here today because of . In the next few minutes I will ask you some
questions about your cc, to figure out a working plan that can help you. If you have any
concerns or questions, please fell free to stop me and let me know.
- Physical examination: Good evening Mr , I am Dr ; I am the physician in charge now. I
understand that you are here today because of . In the next few minutes I will do a physical
exam on your (e.g. shoulder), during which I will ask you to do some movements that may
cause some discomfort and may be some pain, if you feel either, please do not hesitate to stop
me. And if you have any concerns, please let me know. And I will be telling the findings to
the examiner while we proceed.
1- Chief complaint
[A] If the CC not known
- How can I help you today?
1. Start to ask based on the age:
MALE FEMALE
> 65 yrs  Do you take meds on regular basis? Do you have a list of it? Or the
bottles? Do you take sleeping pills?
 Do you have difficulties with sleeping?
 Do you have difficulties with your balance1, any falls?
 Do you have difficulties with urination (incontinence / retention)?
 Do you have changes in your vision / hearing?
 Do you have changes in your mood / memory?
50s  Do you have problem drinking  use CAGE
 Depression2  identify through social history
 ED / Impotence
30s  Psychiatric problems
 SAD  social history
Teen / 20s  Premature ejaculation  Abortion
 STDs  STDs
 Eating disorders

2. Special conditions:
 Fatigue Domestic abuse
 Insomnia
 Headache
 Abdominal pain
 Vaginal bleeding

1
Normal pressure hydrocephalus: ataxia / incontinence / dementia
2
Common triad association: alcohol / depression / suicide

OSCE-guide-III.doc Page 8 of 255


History taking Medicine

 Chest pain Panic attack


 SOB
 Heart racing
 Dizziness
 Numbness

[B] If the CC is known


CLEAR NOT clear
 Any pain: headache, chest pain  Vague symptoms: dizziness, tired
 Cough  When the patient uses medical terms:
 Fever abortion, jaundice, palpitation
 When the patient uses the words: change /
difficulty
Clarify: Clarify:
1. Start first open-ended questions: - Use closed ended questions
- What do you mean?
- Can you tell me more about this
2. Active listening:
- Do not interrupt
- Do not duplicate
3. Body language: nod your head

2- History of present illness


TIME (Os Cf D):
- Onset:
o How did it start (sudden / gradual)?
o Setting: what were you doing?
- Course: from that time till now, is your cc all the time or is it on and off (continuous vs.
intermittent)?
o Continuous:
 From the beginning
 Does it or or the same?
 Frequency: is it your first time?
o Intermittent:
 Frequency: how often did you have it in the last ()?
 Are these attacks similar or different?
in duration (longer) or in severity (more severe) or in
frequency (more often)?
 What brings it? On doing certain thing, occurring at rest, awakening you
from sleep?
- Duration:
o Usually given in the question
o If > 24 hours  empathy: were you able to sleep

Character:
+ PAIN: PQRST always ask from the beginning?
- Position: where did it start? Can you point with one finger on it?
- Quality: how does it feel like? Squeezing, tightness, sharp, stabbing, burning?
- Radiation: does it shoot anywhere?

OSCE-guide-III.doc Page 9 of 255


History taking Medicine

- Severity: on a scale of 10, 1 being the mildest pain you have ever had and 10 is the most
severe, how much do you rate this one?
o If bad pain  empathy: this must be difficult
- Timing:
o Does it change with time; is it more in morning or towards the end of the day?
o Any variation?
- Triggers:
o What brings your headache?
o Is it related to: stress / lack of sleep / over sleep / flashing lights / smells?
o If female: is it related to your periods? Are you taking any meds or OCPs?
o Any diet triggers?

+ Fluids (e.g. bleeding, diarrhea, vomitus):


- COCAB/D: colour, odour, contents/consistency, amount blood/discharge

What or :
- What increases or decreases your cc?
- Examples: noise / quiet places / movements / resting / coughing / leaning forward / lying
down /

[D] ASSOCIATED SYMPTOMS:


- In addition to your cc, did you notice any other symptoms?
- Now, I am going to ask you more questions to see if you have any other symptoms beside
your cc.
+ By systems: e.g. chest pain
1- Same system
2- Near-by systems
3- Constitutional symptoms [RED FLAGS]
4- Risk factors
Now, I am going to ask you more questions to see if you have any medical conditions
that may explain / cause / predispose your cc.
5- Causes and consequences
6- Review of systems:
+ By differential diagnosis: e.g. headache
+ By causes and consequences: e.g. Macrocytic anemia Vit B12 deficiency

3- Past medical history


Because it is the first time I see you, I need to ask you some questions about your past medical
history.
 Allergy / Medications:
a. Allergy
b. Medications (OTC, Rx meds, supplements, herbs)
 Past history of diseases for which you see doctor on regular basis (DM, high blood pressure,
heart attack, stroke, cancer)?
 LMP for females
 Events: any history of hospitalization / procedures?

4- Family history
Because it is the first time I see you, I need to ask you some questions about your family medical
history, and by this I mean your parents and siblings.

OSCE-guide-III.doc Page 10 of 255


History taking Medicine

1- How do you describe their general health?


2- Any long term disease in the family? DM, HTN, heart attack, stroke?
3- Any specific disease runs in the family?

5- Social history
1- How do you support yourself financially?
2- With whom do you live?
a. Alone  are you in any relationship? Are you sexually active?
b. With a family  how is the relation with ? Is she/he supportive?
3- SAD:
a. Do you smoke?
b. Do you drink alcohol?
i. How much?
ii. For how long?
c. Have you ever tried recreational drugs?

OSCE-guide-III.doc Page 11 of 255


History taking Medicine

Notes
EMPATHY:
If during history taking you noticed the patient is in pain  empathy: I can see you are in pain,
please bear with me for few minutes and I will give you a pain medication as soon as I can
- In the short cases (5 minutes)  use at least 1-2 empathy statement
- In the long cases (10 minutes)  use at least 3 empathy statements
- Patient says Im not ok / Im not so good  I am sorry to hear that
- Patient says I fell down  Oh, did you hurt yourself / No I am glad to hear that
- After suicide  It looks like you have gone through difficult times, can you tell me more
about these difficulties you are facing
- Patient is regaining consciousness in the ER  Mr you have had and you are in the
hospital now, you are ok now, I am Dr and we are here to make sure youre ok
I have a concern!
Whenever the patient says: I have a concern: STOP the interview!
- Can you tell me your concerns!
- Why are you concerned?
Worried / occupied patient!
Whenever the patient shows non-verbal clues of being worried / occupied:
- I can see that you are worried / occupied! Would you like to tell me more about your worries
or concerns?
Question types:
- Types of questions you can use: open-ended, closed-ended, choices
- Types of questions you can NOT use: leading questions, stacking questions

Time usage technique: 1/2/3


If you are stuck during the history taking; and could not find questions to ask;
- First time: Summarize
- Second time: Ask about constitutional symptoms / Review of systems
- Third time: PMH / FH / SH
Medications:
When you ask about the medications and even if the patient says NO, in certain diseases, you
need to confirm special medications, by saying, what about
- Diarrhea  what about antibiotics
- Asthma  aspirin / blockers (HTN, heart failure, thyroid disease, social phobias)
- Migraine  what about OCPs? (any birth control pills)
- Bleeding  what about aspirin / blood thinners
- Torticollis  do you take anti-psychotics? Do you see psychiatrist/ did he give you any
medications

MSD (mood / suicide / drinking): whenever you find one, screen for the others
When the patient comes with a chronic long duration complaint, ask him: and what
happened recently that made you decide to seek medical advice now?
Whenever the patient has something affecting his life / social issue: Refer to social worker
/ services
Do NOT criticize other doctors or the patient

OSCE-guide-III.doc Page 12 of 255


History taking Medicine

For breaking bad news [SPIKES] approach:


o Setting
o Perception of the patient: what do you know about
o Invitation: how much details you would like to know about
o Knowledge: give knowledge in understandable pieces, and make sure the patient
understands this info.
o Empathy /+/ Expectations: what are your expectations from todays visit
 You will have tremendous support, you are not working alone
 We will try to make you as comfortable as possible
 Oncology group AND palliative group: Referral: do you have any doctor you
are comfortable with?
o Summarize /+/ Strategy

Counselling:
1. Inform the patient
a. The medical condition is called
b. Explain the pathophysiology
c. Consequences / complications of the condition! May happen again, may affect
ability to do certain things,
d. Investigations that might be needed to conclude the condition OR to look for
complications
2. Preventive measures: e.g. modify the poly-pharmacy
3. Treatment: life style / medications (side effects / alternatives / consequences of not
receiving treatment)
4. Offer more info: brochures / web sites / support groups
5. Break every 30-60 seconds (check & recheck that your patient understands); ask the
patient: does that make sense? Is this acceptable? Reasonable? Is it clear?
6. General tips for the counselling sessions:
- Make it interactive not lecturing
- At the beginning ask whether your patient has a specific concern
- Do not mislead your patient; if you are not sure about any thing, say that this is a very
good question and you are going to check the answer for him.
Whenever you hear car accident:
- I am sorry to hear that!
- Was anyone hurt? I am sorry for that
- Were you driving or a passenger?
If you do not know the answer to a question:
- This is a good question / point, I will check it for you and we will discuss it next visit.
- It is better to refer you to the specialist; there are too many points regarding this issue that it
will be better to discuss it with the specialist.
A good statement to use in different counselling situations: always in medicine, we balance
the benefits and the side effects.
GIT symptoms:
- Nausea / Vomiting - Heart burn / acidic taste in mouth
- Abdominal pain - Distension / bloating / gas
- Change in bowel movements: constipation / diarrhea
- Blood in stools / vomiting blood
- LIVER: yellowish discoloration / itching / dark urine / pale stools

OSCE-guide-III.doc Page 13 of 255


History taking Medicine

Chest pain ACUTE


Introduction: Good evening Mr , I am Dr , I am the physician on duty now, and I
understand that you are here because you have chest pain for the last minutes. In the next few
minutes I will ask you some questions about your chest pain, to figure out a working plan that can
help you. If you have any concerns or questions, please fell free to stop me and let me know
Analysis of  OsCfD: Onset / setting: what were you doing?
CC  PQRST:
Position: where did it start? Can you point with one finger on it?
Quality: how does it feel like? Squeezing, tightness
Radiation: does it shoot anywhere? Your jaw, your shoulders, your back?
 What or :
Breathing / position
How did you come to the clinic today? Ambulance  did they give you
aspirin / nitrates? Did it help you?
Impact  Atherosclerosis:
Hx of stroke? Symptoms of stroke (weakness / numbness / change in vision
/ difficulty finding words)?
Any sexual dysfunction?
Do you feel abdominal pain after eating?
Do you feel cold extremities
Do you feel cold feet? Pain after walking?
 CHF:
SOB? How many pillows do you use? Do you wake up gasping for air?
Any swelling in your LL? How high does it go? Is it related to position?
Eye puffiness? Pain on the liver?
Red flags Constitutional  Fever / night sweats / chills
symptoms  How about your appetite? Any weight changes?
 Any lumps or bumps in your body?
Risk factors CAD
Pericarditis
PE
Differential Same system  Nausea / vomiting
diagnosis  Sweating / feeling tired
 SOB  if yes, analyze (OsCfD)
 Do you feel your heart racing?
 Did you feel dizzy / light headedness / LOC? Are you tired?
 Did you notice swelling in your ankles? Legs? Calf muscles?
Near by  CHEST:
systems Any cough or phlegm? Chest tightness? Wheezes?
Recent fever / flu like symptoms? Muscles/ joint ache?
 GIT:
Difficulty swallowing (esophageal spasm)
Heart burn / acidic taste in your mouth?
Any hx of PUD? Reflux? GERD?
 Chest wall: any trauma, any blisters / skin rash on your skin
 DVT: any pain / swelling / redness in your legs / calves? Any
recent long travel?

OSCE-guide-III.doc Page 14 of 255


History taking Medicine

Risk Factors:
CAD (Coronary Artery Disease):
MAJOR:
o High blood pressure
o High blood sugar
o High cholesterol: have you got your cholesterol measured before?
o Family hx of heart attack at age < 50 yrs
o SAD: Smoking / Cocaine
MINOR:
o Look for obesity
o Do you exercise
o How about your diet, do you eat a lot of fast food?
o Are you under stress?
Pericarditis:
o Recent flu like symptoms
o Medications (Isoniazide / Rifampicin)
o Hx of surgery
o Hx of heart attack
o Hx of kidney disease / puffy face / frothy urine
o Hx of TB
o Hx of autoimmune disease
Pulmonary Embolism:
o Recent long flight
o History of malignancy
o Family history of blood clots
o Female: pregnancy / OCPs / HRT

Chest Pain
Acute Chronic
Minutes hours Hours days Intermittent Continuous
Cardiac: Cardiac: Cardiac:
- CAD - Pericarditis - Unstable angina
- Aortic dissection - Unstable angina
Non-cardiac: Non-cardiac - Cancer
- Tension - Pneumonia - Herpes zoster
pneumothorax - Pleurisy - Trauma
Panic attack Pulmonary embolism Panic attack
GIT: GIT:
- GERD - GERD
- PUD - PUD
- Esophageal spasm - Esophageal spasm

Questions:
 Investigations: ECG / Cardiac enzymes

OSCE-guide-III.doc Page 15 of 255


History taking Medicine

Chest pain CHRONIC


Chest pain for 6 weeks
UNSTABLE ANGINA /+/ GERD GIT CAUSES OF CHEST PAIN

Intro But first I would like to ask you, how do you feel now?
Analysis of  OsCfD: Onset / setting: what were you doing?
CC  PQRST:
Position: where did it start? Can you point with one finger on it?
Quality: how does it feel like? Squeezing, tightness
Radiation: does it shoot anywhere? Your jaw, your shoulders, your back?
 What or :
Breathing / position
Is it related to activity? How many blocks were you able to walk? And
now?
How about rest? And during night?
When was the last attack
 Triggers Angina GERD
Exertion
  Golf (leaning forward)
Stress (emotional)
  Coffee / dairy products
Cold air
  Smoking / Alcohol
Heavy meals
  Heavy / late meals
Sexual activity
  Pregnancy (progestin)
Impact Effect  Atherosclerosis  Chronic cough
 CHF  Change in the voice
Red flags Constitutional  Fever / night sweats / chills
symptoms  How about your appetite? Any weight changes?
 Any lumps or bumps in your body?
Risk factors CAD
Differential Same system  Nausea / vomiting
diagnosis  Sweating / feeling tired
 SOB  if yes, analyze (OsCfD)
 Do you feel your heart racing?
 Did you feel dizzy / light headedness / LOC? Are you tired?
 Did you notice swelling in your ankles? Legs? Calf muscles?
Near by  CHEST:
systems Any cough or phlegm?
Chest tightness? Wheezes?
Recent fever / flu like symptoms? Muscles/ joint ache?
 GIT:
Difficulty swallowing (esophageal spasm)
Heart burn / acidic taste in your mouth?
Any hx of PUD? Reflux? GERD?
 Chest wall: any trauma, any blisters / skin rash on your skin
 DVT: any pain / swelling / redness in your legs / calves? Any
recent long travel?
PMH
FH
SH

OSCE-guide-III.doc Page 16 of 255


History taking Medicine

Counselling:

Concern: The patient has a concern; is this IHD? Is his heart endangered?
This is quite a reasonable concern? What made you think about that?
Especially you have many risk factors that may predispose to heart attack. Right now the
physical exam is ok; it is less likely your condition is due to heart problem. But we still
need to check your heart more, we will do some lab works and an electrical tracing for
your heart (ECG), then if we find that we still need, we may send you to have a stress
ECG, in which, we trace your heart while you are exercising. Then we know for sure the
condition of your heart.
However, we would like to take measures to try to decrease your risk of developing heart
attack, e.g. exercise / diet / smoking / cholesterol.
On the other hand, the most likely diagnosis of what you have is a medical condition
called GERD. GERD stands for Gastro-Esophageal Reflux Disease, any idea about
that? Do you know anything about GERD?
Explain with a drawing: the esophagus (food pipe) / lower esophageal sphincter /
physiologic mechanism to keep it competent / in GERD  weak sphincter  acid
refluxes / irritates the esophagus / impact (short term and long term)
Treatment:
o Avoid triggers
o Life style modifications:
 Raise the head of the bed
 Smaller meals
 Do not eat late
 smoking
o Medications: proton pump inhibitors (PPIs), e.g. pantoprazole
o Side effects of PPIs:
 In general, proton pump inhibitors are well tolerated, and the incidence of
short-term adverse effects is relatively uncommon
 Common adverse effects include: headache (in 5.5% of users in clinical
trials), nausea, diarrhea, abdominal pain, fatigue, and dizziness. Long-term
use is associated with hypomagnesemia
 Because the body uses gastric acid to release B12 from food particles,
decreased vitamin B12 absorption may occur with long-term use of proton-
pump inhibitors and may lead to Vitamin B12 deficiency
 Infrequent adverse effects include rash, itch, flatulence, constipation, and
anxiety

OSCE-guide-III.doc Page 17 of 255


History taking Medicine

Headache
HPI:
OsCfD: gradual onset / all the time / increasing / for few days
PQRST: temporal area / vague deep pain / severe
o Severe:  empathy: this must be difficult, were you able to sleep
o Triggers:
 What brings your headache?
 Is it related to: stress / lack of sleep / flashing lights / smells / diet?
 If female: is it related to your periods? Are you taking any OCPs?
What or ? Lying down / coughing / resting in quiet room /

Associated symptoms / differential diagnosis:


1- Infection:
Fever / night sweats / chills / constitutional symptoms
Neck pain / stiffness
Recent flu like symptoms / skin rash
Bothered by light
Nausea / vomiting

2- Subdural hematoma:
Trauma / fall
SAD (Smoking, Alcohol, Drugs)

3- Subarachnoid hemorrhage:
Very acute /+/ Very severe headache / the worst headache
History of aneurysm or polycystic kidney disease
Visual changes (pupil changes)
Your heart is beating slow

4- Neurological screening:
If while you are doing the neurological screening, you suspect particular cause, e.g.
temporal arteritis  go to TA block then return to complete the neurological screening.
Cranial nerves:
o Any change in smelling perception?
o Any difficulty in vision / vision loss?
o Any difficulty in hearing / buzzing sounds?
o Difficulty finding words? Aphasia?
o Difficulty swallowing?
Brain:
o Any dizziness / light headedness / LOC?
o Any tremors / jerky movements / hx of seizures?
Personality and cognition:
o Any memory / mood / concentration problems?
o Did anybody tell you that you there is a change in your personality recently?
UL/LL:
o Any weakness / numbness / tingling in your arms / legs
o Any difficulty in your balance / any falls?
Spine:
o Any difficulty with urination / need to strain to pass urine?
o Any change in bowel movements?

OSCE-guide-III.doc Page 18 of 255


History taking Medicine

5- Temporal arteritis:
Age > 55 years
When you touch this part of your head, is it painful? Can you comb your hair?
Do you feel cord-like structure?
Do you have any visual disturbances / impairment?
When you are chewing, is it painful, cramps in your jaws?
Any weakness / numbness in your shoulders / hips?
Is there any cough? Mild fever?
6- HTN:
Were you diagnosed before with high blood pressure?
Do you know your blood pressure? Have you had it checked before?
Salty food? Family history of HTN / heart disease?
Any history of repeated headaches?
7- Extra-cranial causes of headache:
Eyes: any hx of glaucoma, red eye, pain in your eyes? Do you usually wear eyeglasses?
Do you see well? Any vision problems? When was last time you saw your optometrist?
E do you have any pain / discharge in your ears?
N nasal discharge / sinusitis / hx of facial pain?
T any teeth pain / difficulty swallowing?
8- Medications:
Do you take any nitrates?
Do use too much of advil (or other NSAIDs)? For how long?
Were you used to take large amounts of coffee and then you stopped abruptly?
OCPs?

Temporal Arteritis:
Investigations: Treatment: If suspect GCA (Giant Call Arteritis),
TA biopsy immediately start high dose prednisone; 1 mg/kg
Doppler OD (to prevent blindness) then maintain dose daily
ESR (in divided doses), then taper prednisone dose
CT head after symptoms resolve.

Polymyalgia Rheumatica:
Constitutional symptoms + Fatigue Treatment: Corticosteroids; 15 mg/day (for long
Age > 50 yrs periods of time). Taper after ESR decreases < 50
ESR > 50 mm/hr mm/hr and stop if ESR normalizes (< 20 mm/hr)

OSCE-guide-III.doc Page 19 of 255


History taking Medicine

PRIMARY HEADACHE

Intermittent / episodic
Headache Tension Migraine Cluster
Duration Days Hours Minutes
Quality Pressing / tightening / Mostly unilateral / Comes in series / severe
bilateral pulsating / interferes with pain / hyperaesthesia
daily activities
Place Band around the head Mostly unilateral Around the eyes / nose
Associated Photophobia / phonophobia Red eyes / lacrimation /
symptoms rhinorrhea / sweating
Aggravating Stress Physical activity / motion Smoking / alcohol
factors Physical injury Light / sound Smell / exercise
Others  Family history
 Types:
+ Classical: with aura
+ Non-classical: no aura
Treatment Acute phase:
- Acetaminophen - Acetaminophen - Oxygen
- NSAIDs - NSAIDs (ibuprofen) - NSAIDs
- Physiotherapy / ms - Triptans / ergotamine
massage / heat Prophylactic:
compresses (neck) - Remove precipitant
- Ca channel blockers
- Triptans (somatriptan)

OSCE-guide-III.doc Page 20 of 255


History taking Medicine

Blood results / Macrocytic Anemia / B12 Deficiency

Cases:
Middle age man received blood report showing Macrocytic anemia
Elderly (65 years old) man presenting with ataxia, dizziness, macrocytic anemia.
Findings: poor diet. Most likely diagnosis: pernicious anemia

Investigations:
CBC / Differential / Peripheral blood film
B12 level in the blood / Folic acid level in the blood

Introduction
Good morning Mr , I am Dr , I understand that you are here today (OR we called you to
come) to get the results of your blood tests (OR x-ray) that you have done few days ago, I have it
and I am going to discuss it with you. However, because it is the first time that I see you, I need
first to ask you some questions to help me get better understanding and interpretation of these
results. Is it OK with you?

+ If patient anxious about results  tell her/him then continue history

1- First let me ask you few questions about the lab test itself (this applies to any blood work, x-
ray, HIV testing, biopsy, jaundice, anemia):
Why have you done this test?
Is it the first time to have it?
Who ordered this test for you? Why?
When did you have it?

2- Give the information:


If it is bad news  SPIKES
If abnormal blood results or x-ray: explain the results to patient, to decrease the patient
anxiety. The blood tests you had show that you have special kind of anemia that we call
Macrocytic anemia in which the size of the red blood cells (which are a component of
your blood) is larger than usual, there is different causes for this.
o If the patient panicked? Is it serious doctor?
 Do you have any concerns?
 There are different causes that may lead to this result, some of them are
serious, could be, we need to do more investigations.

3- I would like to ask you some questions to see how did this (anemia) affect you:
CONSEQUENCES of anemia:
Anemia symptoms:
o Did anyone comment that you are pale, recently?
o Did you notice any in your activity level?
o Heart racing / SOB / chest pain with exercise?
o Any dizziness / light headedness / fainting?
Neuro symptoms:
o Any tingling / numbness / in your feet?
o Difficulty in your balance / any falls?
o Any difficulty concentrating / memory problems?

OSCE-guide-III.doc Page 21 of 255


History taking Medicine

4- I would like also to ask more questions to find out what might be the cause:
CAUSES of Vit B12 deficiency:
Diet intake: Are you vegetarian? For how long? Do you take supplements?
Gastric causes:
o Did you have any surgeries in your stomach? When?
o History of long standing PUD? Any heaviness / fullness after meals /
indigestion? (Lack of acidity)
o Were you ever yourself or any member of your family diagnosed with what is
called autoimmune disease; by this I mean a condition called pernicious
anemia, or rheumatoid disease / lupus?
Terminal ileum:
o Did you have any bowel surgery before?
o Were you diagnosed with Crohns disease before? Any repeated attacks of
diarrhea? Any foul smelling bulky stools?
Pancreatic and liver failure:
o Any hx of liver / pancreatic disease?
o Yellowish discoloration / itching / dark urine / pale stools?
Alcohol:
o Do you drink alcohol? How much? For how long?
Meds:
Do you take medications on regular basis? What kind?
o Have you ever been diagnosed with epilepsy? Do you take anti-epileptics?
o Do you see a psychiatrist? Do you take a mood stabilizer?
o Any hx of chemotherapy? Have you ever taken a drug called methotrexate?
Hematological causes:
o Any recent bleeding (nose / gum / coughing / vomiting blood)? Any bruises /
dark urine / tarry stools?
o Any fever / night sweats / chills? Change in appetite / weight loss? Lumps and
bumps in your body (for LNs)? Bony pains? Any repeated infections?
Parasites:
o Have you ever consumed raw fish (chronic intestinal infestation by the fish
tapeworm: Diphyllobothrium)?

5- PMH
6- FH
7- SH

OSCE-guide-III.doc Page 22 of 255


History taking Medicine

Difficulty swallowing
What do you mean by difficulty swallowing?
Do you feel difficulty initiating the swallowing?
Do you feel pain when you swallow?
Do you feel food is stuck? Can you point where it is usually stuck?
Dysphagia
(esophageal)

Progressive Intermittent
All the time and On and Off Achalasia:
respiratory
symptoms

Progressive, solids Fluids first Fluids and solids Solids only Scleroderma:
then fluids Then solids (Large bolus) reflux / tight skin of
fingers / change
hand color when
exposed to cold
(Reynauds
Mechanical  Achalasia Esophageal spasm Esophageal webs and
disease)
Cancer OR stricture  Scleroderma rings

Mechanical Dysphagia:
Analysis  OsCfD: gradual, progressively, to solids then fluids / PQRST / What /
of CC  Associated symptoms:
The same system:
o Nausea / vomiting / undigested food
o Change in bowel movements
o Change in the size of the abdomen / abd pain / blood in stools
o Liver: yellowish discoloration / itching / dark urine/ pale stools
Near-by systems:
o Any chest pain / tightness
o Any cough / change in your voice / neck swelling (thyroid lump)
Impact Weight loss
Red flags  Constitutional symptoms: fever/ night sweats/ chills / change in appetite / loss
of weight / lumps & bumps
 Risk factors:
GERD / PUD:
o Hx of heart burn
o Were you ever diagnosed with a condition called GERD / PUD
o Have you ever checked with a camera or a light (endoscope)
inserted into your stomach
Smoking / Alcohol
Family history: esophageal cancer
Radiation to chest
Have you ever swallowed any chemical?
Barium swallow: string sign /or/ apple core sign / graded narrowing of intra-esophageal diameter
extending from T5-T8 level  most likely diagnosis: esophageal cancer
Investigations: endoscopy and biopsy / chest x-ray and CT / liver function tests / abdominal US

OSCE-guide-III.doc Page 23 of 255


History taking Medicine

Elevated liver enzymes

Introduction
HPI:
1- First let me ask you few questions about the lab test itself (this applies to any blood work, x-
ray, HIV testing, biopsy, jaundice, anemia):
Why have you done this test?
Is it the first time to have it?
Who ordered this test for you? Why?
When did you have it?

2- Give the information:


Liver enzymes: AST / ALT 2
Explanation: there is increase in one of the markers used to assess / check the liver
functions, it may indicate that there is an injury to your liver, I am happy you came here
today to discuss it so that we can figure this out.

3- I would like to ask you some questions to see how did this affect you:
CONSEQUENCES of liver injury:
Acute phase:
o Any yellowish discoloration / itching / dark urine/ pale stools
o Recently, have you noticed any fever / flu-like symptoms / muscle/joint aches
o Constitutional symptoms: sweats / chills / appetite / weight / lumps
Chronic manifestations:
o Did you notice any increase in the size of your abdomen? Puffiness in your face?
Swelling in your legs/ ankles?
o Bruises in your body?
o Vomited blood? Blood in stools?
o WITH ALCOHOL: did you notice changes in memory and concentration? Any
weakness / numbness? Balance and falls?

4- I would like to ask you more questions to find what might be the cause:
CAUSES of liver injury:
Now, I would like to ask you some questions to see if you were exposed to liver disease
without being aware of that, some of these questions may be personal, but it is important to
ask (start from least offensive to most offensive)
Including the alcohol, during which  Drinking assessment

5- PMH:
Were you ever diagnosed with liver disease before
Were you ever checked for liver disease before
Were you ever vaccinated for liver disease before
6- FH: suicide / depression / drinking / liver cancer
7- SH

OSCE-guide-III.doc Page 24 of 255


History taking Medicine

Now, I would like to ask you some questions to see if you were exposed to liver disease without
being aware of that, some of these questions may be personal, but it is important to ask (start
from least offensive to most offensive)

Oral
Any recent travel outside Canada?
Did you eat any raw shell fish? Did you eat in new place that you are not used to?

Surgical
Any history of surgeries / hospitalization?
Any history of blood transfusion?
Any history of blood donation?

Social
How do you support yourself financially? If hazardous occupation?
Did you get exposed to blood products / body fluids?

Risky behaviour
Any tattooing or piercing?
SAD?
o Do you smoke?
o Drink alcohol? How about the past?  Drinking assessment
o Have you ever tried recreational drugs? Any injected drug use? When was the
last time?
With whom do you live? For how long have you been together?
Before being with your current partner, did you have other partners?
When did you start to be sexually active? How many sexual partners did you have from
that time till now?
What is your preference, men, women or both?
What type of sexual activity do you practice? Did you practice safe sex all the time? And
by that I mean using condoms!
Any history of sexually transmitted infections? And screening for STIs?
Have you ever had sex with sex worker?
Within the last 12 months, have you had any other sexual partners?

When do you need to take extensive Liver investigations


sexual history? Risky behaviour!
 Liver enzymes / Jaundice  AST / ALT / GGT / Alkaline phosphatase
 Fever / Tired  LDH
 LNs  Bilirubin
 Discharge  INR / PTT
 PAP results  Albumin
 HIV test results  Glucose
 Serum ferritin / TIBC / serum ceruloplasmin
 Viral serology: Hep A/B/C antibodies and
Hep B Ag
 Abdominal US
 Liver biopsy

OSCE-guide-III.doc Page 25 of 255


History taking Medicine

Drinking / Alcohol

Drinking assessment

Use / abuse MOAPS Impact


Medical Social Legal
Drinking hx Mood Liver Home
How much? Organic Memory / conc. Work
CAGE Anxiety B12
Psychosis Heart
Self-care / suicide

Use / Abuse:
Do you drink alcohol? How about the past?
What do you drink?
o For how long?
o How often?
How much?
o 2 bottles of wine a day? 12 beers a day? Have you drunk more than 6 drinks in
one setting? Have you ever exceeded the amount you intended to drink?
o Do you drink alone or with other people?
o Did you ever drink to the extent of black out?
o What do you feel if you do not drink? Any shaking / heart racing / sweating?
Have you ever had seizures before? Were you hospitalized? Did you have
delirium tremens?
o Do you avoid going to places where you do not have access to alcohol?
CAGE:
o Did you ever think that you need to cut down on your drinks?
o Do you get annoyed by other people criticizing your drinking?
o Do you feel guilty for your drinking habits?
o Early morning drink?
Problem drinking: 2 of CAGE list for males OR 1 for females

MOAPS:
Mood:
o How is your mood? Interest? If ok  do not proceed
o If not ok  MI PASS ECG
Anxiety:
o Are you the person who worries too much?
o Do you have excessive fears or worries?
Psychosis:
o Do you hear voices or see things that others do not?
o Do you think that someone else would like to hurt you?
Self care / suicide
o Any chance you might harm yourself or somebody else?

OSCE-guide-III.doc Page 26 of 255


History taking Medicine

Impact:
I am going to ask some questions to check what effects does alcohol have on your life?
The medical is already done in the consequences of liver injury
Social:
o With whom do you live? For how long? How is the relation? Is there any
problems? Is it related to your drinking habits?
o How do you support yourself financially? Where do you work? For how long?
 How is the relation with your coworkers / manager?
 Do you miss working days because of your drinking habits?
 Do you need to drink at work?
Legal:
o Did you have any legal issues related to your drinking?
o Fights? Arguments? Were you arrested before because of drinking?
o Were you charged before for DUI (driving under influence)?

Alcoholic beverages:
Beer:
o Alcohol percent around 5%
o Pitcher (60 oz) = 3 pints (pint = 20 ounces)
o Ounce (oz) = around 28-30 ml
Wine / Champaign:
o Alcohol percent around 12%
o Bottle: 750 ml
o Glass: 150 ml
Hard liquor (whisky / gen):
o Alcohol percent around 40%
o 1 glass (shot) = 1 oz (50 ml)

Ethical challenges (patient asks)


Doctor is there any possibility that these test results are wrong? Can we repeat it to make
sure?
 Whenever we do blood work, we take a lot of measures to make sure it is accurate and
usually if it is positive, the labs double check it before they send it. Chances of having a
mistake are very low.
 Based on your history, symptoms and examination, we might not only repeat the test, we
might need to do more tests. It is early to tell now, let us proceed with history and
examination, and at the end I will be able to tell you.

Do you drink yourself, doctor?


 Why do you ask?
 Whether I drink or no does not matter, it is better to discuss your case now.

By the way, are you Canadian graduate, doctor?


 I fulfill all the requirements to practice medicine in Canada!
 I passed all the exams, follow Canadian guidelines, and practice under supervision!

OSCE-guide-III.doc Page 27 of 255


History taking Medicine

Fever / Tired

Intro But first I would like to ask you, how do you feel now?
CC  Fever
 Do you have other concerns?
Analysis OsCfD
 Did you measure it? How often? How? What is highest?
 And medications? Did it help?
 Any flu / illness / sickness
 Any diurnal variation? Any special pattern? Is it more every 3rd or 4th day?
(malaria)
Impact Are you able to function?
Red flags Constitutional symptoms
Differential  CNS: headache / neck pain / stiffness / nausea / vomiting / vision changes /
diagnosis bothered by light / weakness / numbness
 ENT:
Extensive Ears: pain / discharge
review of Nose: runny nose / sinusitis (facial pain)
systems Throat: sore throat / teeth pain / difficulty swallowing
 Cardiac: chest pain / heart racing (pericarditis)
 Lung (pneumonia, PE (DVT), TB, cancer): cough / blood / phlegm / wheezes /
chest tightness / contact with TB pt
3
 GIT (except the liver ): abd pain / distension / change in bowel movements /
blood in stools
 Urinary: burning / frequency / flank pain / blood in urine
 Do you have any discharge? Ulcers? Blisters? Warts?
 MSK: joint pain / swelling / ulcers in your body / mouth / skin rash / red eye
 Autoimmune: fm hx / dx before with autoimmune dis
 The LIVER:
Local: yellow / itching / dark urine / pale stools
Dx before with liver dis? Screened? Vaccinated?
Transition to risky behaviour
PMH Cancer / Autoimmune disease
FH Cancer / Autoimmune disease
SH Does your partner have any fever? Discharge? Skin rash?

 During the last 6 weeks, did you seek medical attention?


 What made you choose to come today? Compared to before, any special changes?

3
The liver will be put at the end as a transition to ask about risky behaviour (see liver enzymes case)

OSCE-guide-III.doc Page 28 of 255


History taking Medicine

Diarrhea ACUTE

Analysis of OsCfD  How many times?


CC COCA  What bout during night?
B/Mucous Yes  organic
No  irritable bowel syndrome (IBS) day only
How does if affect your sleep?
 Consistency: watery / loose / formed / bulky. Any floating
fat droplets / difficult to flush / undigested food
 Did you notice blood? When did it start?
Before you have your bowel move?
Mixed (higher source of bleeding)?
On the surface?
AS  Pain  OCD / PQRST
If pain improves after bowel movement: IBS
 Vomiting
 Alteration with constipation
Impact  Acute dehydration: thirsty / dizziness / light headedness / LOC / weak
Red flags  Constitutional symptoms for infection / cancer
 Flu like symptoms
 Any body around you have the same diarrhea?
Differential  Other causes:
diagnosis Hyperthyroidisms: heat intolerance
Stress? What do you do for life? Any stress? Does the diarrhea with
stress? How about your mood?
Infectious: travel / camping / with whom do you live? Any other person
at home with diarrhea?
HIV if risk factors
Diet: Celiac disease / a lot of dairy products / lactose intolerance / lots of
juice / sugars
Medications: antibiotics / stool softeners
 Rheumatic diseases: red eyes / mouth ulcers / skin changes/ rash / nail
changes / hx of psoriasis / joint pain / swelling / back pain / stiffness
(especially in morning) / discharge / renal stones
PMH
FH
SH

Bloody diarrhea DD:


GE (gastroenteritis)
IBD (inflammatory bowel disease)
Bleeding peptic ulcer

Investigations for clostridium difficile Treatment for clostridium difficile


 CBC / differential / lytes and chemistry  Stop the antibiotic
 Stool culture for parasites  Metronidazole (500 mg tid x 7 days)
 Stool assay for clostridium toxin  If metronidazole is not effective or severe
 Endoscopy case  vancomycin (125 mg qid x 14 d)
 Blood grouping and cross matching

OSCE-guide-III.doc Page 29 of 255


History taking Medicine

Diarrhea CHRONIC

The same as acute diarrhea, except the impact and red flags

Introduction
CC
Analysis of OsCfD  How many times?
CC COCA  What bout during night?
B/Mucous Yes  organic
No  irritable bowel syndrome (IBS) day only
How does if affect your sleep?
 Consistency: watery / loose / formed / bulky. Any floating
fat droplets / difficult to flush / undigested food
 Did you notice blood? When did it start?
Before you have your bowel move?
Mixed (higher source of bleeding)?
On the surface?
AS  Pain  OCD / PQRST
If pain improves after bowel movement: IBS
 Vomiting
 Alteration with constipation
Impact  Acute dehydration: thirsty / dizziness / light headedness / LOC / weak
 Chronic  weight loss
Red flags  Constitutional symptoms for infection / cancer
 For cancer: Age / family hx of Ca colon / change in the calibre of stools /
what kind of diet
Differential  Rheumatic diseases: red eyes / mouth ulcers / skin changes/ rash / nail
diagnosis changes / hx of psoriasis / joint pain / swelling / back pain / stiffness
(especially in morning) / discharge / renal stones
 Other causes:
Hyperthyroidisms: heat intolerance
Stress? What do you do for life? Any stress? Does the diarrhea with
stress? How about your mood?
Infectious: travel / camping / with whom do you live? Any other person
at home with diarrhea?
HIV if risk factors
Diet: Celiac disease / a lot of dairy products / lactose intolerance / lots of
juice / sugars
Medications: antibiotics / stool softeners
PMH
FH
SH

Rheumatic diseases: IBS / ankylosing spondylitis / psoriasis / reactive arthritis

OSCE-guide-III.doc Page 30 of 255


History taking Medicine

Counselling:
Explanation:
o From what you have told me, the most likely explanation for your diarrhea is the
medical condition known as Irritable Bowel Syndrome, it is like unhappy colon
o What do you know about IBS?
o We do not know the exact mechanism behind this disease, and it is a common
condition, a lot of people have it, this is a long term disease, but it is treatable.
o What I need to do is to do physical exam, and do some blood works and stool
analysis to rule out other causes, how do you think about that?
o Is it serious condition doctor?
 It is not serious, as it does not affect life expectancy, and around 80% of
patients improve over time
Management:
o Psychotherapy:
 Establish good relationship with the patient
 CBT (cognitive behavioural therapy)
 If mood is low  depression counselling, it might be a mood problem
o Life style modifications:
 Stress management and relief
Relaxation techniques such as meditation
Physical activities such as yoga or tai chi
Regular exercise such as swimming, walking or running
 Diet modification: lactose-free diet or a diet restricting fructose is sometimes
recommended
 If drinks too much alcohol  advise to decrease alcohol
o Medications
 Abdominal pain:
Hyoscyamine (antispasmodic): 0.125 to 0.25 mg PO or SL q4h or PRN
/OR/ extended-release tablets: 0.375 to 0.75 mg orally every 12 hours
(do not exceed 1.5mg in 24 hours)
Amitriptyline (10 mg qhs)
 Diarrhea:
Imodium up to 8 tab / day
Lomotil
 Constipation:
fibre content in diet
Metamucil (psyllium): bulk-producing laxative and fibre supplement
 SSRIs
o Alternative medicine:
Probiotics
Herbal remedies, e.g. peppermint oil:
Offer more information:
o I will give you some brochures and web sites in case you want to read more
about that
Associated diseases:
- IBS
- Fibromyalgia
- Chronic fatigue syndrome
- Interstitial cystitis

OSCE-guide-III.doc Page 31 of 255


History taking Medicine

ASTHMA
Mr comes to your office as post-ER visit follow-up, he had asthmatic attack three days ago.
He went to ER; he was treated and discharged with advice to see his family physician.
Introduction How do you feel now?
EVENT  O S Cf D
 Which medication was used? How many times did you need to puff?
 Symptoms: SOB / Tightness / Wheezes / Sweating / heart racing /
LOC / did you turn blue? Were you able to talk?
 Did you call 911 or someone called for you? Did they give you meds?
What were these meds?
 Were you admitted to hospital? ER? Did they need to put a tube?
What were the discharge meds?
Asthma history  When were you diagnosed? How? Type of buffers?
 Were you controlled? How many times do you puff (excluding
exercise)? Are you using spacer?
 Recently, did you notice a need to the doses?
 Any attacks during the night?
 Do you use peak flow meter?
 Did you have PFTs (pulmonary function tests) done?
 How many times did you have to go to ER?
Triggers Infection  Recent chest infection? Flu-like symptoms? Fever / chills?
Medications  How do you use puffers? Stored properly? Not expired?
 Did you start new medication? -blockers? Aspirin? Any recent in
dose of these medications?
Outdoor  Exercise
 Cold air
 Pollens (is it seasonal?)
 Dust: construction / smug (smoke/ fog/ exhaust)
Indoor  Do you smoke? Anybody around you?
 Do you have pets? People around you?
 Fabrics related: carpets floor? Any change in linen? Pillows?
Blankets? Mattress? Curtains?
 Relation to any type of food?
 Perfumes
 Do you live in a house (basement  mold)?
 Any construction renovation? Exposure to chemicals?
Stress  Any new stressful situations?
PMH and FH  Skin allergies
 Other allergies

Asthma Management
1- Confirm diagnosis:
Symptoms:
o Cough (dry / more at night / more with exercise / induced by allergens)
o Wheezes (noisy breathing)
o Chest tightness
Examination: wheezes
Diagnosis:
o Chest x-ray: R/O pneumonia / infection / cancer
o Pulmonary Function Tests (PFTs):
 FEV1/FVC < 80% of expected  obstructive lung disease
 Give bronchodilators, repeat PFTs after 20 min, if > 12%  Asthma

OSCE-guide-III.doc Page 32 of 255


History taking Medicine

2- Management:
Environment control: avoidance of irritant and allergic triggers (e.g. avoid smoking /
change -blocker for treatment of HTN)
Patient education: the allergic nature of the disease and triggering factors
Written action plan: see the diagram below (next page)

3- Medications:
Type Symptoms Treatment Notes
Mild < 2 times / week Short acting 2-agonist: Does not need daily
intermittent 1-2 puffs (PRN and medication
before exercise)
Mild > 2 times / week Short acting 2-agonist Low dose ICS LTRAs are second-line
persistent but < 1 time / day (Ventolin 100 mcg 1- (Flovent 125 monotherapy for mild
2 puffs qid) mcg 1 puff bid) asthma
Moderate Daily LABA Moderate dose 6-11 yrs: ICS should be
(Serevent 50 mcg 1 ICS to moderate dose
puff bid) (Flovent 250 > 12 yrs: LABA should be
mcg 1 puff bid) considered first
Severe Continuous / Add LABA or LTRA High dose ICS Oral Omalizumab (anti IgE)
Uncontrolled (Singulair 10 mg PO (Flovent 250 prednisone may be considered in
qhs) mcg 2 puffs patients > 12 yrs
bid)
ICS : Inhaled Corticosteroids; 1 puff = 100 mcg
LABA : Long-acting beta2-adrenoceptor agonist
LTRA : Leukotriene receptor antagonist

Puffers and LTRA tablets:


Medication Color Active ingredient / puff Class Notes
Ventolin Blue Salbutamol 100 mcg Short acting 2-agonist
Serevent Blue Salmeterol 50 mcg LABA Also available as diskus
Flovent Orange Fluticasone propionate 125 mcg ICS
250 mcg
500 mcg
Advair Purple Fluticasone 250 mcg ICS Also available as diskus
Salmeterol 50 mcg LABA
Pulmicort Brown Budesonide 100 mcg ICS
200 mcg
400 mcg
Symbicort Red Budesonide 400 mcg ICS
Formoterol 12 mcg LABA
Atrovent Green Ipratropium bromide 20 mcg Anti-cholinergic For Asthma and COPD
Spiriva Spiriva Tiotropium bromide 18 mcg Long-acting, 24 hrs, anti- For COPD
Handihaler cholinergic bronchodilator
Singulair Tablets Montelukast 4 mg Leukotriene receptor
10 mg antagonist (LTRA)

4- Emergency treatment of Asthma:


Oxygen, aim for SaO2 > 94%
Bronchodilators:
o Beta 2 agonists: salbutamol 100 mcg 4 puffs q 15-20 min x 3 AND
o Ipratropium bromide 4 puffs q 15-20 min x 3
Corticosteroids: hydrocortisone 200 mg IV or prednisone 40-60 mg PO
Rehydration: aggressive IV fluids to liquefy bronchial secretions

OSCE-guide-III.doc Page 33 of 255


History taking Medicine

Canadian Thoracic Society Asthma Management Continuum 2010

Asthma action plan:

OSCE-guide-III.doc Page 34 of 255


History taking Medicine

COPD management

Prolong survival  Smoking cessation


 Vaccination: influenza virus, pneumococcus (Pneumovax)
 Home oxygen: to prevent cor pulmonale and decrease
mortality if used > 15 hrs/day (indications: PaO2 < 55
mmHg; or < 60 mmHg with cor pulmonale or
polycythemia)
Bronchodilators Short acting Ventolin (q6h PRN) + Atrovent (1-2 puffs q6h)
Long acting  LABA (Serevent 50 mcg/dose) Atrovent /or/
 LACA (Spiriva): 18 mcg qAM + must stop Atrovent
Respiratory rehabilitation
Corticosteroids Inhaled Combination of ICS + LABA
 Advair: Fluticasone / Salmeterol (250mcg/50mcg) od or
bid
 Symbicort: Budesonide / Formoterol (400mcg/12mcg) 2
puffs bid
Oral Short course of oral corticosteroids: 50mg/d x 5 days

COPDE (COPD exacerbation):


 Definition: episode of increased dyspnea, coughing, increase in sputum volume or purulence
 Etiology: viral URTI, bacteria, air pollution, CHF, PE, Ml must be considered
 Management:
Assess ABC, consider assisted ventilation if decreasing LOC or poor ABGs
Supplemental O2 (controlled FiO2: target 88-92% SaO2 for CO2 retainers
Bronchodilators by nebulizer
o Short acting beta2-agonists used concurrently with anti-cholinergics
o Salbutamol and Ipratropium bromide via nebulizers x 3 back-to-back
Systemic corticosteroids: IV solumedrol or oral prednisone (50mg/d x 5 days)
Antibiotics: often used to treat precipitating infection:
o Indications (2 out of 3): increased SOB, increased sputum amount, or increased
sputum purulence (change in colour, e.g. greenish)
o Type of antibiotics: see below
Post exacerbation: rehab with general conditioning to improve exercise tolerance
 ICU admission
For life threatening exacerbations
Ventilator support
o Non-invasive: NIPPY, BiPAP
o Conventional mechanical ventilation

Antibiotics:
Outpatient: resp fluoroquinolones: levofloxacin 750 mg PO q24h x 5 days OR
beta-lactam + macrolide (amoxicillin 1000 mg PO tid + clarithromycin 500 mg PO bid)
Risk factors (group home / hospital infection / immunocompromised):
ceftriaxone (1 g IV q24h) + azithromycin (500 mg IV q24h x 5 days).
Step-down to oral therapy when tolerated
Susceptible for pseudomonas / recent use (within 3 months) of antibiotics or cortisone:
piptazo (piperacillin / tazobactam); 3.375 gm IV q6h)
MRSA: Vancomycin 1 gm IV q24h

OSCE-guide-III.doc Page 35 of 255


History taking Medicine

Ankle swelling Bilateral

Introduction
CC uni- vs. bi- lateral
Analysis of OsCfD  What ? Walking / standing what ? Raising legs
CC  How high does it go?
AS  Local symptoms:
Pain / fullness / heaviness / tightness
Skin changes (redness / swelling / do you feel your feet warm?)
Nail changes
 Other swellings in your body:
How about swelling in your face? Eye puffiness? Do you find it
difficult to open your eyes in the morning?
How about your belly? Did you need to the size of your belt?
Hands, did you feel it is tight to wear your ring?
Impact  How does this affect your life?
Red flags  Constitutional symptoms for infection / cancer
Differential Differential diagnosis of BILATERAL ankle swelling:
diagnosis  Failure Heart
 Failure Liver
 Failure Kidney: history of kidney disease (changes in urine / bruising /
frequency / burning / frothy urine / clear or no)
 Hypoalbuminemia
 Thyroid diseases
Specific cause within this system (e.g. kidney)
 Hx or Dx of DM
 Any medications (penicellamine, gold, NSAIDs, )
 Recent sore throat
 Any skin infection / rash
 Hx of autoimmune disease
 How about diet? Is it balanced? Any diarrhea?
PMH
FH
SH

Case: patient with face swelling, BP 150/90, protein in urine, ketones, no blood, no glucose, no
WBCs
Diagnosis: nephritic syndrome (minimal changes)
Investigations:
Kidney function tests / urinalysis / 24 hrs protein in urine / renal biopsy
Lipid profile / blood glucose studies
Hepatitis B serology / ANA / C3 and C4
Management:
Salt restriction / avoid fats
Diuretics / monitor fluids in and out
Anti-HTN: ACE inhibitors
Prednisolone

OSCE-guide-III.doc Page 36 of 255


History taking Medicine

Ankle swelling Unilateral

Introduction
CC uni- vs. bi- lateral
Analysis of OsCfD  What / ?
CC  How high does it go?
 If pain  PQRST
AS  Local symptoms:
Pain / fullness / heaviness / tightness
Skin changes (redness / swelling / do you feel your feet warm?)
Nail changes
 Other joints? Toes? Other ankle?
Impact  How does this affect your life?
Red flags  Constitutional symptoms for infection / cancer
Differential Differential diagnosis of UNILATERAL ankle swelling:
diagnosis  Any trauma, any twist in your ankle?
 Gout; previous attacks, screen kidney  for kidney stones
 Infection, sepsis, cellulitis; fever, pus, discharge, tenderness
 Gonorrhea septic arthritis; Sexual history, penile discharge? Unprotected sex
recently?
 DVT
Specific cause within this system (e.g. gout)
 Tell me more about your diet? Too much protein?
 How about alcohol?
 Medications? Pain meds (aspirin) / diuretics (furosemide, thiazides)?
 Hx of cancer / chemotherapy (cytotoxic drugs) / radiation?
 Family hx of gout / kidney stones?
PMH
FH
SH

DVT: see the physical examination section

OSCE-guide-III.doc Page 37 of 255


History taking Medicine

Congestive heart failure CHF

68 years old man comes to ER with 4 weeks of SOB

Analysis of Clarification 1- When do you say SOB; what do you mean? Cardiac or chest?
CC Is it difficult to breathe in and out?  cardiac / anemia
Is it difficult to breathe out?  COPD / asthma
2- Do you have any hx of asthma? Lung disease?
Any wheezes? Chest tightness? Cough?
3- Do you have any hx of heart disease?
No  newly dx
Yes  ? acute on top of CHF
Any racing heart? Dizziness? LOC? Any hx of HTN?
OSCfD  Is it first time? Or you had it before? When and how were you
PQRST diagnosed? How about treatment?
 Is it related to activity? How many blocks were you able to walk?
And now?
 How about at rest? And at night?
Impact  Left ventricle:
SOB? How many pillows do you use?
Do you wake up at night gasping for air?
Cough / crackles?
 Right ventricle:
Any swelling in your LL? How high does it go? Related to position / standing?
Weight gain?
Eye puffiness? Swollen face? Pain on the liver?
 Other cardiac symptoms:
Chest pain? Nausea/vomiting? Sweating?
Heart racing / dizziness / LOC? Do you feel tired?
Red flags  Constitutional symptoms for infection / cancer
 Risk factors for ischemic heart diseases IHD
DD Causes (that precipitated acute on top of CHF):
 Compliance
 Diet
 Medical
PMH DM / Kidney / Liver diseases
FH HTN / heart attacks
SH SAD

DD (Causes that precipitated acute on top of CHF):


 Compliance:
Are you receiving treatment? Which medications do you take? How much? For how long? Any
change in medications? Change in dose?
Do you take it on regular basis? Any chance that you may skip one or more doses?
Do you take it by yourself or do you need help?
Did you get your Digoxin level measured before / recently?
Did you start new medication? Rx or (OTC) over the counter? e.g. indomethacin
How about water pills?
Are you under regular F/U? How often? When was the last time? Were you symptoms free at that
time?
 Diet:
Do you have special diet? Salt-free diet? Do you monitor that?
Any new changes in diet?
Any chance of salty food, e.g. pickles, canned food, dried meet and fish

OSCE-guide-III.doc Page 38 of 255


History taking Medicine

 Medical:
Do you take medications on regular basis? Any new medication? Advil?
Any hx of thyroid dx, any sweating / diarrhea?
Any hx of heart disease / HTN ( A Fib) / heart attack / CAD (ischemia) / did you feel your
heart bouncing (arrhythmias)? Any congenital or valvular disease / Chest pain / tightness /
dizziness / light headedness / LOC?
Any chest / lung disease (wheezes, cough, chest tightness)
Any kidney disease? Renal failure?
Any bleeding? Anemia?

New York Heart Association functional Classification of heart failure:


 Class I: ordinary physical activity does not cause symptoms of HF
 Class II: comfortable at rest, ordinary physical activity results in symptoms
 Class III: marked limitation of ordinary activity; less than ordinary physical activity  symptoms
 Class IV: inability to carry out any physical activity without discomfort; symptoms may be present at rest

Investigations:
 Labs: CBC / lytes / ABG (arterial blood gases) / glucose / INR / PTT / serial cardiac enzymes (q8h x 3)
/ ECG / fluid balance
 Chest x-ray findings of CHF: (1) Enlarged heart, (2) Upper lobe vascular redistribution, (3) Kerley B
lines (thin linear pulmonary opacities caused by fluid or cellular infiltration into the interstitium of the
lungs), (4) Bilateral interstitial infiltrates, (5) Bilateral small effusions

Treatment:
Acute heart failure:
o Treat acute precipitating factors (e.g. ischemia. arrhythmias)
o L Lasix (diuretics)  pre-load (furosemide: 40-500 mg IV)
o M Morphine; 2-4 mg IV decreases anxiety and preload (venodilation)
o N Nitrates (venous and arterial dilator  kidney perfusion)
o O Oxygen
o P Positive airway pressure (CPAP/BiPAP) decreases preload and need for
ventilation / Position (sit patient up with legs hanging down unless hypotensive)
o In ICU or failure of LMNOP: sympathomimetics (dopamine or dobutamine)
Chronic heart failure (long term management):
o ACEI (slow progression and improve survival) or ARBs (if ACEI not tolerated)
o Beta blockers: slow progression and improve survival
 Should be used cautiously, titrate slowly because may initially worsen CHF
 Side effects: fatigue / bradycardia
 If pt on -blockers  exacerbation  stop the -blockers for 2 days
o Digoxin (if A Fib OR symptomatic on ACEI)
o Diuretics: symptom control, management of fluid overload; furosemide 80 mg OD
(furosemide opposes the hyperkalemia induced by beta-blockers, ACEIs)
Spironolactone for class Ill-b and IV CHF already on ACEI and loop diuretic
If still uncompensated: Implantable Cardioverter Defibrillator (ICD)
o Anti-arrhythmic drugs: for use in CHF with arrhythmia can use amiodarone, beta-
blocker, or digoxin
o Anticoagulants: warfarin for prevention of thromboembolic events

Digoxin overdose:
Anorexia, nausea, vomiting
Bradycardia, dizziness, LOC
ECG: PVC, heart block
Vision: yellow hallos around objects

OSCE-guide-III.doc Page 39 of 255


History taking Medicine

Heart racing

For few weeks


Introduction But first I would like to ask you, at the moment, how do you feel?
CC
Analysis of Clarification When do you say your heart is racing; what do you mean?
CC Do you feel your heart is going fast
Or is skipping beats
Can you tap it for me please?
It sounds irregular for me!
OSCfD  Is it first time? Or you had it before? When and how were you
PQRST diagnosed? How about treatment?
 Is it related to activity? How many blocks were you able to
walk? And now?
 How about at rest? And at night?
 When was the last attack? And what is the duration of the
longest attack?
 Is it related to caffeine, chocolate, coke, any other type of
food?
 SAD (cocaine or any other stimulant)
Impact  CVA (any weakness / numbness / difficulty finding words / visual problems)
 Heart failure (SOB / limitation of activity / swelling in your legs / how many
pillows do you use??
 Other cardiac symptoms:
Chest pain? Nausea/vomiting? Sweating?
Heart racing / dizziness / LOC? Do you feel tired?
Red flags  Constitutional symptoms for infection / cancer
 Risk factors for ischemic heart diseases IHD
DD  Do you take medications on regular basis? Any new medication?
 Any hx of thyroid dx, any sweating / diarrhea?
 Any hx of heart disease / HTN ( A Fib) / heart attack / CAD (ischemia) /
did you feel your heart bouncing (arrhythmias)? Any congenital or valvular
disease / Chest pain / tightness / dizziness / light headedness / LOC?
 Any chest / lung disease (wheezes, cough, chest tightness)
 Any kidney disease? Renal failure?
 Any bleeding? Anemia?
PMH DM / Kidney / Liver diseases
FH  Family history of sudden death at a young age? (cardiomyopathy)
 HTN / heart attacks
SH SAD
Physical  Vitals
examination  Cardiac exam (looking for mid-diastolic, mitral stenosis, rumbling character)
 Thyroid exam
 Neurological exam: brief / gross motor and reflexes

OSCE-guide-III.doc Page 40 of 255


History taking Medicine

The patient daughter has a concern: my mother was diagnosed with AF,
Should I worry about this?
1. This is a reasonable concern?
 AF may lead to embolic event (CVA)
 AF may lead to heart failure
 AF may lead to V. Fib
2. However, this is not uncommon condition, and it is treatable with medications

Causes for AF: Causes for TACHYCARDIA


1- IHD 1- Exercise
2- Hypertension / CHF 2- Pregnancy
3- Hyper-thyroidism 3- Caffeine / stimulants
4- Medications (e.g. digoxin, some anti- 4- Anemia
arrhythmic meds class I) 5- Hypovolemia
5- Electrolytes imbalance 6- Fever
6- Too much alcohol (holiday heart) 7- Stress
7- Cardiomyopathy 8- Smoking
8- Valvular heart diseases 9- Hyper-thyroidism
9- Congenital heart diseases 10- Pheochromocytoma
10- Loan AF
11- Myocarditis
12- COPD / pneumonia
13- Cardiac surgery
14- Cocaine
15- Any condition that lead to tachycardia in a
susceptible person

Atrial fibrillation
Stable Unstable

< 48 hours > 48 hours  SOB


If in doubt  TEE  Rate control:  BP < 90/60
-blockers  Chest pain
Ca ch blockers (diltiazem, verapamil)  Confusion
Digoxin (in patients with heart
failure)
 Anti-coagulation (3 weeks prior and 4
weeks after cardioversion)

Cardioversion:
 Electrical: 150 joules for A Fib (50 joules for A Flutter)
 Pharmacological: procainamide; 1 g / 1 hr infusion

Anti-coagulation:
Assess stroke risk: determine CHADS2 score in patients with non-valvular AF
Risk factor Points CHADS2 score Anti-coagulation
CHF 1 0-1 Aspirin 81-325 mg daily
Hypertension 1 2 moderate risk Warfarin
Age > 75 yrs 1 factors or any high risk
Diabetes 1 factor (prior stroke,
Stroke / TIA 2 TIA or embolism,
mitral stenosis,
prosthetic valve)

OSCE-guide-III.doc Page 41 of 255


History taking Medicine

Fall

Orthostatic hypotension
76 years old male patient came to clinic because he fell few days ago. He was getting out of bed,
when he fell to the ground

Introduction  Did you hurt yourself? How do you feel?


HPI: analysis of CC  Was this the first time, or did it happen before?
 Was it related to Emotions? Coughing? Urination?
Associated During  Did you lose conscious? Did you hit your head?
Symptoms  Were you alone or with someone? Did your wife describe it to
you? Is she with you? If it is ok with you, after we finish, I would
like to speak with her to get some info.
 Were you shaking? Certain part of your body or whole?
For any  Were you breathing? Did you turn blue?
Fall,  Did you bite your tongue? Roll your eyes? Wet yourself?
LOC Before  Were you able to take few steps or did you fall immediately
or seizure (orthostatic hypotension)?
 Before you lose consciousness, did you feel:
Dizzy, lightheaded, nausea, vomiting, any chest pain, heart
racing (cardiac)
Things are spinning around you (vertigo)
Weakness, numbness, vision changes (stroke)
Any flashing light, strange smells (epilepsy)
Sweaty, shaky, hungry (hypoglycemia)
After  How long did it last?
 How did you regain consciousness? By yourself or did you need
intervention?
 After you regain consciousness; were you able to recognize the
surroundings? Able to talk? Able to move?
 Did you feel any weakness, numbness?
Impact  Did you hurt yourself? How do you feel now?
Red flags  Constitutional symptoms
 Risk factor for IHD
Any geriatric  Balance
patient; ASK about:  Vision
 Hearing
 Urination
DD  Diseases: arrhythmia / CAD, CVA, seizure, hypoglycemia
(already analyzed before the event)
 Environment: is your room well lit? Any chance you tripped?
 Do you take any medications? Do you have a list?
Go through it one by one
Which one was added / changed recently?
Each medication: ask about the disease, when started?
Any OTC? Aspirin? Who prescribed it to you?
 Are you getting enough fluids

OSCE-guide-III.doc Page 42 of 255


History taking Medicine

Counselling:
 Inform the patient
The most likely explanation to what happened is a condition called postural orthostatic
hypotension. It means drop in the blood pressure with change of posture.
Explain the pathophysiology:
o When we change position from lying or sitting to standing, blood tends to pool in
the lower extremities, and this leads to drop in blood pressure. Normally, blood
vessels in our body react by narrowing in order to prevent this and to maintain
normal blood pressure.
o In patients having orthostatic hypotension, and this could be due to age /
medications / DM or combination, their blood vessels fail to react fast enough,
this leads to pooling of blood in lower extremities  amount of blood reaching
to heart  blood reaching the brain  they end-up losing their consciousness
temporarily.
Consequences: this might happen again
Investigations:
o Blood works / CBC / differential / lytes / kidney and liver function tests
o ECG
 Preventive measure:
Contact the psychiatrist to check the poly-pharmacy, to discuss with him the possibility
of decreasing the dose or changing medications.
Meanwhile, if you are changing positions, do this slowly, on steps, e.g. from lying down,
sit for a couple of minutes on the bed before standing up, and before you stand up, push
your feet against the ground for few seconds.
 I will give you brochures and web sites in case you need to read more.

Notes:
 The patient will have a list of medications:
Lipitor
Hydrochlorothiazide  ask about fluids intake
-blocker
Aspirin  ask about bleeding
Lorazepam
Oxazepam  I can see that you are taking 2 sleeping pills, who prescribed them to you?
The same doctor or no?
Metformin
B12 / B complex

 If the patient looks sad / depressed  you look down for me, any chance you are depressed

OSCE-guide-III.doc Page 43 of 255


History taking Medicine

Peripheral vascular disease:

Calf pain / swelling

Introduction
Analysis of  OsCfD
CC  PQRST
P: unilateral or bilateral
R: what about other joints, knees? Thighs? Feet?
 What or : did you notice that your pain while walking up or down hill?
while walking uphill: peripheral arterial disease
while walking downhill: spinal stenosis
 Is it first time? Or you had it before? When and how were you diagnosed?
How about treatment?
 Is it related to activity? How many blocks were you able to walk? And
now?
 How about at rest? And at night?
 When was the last attack? And what is the duration of the longest attack?
Impact  History of strokes / TIAs / neurological symptoms
 Chest pain / SOB / heart racing
 Pain after eating (intestinal ischemia)
 Effect of pain on daily activities / work?
 Leriche syndrome (aorto-iliac occlusive disease): numbness in buttocks &
thighs / absent or decreased femoral pulses / impotence
Red flags  Constitutional symptoms for infection / cancer
 Risk factors for ischemic heart diseases IHD
Smoking? How much and for how long?
High blood pressure? For how long? Controlled or not?
Diabetes mellitus
Cholesterol measured? When? What was it?
DD Peripheral Arterial Disease versus Spinal Canal Stenosis
Vascular symptoms Neuro symptoms
 Cold feet / ulcers  Weakness / numbness / tingling
 Swelling / redness  Back trauma / back pain
 Delayed wound healing  Sexual dysfunction / difficulty with
 Nail changes / hair loss erection
PMH Past history of heart disease / stroke / symptoms of stroke / DM / Kidney / Liver
diseases
FH Family history of heart disease / HTN / heart attacks
SH SAD

N.B. the 6 Ps of ischemia:


Pallor / Pain / Parathesia / Paralysis / Pulseless / Polar (cold)

OSCE-guide-III.doc Page 44 of 255


History taking Medicine

Urinary symptoms:

 Obstructive (prostatic disease in  anuria):


Difficulty to initiate urine? Do you need to strain?
Any changes in the stream?
Any dripping?
After you pass urine, do you feel that you emptied your bladder completely or do you
need to go again?
 Irritative (frequency  UB disease):
How many times do you go to the washroom?
o How about before? Any change?
o How about during night time? How does this affect your sleep? How about your
concentration and mood?
Do you need to rush to washroom? Are you able to make it all the time?
Have you ever lost control or wet yourself?
Any burning sensation? Any flank pain?
Fever / night sweats / other constitutional symptoms
 Urine analysis (changes):
COCA + B (content: frothy / cloudy / not clear)

Summary of irritative symptoms: FUND


Frequency / Urgency / Night time / Discomfort

OSCE-guide-III.doc Page 45 of 255


History taking Medicine

Anuria
Introduction Empathy how do you feel right now?
Analysis of CC  OsCfD
 PQRST
 What or
 Is it first time? Or did it happen before? When and how were you diagnosed? How
about treatment?
Associated symptoms:
 Obstructive symptoms
 Irritative symptoms
 Urine analysis (changes): COCA Blood
Local symptoms:
 Any problems with passing stools? What? When?
 Any masses in the groin / pelvic mass / pain?
 Abdominal pain? Distension?
Impact Metastasis  Back: pain / weakness / numbness
 Liver: yellow / itchy / urine / stools
 Lungs: cough / phlegm / hemoptysis
 Brain: headache / nausea / vomiting
Renal failure Generalized swelling / face puffiness / itching
Sexual Sexual dysfunction
Red flags  Constitutional symptoms for infection / cancer
 Risk factors for cancer prostate / bladder
Were you ever diagnosed with prostate disease? Screened for prostate
diseases? (DRE or PSA)
Family history of prostate disease / cancer?
Ca bladder (radiation / exposed to chemicals / aniline dye)
Smoking? Alcohol?
DD  Renal stones: Have you ever had a renal stone? Any history of colicky pain in
flanks? Have you ever passed a small crystals or stone during voiding? Hx or
repeated UTIs?
 Medications: glaucoma / anti-psychotic meds / anti-cholinergic drugs; like those
used for incontinence; e.g. Ditropan (Oxybutynin), Detrol (Tolterodine)
 2 Neuro:
Back problem: trauma metastasis cauda equine (spoiled himself with stools
/ buttocks numbness)
Stroke (diagnosed / weakness / numbness / difficulty)
 2 Cancer:
Cancer prostate
Ca bladder (hematuria)
PMH AMPLE
FH DM / anemia / polycystic kidney disease / renal stones
SH SAD
Most likely diagnosis: BPH
Other possible diagnoses: UTI / prostatitis / Ca prostate
Investigations: urea & creatinine / urinalysis / renal US / DRE & PSA / TRUS
If cancer is suspected: bone scan / CT
Treatment:
Watchful waiting: may resolve spontaneously
Medical treatment: -adrenergic antagonists (doxazosin, terazosin) / 5--reductase
inhibitors (finasteride)
Surgery: open surgery / TURP / minimally invasive (stent / laser ablation / cryosurgery)

OSCE-guide-III.doc Page 46 of 255


History taking Medicine

Hematuria
Introduction Empathy how do you feel right now?
Analysis of  OsCfD
CC  Timing:
Initial versus terminal or total
Diurnal variation
 What or
 Painful or Painless
 Is it first time? Or did it happen before? When and how were you
diagnosed? How about treatment?
Associated symptoms:
 Obstructive symptoms  prostate disease
 Irritative symptoms  UB disease
 Urine analysis (changes): COCA Blood
Local symptoms:
 Any problems with passing stools? What? When?
 Any masses in the groin / pelvic mass / pain?
 Abdominal pain? Distension?
Impact Metastasis
Renal failure Generalized swelling / face puffiness / itching
Sexual Sexual dysfunction
Red flags  Constitutional symptoms for infection / cancer
 Risk factors for cancer prostate / bladder / RENAL
Were you ever diagnosed with prostate disease? Screened for prostate
diseases? (DRE or PSA)
Family history of prostate disease / cancer?
Family history of cancer bladder or kidney?
Ca bladder (radiation / exposed to chemicals / aniline dye)
Smoking? Alcohol?
DD  Renal stones: Have you ever had a renal stone? Any history of colicky pain
in flanks? Have you ever passed a small crystals or stone during voiding?
Hx or repeated UTIs?
 Medications: blood thinners / aspirin / bleeding from other sites?
 Pseudo-hematuria:
Diet: eating too much beet
Medications: Rifampicin
Other bleeding: bleeding per rectum / vaginal bleeding
PMH  AMPLE
 History of hemolytic anemia / polycystic kidney
FH DM / anemia / polycystic kidney disease / renal stones
SH SAD
Investigations:
(1) Kidney: urinalysis (casts / crystals / C&S / cytology) / ultrasound (abd/pelvic) / IVP / KFTs
(2) Bladder: cystoscopy
(3) Prostate: PSA / TRUS
(4) Others: CBC / differential / INR
Case: patient on warfarin for A. fib for 2 yrs; went to walk in clinic for sore throat and was prescribed
Biaxin, developed hematuria. Diagnosis: coagulopathy.

OSCE-guide-III.doc Page 47 of 255


History taking Medicine

Renal stones

Risk Factors
Hereditary: RTA, G6PD, cystinuria, xanthinuria, oxaluria, etc.
Dietary excess: Vitamin C, oxalate, purines, calcium
Dehydration (especially in summer months)
Sedentary lifestyle
Medications: thiazides
UTI (with urea-splitting organisms)
Hypercalcemia disorders: hyperparathyroidism, sarcoidosis, histoplasmosis, etc.

Investigations
Screening labs
o CBC: elevated WBC in presence of fever suggests infection
o Electrolytes, Cr, BUN  to assess renal function
o Urinalysis: R&M (WBCs, RBCs, crystals), C&S
Imaging
o Kidneys, ureters, bladders (KUB) x-ray to differentiate opaque from non-opaque
stones (e.g. uric acid, indinavir) / 90% of stones are radiopaque
o CT scan: no contrast; distinguish radiolucent stone from soft tissue filling defect
o Abdominal ultrasound: may demonstrate stone (difficult for ureters) / may
demonstrate hydronephrosis
o IVP (not usually done): anatomy of urine collecting system, degree of
obstruction, extravasation
Cystoscopy for suspected bladder stone
Strain all urine  stone analysis
If recurrent stone formers, conduct metabolic studies
o Serum electrolytes, Ca, PO4, uric acid, creatinine and urea
o PTH if hypercalcemic

Approach to renal stone:

OSCE-guide-III.doc Page 48 of 255


History taking Medicine

Treatment Acute:
Medical:
o Analgesics (Tylenol #3)
o NSAIDs help lower intra-ureteral pressure
o antibiotics for UTI
o (antiemetic + IV fluids) for vomiting
Interventional:
o Ureteric stent (cystoscopy)
o Percutaneous nephrostomy (image-guided)
Admit if necessary:
o Intractable pain
o Intractable vomiting
o Fever (? infection)
o Compromised renal function
o Single kidney with ureteric obstruction / bilateral obstructing stones

Treatment Elective:
Medical:
o Conservative if stone < 5 mm and no complications
o Fluids to increase urine volume to > 2 L/day (3-4 L if cystine)
o Specific to stone type:
 Calcium oxalate stones: thiazides / potassium citrate (alkalinization of urine)
 Calcium struvite: antibiotics for 6 wks (stone must be removed to treat infection)
 Uric acid: allopurinol / potassium citrate (alkalinization of urine to pH 6.5 to 7) /
shockwave lithotripsy not effective
 Cystine: alkalinize urine (bicarbonate / potassium citrate) / penicellamine / captopril
(forms complex with cystine) / shockwave lithotripsy not effective
Interventional:
o Procedural / surgical: If stone is > 5 mm or presence of complication
o Kidney
 Extracorporeal shockwave lithotripsy (ESWL) if stone < 2.5 cm
 Percutaneous nephrolithotomy; indications:
+ Size > 2.5 cm + Staghorn + UPJ obstruction
+ Calyceal diverticulum + Cystine stones
o Ureter
 ESWL is the primary modality of treatment
 Ureteroscopy (extraction or fragmentation) if failed ESWL / Ureteric stricture
o Bladder
 Transurethral cystolitholapaxy
 Remove outflow obstruction (TURP or stricture dilatation}

Management of UTI:
Investigations:
o Urine for culture and sensitivity
o Blood: CBC / differential
o Imaging (if suspect complicated pyelonephritis or symptoms do not improve with 72
hours of treatment): Abd/pelvic U/S / IVP / Cystoscopy / CT
Pregnant: amoxicillin 500 mg TID x 7 days
Non-pregnant:
o Septra (sulfamethoxazole and trimethoprim) DS (800/160): 1 tab bid x 7 days
o /OR/ Ciprofloxacin 500 mg bid x 7 days
Pyelonephritis:
o Ceftriaxone (third-generation cephalosporins): 1 g IV q24hrs x 2 days
o Then continue oral ciprofloxacin x 7 days
Abscess: + drain

OSCE-guide-III.doc Page 49 of 255


History taking Medicine

Incontinence
Obstructive / 62 years old female, with hx of 3 years of urinary incontinence
Introduction Empathy how do you feel right now?
Analysis of  OsCfD
CC  What or : lifting objects / coughing / straining
 Is it first time? Or did it happen before? When and how were you
diagnosed? How about treatment?
Associated symptoms: If at any time there is a frequency or
 Obstructive symptoms some new symptom  analyze it
 Irritative symptoms first then resume!
 Urine analysis (changes): COCA
Blood Frequency in  UTI
Local symptoms:
 Any problems with passing stools? What? When?
 Any masses in the groin / pelvic mass / pain?
 Any perineal skin lesions?
Impact  How does it affect your life? Daily activities?
Red flags  Constitutional symptoms for infection / cancer
 Risk factors (MGOS):
M  Menopausal symptoms, and HRT
 LMP
G  Gynaecological history
 Previous abdominal or pelvic surgeries
O  Obstetric: How many pregnancies? Route of delivery?
S  Sexual: Repeated infections / dryness / dyspareunia
DD  Overflow incontinence
 Urge incontinence
 Detrusor overactivity: CNS lesion, inflammation / infection (cystitis),
bladder neck obstruction (tumour, stone)
 Stress incontinence
 Urethral hypermobility: childbirth, pelvic surgery, aging
 Intrinsic sphincter deficiency (ISD): pelvic surgery, neurologic
problem, aging and hypoestrogen state
Diagnosis:
History
Urinalysis + C&S (if infection suspected)
Urodynamics
Stress test
Treatment of urge incontinence Treatment of stress incontinence
Bladder habit training Weight loss
Botox (botulinum toxin) injection Kegels exercises
Medications: anti-cholinergics; Bulking agents
Tolterodine (Detrol), Oxybutynin Surgery (slings, TVT / TOT4, artificial
(Ditropan), TCAs sphincters)
N.B. Causes of reversible urinary incontinence (DIAPERS): Delirium, Inflammation / Infection, Atrophic
vaginitis, Pharmaceuticals / Psychological, Excess urine output, Restricted mobility, Stool impaction

4
TVT: Tension-free Vaginal Tape, TOT: Trans Obturator Tape

OSCE-guide-III.doc Page 50 of 255


History taking Medicine

Lump Neck Swelling

Introduction
Analysis of CC:  Can you point to it?
The lump  OSCfD / Anything special at that time? Fever? Rash?
 Is it painful? PQRST
 Can you estimate its size for me? Is it like a lent, olive, lemon, or
larger? Did it change in size? How fast was the change in size?
 Did you try to feel it? Does it feel soft / rubbery / hard?
 Do you feel it is fixed or moving?
 Any skin changes? Redness? Ulcers?
 Any history of trauma?
 Is it the only one? Did you notice other lumps in your body? How about
other side of your neck? Arm pits? Groins?
Associated (local)  Rule out infection: Any recent flu-like symptoms? Do you feel tired/
symptoms fatigue? History of sinusitis/ Pain in your face? Runny nose?
Pain/discharge in ears? Any sore throat/ oral ulcers/ tooth pain?
Difficulty swallowing? Neck stiffness/pain? Headache? Vomiting?
 Thyroid (if central): heat vs. cold intolerance / sweating / hand shaking
/ heart racing / diarrhea vs. constipation
Impact  How does this affect your life?
 Do you feel tired? ? HIV
 Easy bruising? Repeated infections? ? Leukemia
Red flags  Constitutional symptoms
 Bone pains / Tender points
HEAD SSS  risky behaviour:
 A: includes recent travel
 SAD: how about injection drugs? Did you share needles?
 Sexual hx: Detailed (safe sex, last time, how many partners). Did you
notice any vaginal discharge/ bleeding? Any pain/ blisters/ warts?
Discoloration/ itchiness?
Differential HIV / Lymphoma / Leukemia / Infectious mono-nucleosis
Diagnosis
PMH History of cancer
FH History of cancer / lymphadenopathy
Physical exam  Vital signs
 Neck exam / Thyroid exam if the swelling is central
 LNs / Lymphatic system / LNs in groin / pelvic exam
 Liver / Spleen

Notes:
Whenever there is IV drugs  screen for liver symptoms / HIV
Whenever there is risk for STIs  screen for liver symptoms and PID

OSCE-guide-III.doc Page 51 of 255


History taking Medicine

Lump Breast
Introduction
Analysis of CC:  Can you point to it?
The lump  Is it one breast or both? Where did you notice it? You can ask
verbally, is it LT / RT? Upper / Lower? Outer / Inner? How about
the other breast?
DO NOT POINT WITH YOUR HANDS OR FINGERS!
OSCfD / Anything special at that time? Fever? Rash?

Is it painful? PQRST

Can you estimate its size for me? Is it like a lent, olive, lemon, or

larger? Did it change in size? How fast was the change in size?
 Did you try to feel it? Does it feel soft / rubbery / hard?
 Do you feel it is fixed or sliding (moving)?
 Any skin changes? Redness? Ulcers?
 Any history of trauma?
 Is it the only one? Did you notice other lumps in your body? How about
your neck? Arm pits? Groins?
 Is it related to your period? Does it change with the period?
 Any nipple changes? Discharge? Bleeding? Itching?
Associated (local)  Rule out infection: Any recent flu-like symptoms? Do you feel tired/
symptoms fatigue?
Impact  Headache/ vomiting?
(consequences of  Back pain/ weakness/ numbness/ tingling in arms or legs?
cancer:  Chest pain/ cough/ phlegm/ wheezes/ heart racing?
metastasis)  Liver: yellow discoloration/ itching/ urine/ stools?
Red flags  Constitutional symptoms
 Bone pains / Tender points
Risk factors of cancer: MGO
 Menstrual history: first period / last period / regular?
 G: OCPs?
 Obstetric: History of pregnancies? Number of pregnancies? First
pregnancy at what age?
 Breast feeding?
 Diet rich in fat
 PMH or FH of cancer breast / ovarian carcinoma
Differential  Benign disease
Diagnosis  Trauma  fat necrosis
PMH History of cancer breast / ovarian carcinoma
FH History of cancer breast / ovarian carcinoma
DD for Breast Mass:
Breast Cancer Sclerosing adenosis
Fibrocystic changes Lipoma
Fibroadenoma Neurofibroma
Fat necrosis Granulomatous mastitis (e.g. TB,
Papilloma / papillomatosis sarcoidosis)
Galactocele Abscess
Duct ectasia Silicon implant
Ductal / lobular hyperplasia

OSCE-guide-III.doc Page 52 of 255


History taking Medicine

 Galactorrhea (prolactinoma): normal prolactin level: 10-20 ng/ml (non-pregnant)


 If < 40 ng/ml: follow-up
 If > 40 ng/ml: CT and bromocriptine
 If progressive / headache / affecting vision  surgery
 Benign Breast Lesions:
Non-proliferative lesions:
o Aka fibrocystic changes, chronic cystic mastitis, mammary dysplasia
 Benign condition characterized by fibrous and cystic changes in the breast. No risk
of breast cancer / Age 30 to menopause / pre-menstrual.
 Breast pain, focal areas of nodularity or cysts often in upper outer quadrant
 Treatment: Evaluation of breast mass and reassurance / Analgesia (ibuprofen, ASA) /
If > 40 years old: mammography every 3 years or biopsy
Proliferative lesions:
o Fibroadenoma:
 Most common benign breast tumour in women under age 30
 Risk of subsequent breast cancer is increased only if fibroadenoma is complex, there
is adjacent atypia or a strong family history of breast cancer
 Clinical features: nodules: smooth, rubbery, discrete, well-circumscribed, non-
tender, mobile, hormone-dependent. Needle aspiration yields no fluid
 Investigations: Core or excisional biopsy required
 Treatment: Generally conservative; serial observation. Consider excision if size 2-3
cm, rapidly growing on serial US, if symptomatic or pt preference
o Intra-ductal Papilloma
 Solitary intra-ductal benign polyp
 Present as nipple discharge (most common cause of spontaneous, unilateral bloody
nipple discharge), breast mass, nodule on U/S
 Treatment: excision of involved duct to ensure no atypia
Other lesions:
o Fat Necrosis
 Uncommon, result of trauma (may be minor commonly a tight bra, positive history
in only 50%), after breast surgery (i.e. reduction)
 Firm, ill-defined mass with skin or nipple retraction, tenderness
 Regress spontaneously, but complete imaging biopsy to R/O cancer
o Mammary Duct Ectasia
 Obstruction of a subareolar duct  duct dilation, inflammation, and fibrosis
 May present with nipple discharge, bluish mass under nipple, local pain
 Risk of secondary infection (abscess, mastitis)
 Resolves spontaneously
o Abscess
 Lactational vs. periductal / subareolar
 Unilateral localized pain, tenderness, erythema, subareolar mass, nipple discharge,
nipple inversion
 Rule out inflammatory carcinoma, as indicated
 Treatment: initially broad-spectrum antibiotics and I&D (incision and drainage), if
persistent total duct excision (definitive)
 If mass does not resolve: fine needle aspiration (FNA) to exclude cancer, U/S to
assess for presence of abscess
 Breast Cancer:
1/9 women in Canada will be diagnosed with breast cancer in their lifetime
Risk factors:
 Prior history of breast cancer
 1st degree relative with breast cancer (greater risk if relative was premenopausal)
 Increased risk with high breast density, nulliparity, first pregnancy >30 years old,
early menarche (< 12 yrs), late menopause (> 55 yrs), >5 years HRT
 Decreased risk with lactation, early menopause, early childbirth

OSCE-guide-III.doc Page 53 of 255


History taking Medicine

Investigations
o Mammography
 Screening: every 1-2 years for women age 50-69 / If positive family history in 1st
degree relative: every 1-2 years starting 10 years before the youngest age of
presentation
 Diagnostic: investigation of patient complaints (discharge, pain, lump)
 Follow-up after breast cancer surgery
 Findings indicative of malignancy: mass that is poorly defined, spiculated border,
micro-calcifications, architectural distortion, normal mammogram does not rule out
suspicion of cancer based on clinical findings
o Other radiographic studies:
 Ultrasound differentiates between cystic and solid
 MRI high sensitivity, low specificity
 Galactogram / ductogram (for nipple discharge): identifies lesions in ducts
 Metastatic workup as indicated (usually after surgery or if clinical suspicion of
metastatic disease) bone scan, abd U/S, CXR, head CT
Diagnostic Procedures
o Needle aspiration: for palpable cystic lesions; send fluid for cytology if blood or cyst
does not completely resolve
o Fine needle aspiration (FNA): for palpable solid masses; need experienced practitioner
for adequate sampling
o U/S or mammography guided core needle biopsy (most common)
o Excisional biopsy: only performed as second choice to core needle biopsy; should not be
done for diagnosis if possible
Genetic Screening: consider testing for BRCA 1/2 if:
o Patient diagnosed with breast AND ovarian cancer
o Strong family history of breast / ovarian cancer (e.g. Ashkenazi Jewish)
o Family history of male breast cancer
o Young patient ( <35 years old)
Pathology
o Non-invasive: ductal carcinoma in situ (DCIS): completely contained within breast ducts,
often multifocal / 80% non-palpable, detected by screening mammogram.
 Treatment: lumpectomy with wide excision margins + radiation OR mastectomy if
large area of disease, or high grade
o Invasive:
 Invasive ductal carcinoma (most common 80%): hard, infiltrating tentacles
 Invasive lobular carcinoma (8-15%): 20% bilateral. Does not form micro
calcifications, harder to detect mammographically (may benefit from MRI)
 Paget's disease (1-3%): ductal carcinoma that invades nipple with scaling,
eczematoid lesion
 Inflammatory carcinoma (1-4%): ductal carcinoma that invades dermal lymphatics,
most aggressive form of breast cancer.
Clinical features: erythema, edema, warm, swollen, tender breast lump
Peau d'orange indicates advanced disease (III-b IV)
Treatment of breast cancer:
Stage Primary treatment options Adjuvant systemic
therapy
0 (in situ) BCS + radiotherapy None
I BCS (or mastectomy) + axillary node dissection + May not be needed
II radiotherapy Chemotherapy and /
III mastectomy + axillary node dissection + radiotherapy or hormone therapy
Inflammatory
IV Surgery as appropriate for local control
BCS = breast-conserving surgery

OSCE-guide-III.doc Page 54 of 255


History taking Medicine

Dizziness

Causes Vertigo Non-vertigo

Peripheral Central Cardiac Non-cardiac


Benign paroxysmal Stroke Arrhythmias Vaso-vagal
positional vertigo TIAs CAD / MI episode
Mnire's disease Brain tumour CHF Panic attack
Vestibular neuritis MS Aortic stenosis Somatization
Labyrinthitis Cerebellar lesion Postural
Acoustic neuroma hypotension
Symptoms EAR Brain / Neuro
Imbalance Mild-moderate Severe
N&V Severe Variable
Auditory Common
Neurological Common
Nystagmus Unidirectional Bidirectional
(horizontal or (horizontal or
rotatory) vertical)

Clarification Do you feel


 You have blackout (syncope)
 OR the room is spinning around you (vertigo)
Analysis of CC  OCD
 Timing: when / frequency
 What or : certain position
Impact  Did you lose consciousness?
 Did you fall to ground? Did you hit your head?
Red flags Constitutional symptoms
DD Syncope  Vasovagal attack: LOC / while straining or urinating / nausea / do you feel
warning signs before the dizziness?
 Cardiac (tight AS / arrhythmia): heart racing / chest pain / immediate (no
warning signs)
 Hypotension: antihypertensive meds; change of dose or new medication
 Postural hypotension: diabetes / dehydration / parkinsonism
 Neuro (stroke / TIAs): vision changes / loss, speech impairment, weakness
Vertigo Condition Duration Hearing loss Tinnitus Other features
Benign Paroxysmal Seconds to minutes Certain
Positional Vertigo positions
Nystagmus
Mnire's disease Minutes to hours Fluctuating + Ear fullness
Vestibular neuritis Hours to days Unilateral
Labyrinthitis Days Unilateral Whistling Recent AOM
Acoustic neuroma Chronic Progressive + Ataxia
CN VII palsy
Cerebellar  Risk factors for CAD: HTN / DM / cholesterol / smoking
lesion  Unbalanced gait
 Aspirin / blood thinners
 Alcohol: neuropathy / cerebellar degeneration
PMH / FH / SH

OSCE-guide-III.doc Page 55 of 255


History taking Medicine

Condition Management
Benign Acute attacks of transient vertigo lasting Reassure patient that process resolves spontaneously
Paroxysmal seconds to minutes initiated by certain Particle repositioning manoeuvres: Epleys manoeuvre
Positional head positions, accompanied by torsional (performed by MD) OR Brandt-Daroff exercises
Vertigo (rotatory) nystagmus (performed by patient)
Diagnosis: Surgery for refractory cases
(BPPV) Anti-emetics for nausea/vomiting
History
Positive Dix-Hallpike manoeuvre Drugs to suppress vestibular system delay eventual recovery
and are therefore not used
Mnire's Episodic attacks of tinnitus, hearing loss, Acute management may consist of bed rest, anti-emetics,
disease aural fullness (pressure / warmth), and anti-vertiginous drugs (betahistine)
vertigo lasting minutes to hours Long term management may include:
 Medical: (1) Low salt diet, diuretics
(hydrochlorothiazide), (2) Local application of
gentamicin to destroy vestibular end-organ, results in
complete SNHL, (3) Betahistine (Serc) prophylactically
to decrease intensity of attacks
 Surgical: selective vestibular neurectomy or
transtympanic labyrinthectomy
Must monitor opposite ear (bilaterality in 35% of cases)
Vestibular Acute onset of disabling vertigo often Acute phase:
neuritis accompanied by nausea, vomiting and  Bed rest, vestibular sedatives (Gravol), diazepam
imbalance without hearing loss that Convalescent phase:
resolves over days leaving a residual  Progressive ambulation especially in the elderly
imbalance that lasts days to weeks  Vestibular exercises: involve eye and head movements,
sitting, standing, and walking
Labyrinthitis Acute infection of the inner ear Investigations:
resulting in vertigo (days) and CT head
hearing loss If meningitis is suspected: lumbar puncture, blood cultures
May be serous (viral), or purulent Treatment:
(bacterial) IV antibiotics
Occurs as complication of acute and Drainage of middle ear
chronic otitis media, bacterial mastoidectomy
meningitis and cholesteatoma
Acoustic Schwannoma of the vestibular portion of Investigations:
neuroma CN VIII MRI with gadolinium contrast is the gold standard
Audiogram SNHL (sensori-neural hearing loss)
Vestibular tests: normal or asymmetric caloric weakness (an
early sign)
Treatment
Expectant management if tumour is very small or in elderly
Definitive management is surgical excision
Other options: gamma knife, radiation
Dix-Hallpike Positional Testing: the
patient is rapidly moved from a sitting
position to a supine position with the
head hanging over the end of the table,
turned to one side at 45 holding the
position for 20 seconds. Onset of
vertigo is noted and the eyes are
observed for nystagmus

OSCE-guide-III.doc Page 56 of 255


History taking Medicine

INR Counselling

Analysis History / Give the information


Impact DVT relapse / Bleeding
Red flags Female: OCP / pregnancy / LMP / vaginal bleeding

DVT Causes / Complications


Warfarin / Blood thinners
Decision

Conclusion Offer brochures, support. If you have time: SAD / PMH

 Analysis:
History:
o Why are you doing this INR?
o When were you diagnosed? How?
o Were you admitted through the ER or outpatient?
o Was there any involvement of your lungs?
o Which medications were you taking?
o Do you measure your INR regularly? When was the last time? What was the result? What
is your target INR?

Give the information: Your measurement today shows INR of 1, any idea why?
o Compliance: Are you still taking your warfarin? On regular basis? Did you stop your
medication? Why?
o Forget: Do you take your medications on your own, or does someone else help you? Any
chance that you missed a dose?
o New medications: Did you start a new medication? What? Why? When?
o Diet: Do you eat a lot of spinach? Or dark green vegetables? (rich in vit K)

 Impact:
Now, I would like to ask you some questions to check if you have relapse of your DVT or bleeding,
then we will go from there
DVT relapse:
Because you stopped your medication, I would like to make sure that there is no relapse
o DVT: Have you had any pain / swelling / redness in your calf muscles?
o PE: Have you had any SOB, chest pain, heart racing?
o Stroke: Any confusion? Vision changes? Difficulty finding words? Weakness?
Bleeding:
o Did you notice any bleeding?
o Did you notice bleeding from your gums / nose / coughing or vomiting blood / bruises in
your body / dark urine / urine in stools?
o Any weakness / numbness / difficulty finding words / vision difficulty?
o Did any one tell you that you look pale? Do you feel fatigued?

Based on what you have told me, there are no obvious serious consequences, if it is ok with you, we
can discuss your situation now!

 Red flags: for FEMALES


Are you taking any OCPs? OCPs might increase the risk of developing DVT.
Are you pregnant? Warfarin is not to be used during pregnancy; we will use heparin instead of it.
Have you had any vaginal bleeding?

OSCE-guide-III.doc Page 57 of 255


History taking Medicine

 What is your understanding about DVT?


Causes:
o It might happen after prolonged sitting without movements (like very long flights)
o Or due to certain medical condition,
The blood tends to form clots in the deep veins of the lower extremities
Complication:
o Relapse 8%: without treatment, and that is concerning!
 Whenever we treat the patient, our target is to decrease the relapse rate to 0.8%
which is 1/10 of the risk without treatment
o These clots are not fixed, and sometimes they get dislodged from your leg and travel
along your blood vessels, all the way to the lungs (chances are 3%):
 If large enough  might cause sudden death
 If showers of small clots  you may not feel anything right now, but it later will
cause what we call pulmonary HTN, which is a debilitating disease, with serious
consequences and we do not have treatment for it right now
o Always in medicine, we try to balance the benefits and the side effects, and in this
condition, the benefits largely outweigh the risks.

 Now, what do you know about blood thinners?


o It is a medication used to make our blood thin, preventing our body from forming clots
by competing with vitamin K, which is needed for the formation of the elements of blood
clots.
o We take warfarin seriously, and that is why we monitor it closely and regularly, by
assessing the INR which is an indicator of the effect of warfarin. Therefore, as long as
your INR is within your target, the risk of bleeding is less than 1%, and almost near 0%
to have intra-cranial (brain) hemorrhage without having external bleeding first. That is
why you need to keep monitoring yourself, and seek medical attention if you notice any
signs of bleeding.

 Decision:
If the patient decides that he will restart the treatment:
We will do it the same way as we did the first time:
o We will start heparin and warfarin together then stop heparin after 3 days
We will need to measure the INR daily (till we reach our target) then twice a week, then weekly,
then every 2-4 weeks

NOTES:
Numbers to remember:
o Relapse (recurrence) of the DVT: 8% without treatment and 0.8% with treatment.
o Possibility of DVT  clots and PE: 3%
o Chances of having bleeding with warfarin: 1%, and almost near 0% chance of having
intra-cranial bleeding without having an extra-cranial bleeding.
The initial DVT counselling should have been done in the first time, when the patient was
diagnosed; which includes:
o General knowledge about DVT
o Causes and risk factors
o INR follow up
My best friend was taking warfarin, and he had brain hemorrhage!
o I am sorry to hear that, this must be stressing / worrying, especially that you are taking
the same medication and he is a close friend to you.
o We prescribe warfarin for many reasons, the issue here is that your friend was not my
patient, and I do not know about his condition, so I am not in a position to comment on
this situation.
o I am glad you came here today, so that we can discuss this together.

OSCE-guide-III.doc Page 58 of 255


History taking Medicine

Patient is receiving blood counsel for adverse reactions

You were called to assess a patient who is receiving blood, and the nurse has concerns.

Adverse effects of blood transfusion:


 Febrile reaction: most common / not serious
 Anaphylactic reaction: not common / serious
 Haemolytic reaction: not common / serious

Introduction to nurse / what is your concerns / ethical challenge


Introduction to patient / ethical challenge
 ABCD
 History
 Brief physical exam
Adverse reactions of blood transfusion
Plan

 Introduction to nurse / what is your concerns / ethical challenge


Good afternoon, I am Dr May I get your name please?
How can I help you? OR What are your concerns?
o There is a mistake!
What do you mean by mistake? How is the patient doing?
o This was wrong blood!
Did you stop the transfusion?
o Yes
That is great, this is the first step in the right direction
Now, what do you mean by wrong blood? Is it the same bld group or no? Cross matched or not?
Do we have the patient name on the units?
o It is the same group but with other patient name
o Doctor, please do not tell the patient!
Why? Do you have any concerns?
o I might be fired!
I see you have concerns here, but we need to stabilize the patient first. Then we will speak about
that. However, we need to investigate before making decisions.
Can you tell me when did this happen? How much did he receive?

 Introduction to patient / ethical challenge


Good afternoon Mr , I am Dr The nurse was updating me about your condition. It looks like
there was an unintentional medical error took place, and I need to make sure you are ok.
o Is it serious doctor?
Could be! There are different possibilities; I need first to check you.
o Whose mistake is this? Is it the nurse mistake?
Usually in the blood transfusion process, there are many steps; any one of those might go wrong.
It is early now to judge. I need first to make sure you are ok and stable, and then I will file an
incidence report. Investigations will be done, and you will be informed with the results.

 ABCD
Let us make sure you are safe and stable first.
AB:
Can you please open your mouth? Mouth is clear with no swelling. Do you have any itchiness or
swelling in your mouth?
Trachea is central, no engorged jugular veins. Can I listen to your heart please! Normal heart
sounds.

OSCE-guide-III.doc Page 59 of 255


History taking Medicine

C:
Can I know the vitals please? Normal / stable.
Can you remove the blood unit please, and send it to the blood bank. We need to re-cross this
patient blood with this unit.
Can you put another IV line please! We need to take samples for: CBC / differential / lytes / blood
grouping and re-crossing / haptoglobin / bilirubin level
D:
I am going to shine light in your eyes!
Can you hold my fingers please? Do not let go.
Do you feel me touching you?
Patient is grossly neurologically free.

If fever: give 2 tablets Tylenol 325 mg


Can you please prepare:
o Allergic reaction: Benadryl
o Anaphylactic reaction: Benadryl / Epinephrine / Steroids
o Haemolytic reaction: Diuretics and fluids

 History
Now, I would like to ask you some questions:
Why are you taking blood? They have found that I have anemia
Did you take blood before? Or is this the first time?
Do you feel warm? Shivering? Chills?
Do you feel any itching or swelling in your lips / mouth?
Any heart racing? SOB? Wheezes? Dizziness?
Any flank pain? Back pain? Weakness?

 Brief physical exam


No IV line oozing / No hives on skin / No mouth swelling
Listen to heart / lungs  clear
Press on flanks / spine  no tenderness

 Adverse reactions of blood transfusion


Blood transfusion is a commonly used procedure, and it is life saving. A lot of precautions are
taken to make sure it is completed safely. However, like any other medical intervention, it has
some side effects.
The most common reaction that might happen is called febrile reaction. This is not serious
reaction and it is self limited. It might happen again, so if it happens, next time we give you
Tylenol before the transfusion.
Another adverse effect, which is less common but more serious, it is called anaphylactic
reaction. This is a form of severe and serious sensitivity reaction, in which the blood pressure
drops suddenly, and there is a swelling of the tongue, lips, and mouth, with difficult breathing. We
do not have a method to predict it. However, based on your symptoms, your physical exam and
vital signs it is less likely you have that.
The third adverse effect is called hemolytic reaction, and it happens if the patient receives blood
that belongs to another blood group. It causes damage to blood cells which leads to back pain and
flank pain, and could have serious consequences.
Again, based on your symptoms, physical exam and vital signs, your condition does not cope with
this reaction too. And the fact that you received blood from the same blood group makes it less
likely you will have hemolytic reaction.
We prepared medications to deal with any reaction and we will keep you for a while to monitor
you, to make sure that will not happen.

 Plan
Call the blood bank to withhold the other units (previously cross-matched)
File an incident report

OSCE-guide-III.doc Page 60 of 255


History taking Medicine

Counselling Ventilator

Mr Johnson is 75 years old gentleman, his life-long wife for 50 years has a terminal COPD, with
severe pneumonia, and she is on ventilator for the last 3 weeks, and it is not possible to wean her
from ventilator, you called him to inform him about the condition.

 What do you know about your wifes condition?


Listen carefully
Show understanding and empathy
 Give information about her condition,
Give clear simple information
Stress on the progressive, irreversible nature of the disease

 Give alternatives:
Remain on ventilator, with no evidence that she will be able to breath by own, and with
the possibilities of fatal complications like infections, bed sores, Some people does not
like to have this quality of life
Stop the ventilator and she will pass away in peace
 As regarding her condition now, have you ever discussed this with her? Has she ever
expressed her wishes about what would she like to be done to her if she needs to be
resuscitated or put on ventilator? Does she have any advance directives or living will?
 What do you think about this now?

 Offer time if he needs to discuss it with other close family members, or if he needs to arrange
any thing (e.g. I am just giving you information, and we can arrange a meeting with the
family within 2 days so that I can explain to them).

 What if she does not want to be on ventilator but he would like to leave her on the ventilator?
Mr Johnson, I am sorry to tell you that, actually it is not our decision or your decision, it
is her choice. And she expressed her wishes before; she decided that she does not want to
have this poor quality of life. We have to respect her wishes.

OSCE-guide-III.doc Page 61 of 255


History taking Medicine

Ethical questions

 Patient has the right to access his/her medical file, we can not withhold it
 Patient wants to leave you as family physician  it is his right, and he/she has the right to
take all his/her medical data and file
 If you want to terminate a patient from not seeing you as family physician:
o Give proper notice period
o See him/her for emergency
 Confidentiality; when to break confidentiality? To report for the ministry of transportation for
example:
o Dementia / delirium
o Vision problems
o Seizure disorders
o Schizophrenia (case-based)
o Heart attack  1 month not allowed to drive
o Alcoholic with liver failure (based on Childs criteria: albumin / ascites / INR /
bilirubin)
 Report for child safety  CAS (Children Aid Society)
o Even if POTENTIAL or SUSPECTED
o Child neglect / abuse
 Patient wants to leave hospital against medical advice; e.g. patient has just had a heart attack,
and still insists to leave the hospital!
o I would like to make sure he is competent, not under influence of alcohol or any
substance, and to rule out suicidal ideation
o I would explain to the patient: diagnosis / treatment / side effects of treatment /
complications of not receiving treatment / alternatives
o I will document this, and I will ask the patient to sign a LAMA (leaving against
medical advice), and I will let him go
 Biological parent wants to know the medical details of his/her son, who is adopted by another
family!
o In order to determine whether I should release any information or no, I would
like first to know who has the legal custody (guardian) of this child. It might be
the adopting father, a social worker (case manager)
 Any unconscious patient  ask for DNR or advanced directives
 MMS exam score < 24  patient is incompetent;
o You have a case of patient, who had surgery, is taking medications, but he
developed delirium post-operative and now he wants to discontinue his
medications  NO; he is delirious, incompetent to change decisions, he already
consented to take the medications before he entered this delirium.
o What if this patient broke his leg; do you want to operate him without consent?
This is a new condition; we do not know what would be his competent wishes 
look for SDM (substitute decision maker).

OSCE-guide-III.doc Page 62 of 255


History taking Medicine

HIV post-test counselling

Usually you are covering for other physician to give the test result which means this is a new pt
to you.
Be sensitive, empathetic, and flexible

 Introduction:
Your Dr. is away, I am covering for him/her, and I have your file with me, I just need to understand the
situation here,
What have you discussed last time?
Why did you ask for the test last time?
Did you feel sick in any way?
Was there anything made you worried about your own health?

 Give the test result:


It is positive, sure about the result, because they do two tests before giving the positive result.
Check with patients reaction: normalize patients feeling; all the feelings you are having now
are very normal.
What kind of thoughts are going through in your mind? What concerns you the most right
now?
AIDS: dont have AIDS, will not die tomorrow or so. Prognosis is variable, but many people carry
it without feeling it for quite long time, years

 Consequences of HIV:
Repeated infections / LNs
Tired / fatigue
Memory dementia
Depression

 Causes of HIV:
SAD shared needles
Sexual:
o Risky behaviour
o Confidentiality how to inform the partner?
 Get the background info: duration of the relationship, how close to each other,
 Partner has to know: Risk of infection / Needs to be tested
 Will know anyway, either from public health or him. Prefer him to tell, offer help to
tell.

 Before discharge the patient:


Arrange follow-up visit in the next couple days after patient digests the info
Make sure that patient is safe to go home, safe to drive back, no suicidal or homicidal ideation,
inquire about support system.
Education: emphasize the importance of safe sex: advice use barrier contraceptive methods all the
time with all partners in the future to prevent the transmission.

Case: HIV patient with diplopia  cranial nerve examination


CT brain: enhanced multiple rings  toxoplasmosis with HIV
DD: TB / toxoplasmosis / CMV / CNS lymphoma
Management: refer to infectious disease specialist

OSCE-guide-III.doc Page 63 of 255


History taking Medicine

Lung Nodule
 Introduction:
Why X-ray was taken? When?
When was last normal X-ray? Do we have it?
 Give the test result:
Solitary Lung Nodule. Definition: a round or oval, sharply circumscribed radiographic lesion, size
up to 3-4 cm, which may or may not be calcified, and is surrounded by normal lung. Can be
benign or malignant
Any ideas about what could be causing this nodule

 Consequences:
Local symptoms: cough, phlegm, haemoptysis, SOB, wheezing
Constitutional symptoms: fever, chills, night sweat / change of appetite, weight loss, fatigue /
pumps or lumps in the neck or elsewhere in the body
Impact / screen for metastasis:
o Brain: headache/ vomiting?
o Back: back pain/ weakness/ numbness/ tingling in arms or legs?
o Lungs: chest pain/ cough/ phlegm/ wheezes/ heart racing?
o Liver: yellow discoloration/ itching/ urine/ stools?

 Causes:
Smoking
Exposure to chemicals / smokes at work
T.B.: Contact with sick person (T.B.) / Recent travel / T.B. skin test
Sarcoidosis: associated symptoms; joint pain, skin rash
History of lung disease
History of cancer
HIV status
Family History of T.B. or Lung cancer

 Management:
Investigations
o CXR: always compare with previous CXR
o CT densitometry and contrast enhanced CT of the thorax
Sputum cytology / stains
TB skin test
o Biopsy: bronchoscopic or percutaneous(CT-guided) or excision (thoracoscopy or
thoracotomy): if clinical and radiographic features do not help distinguish between
benign or malignant lesion
 If at risk for lung cancer, biopsy may be performed regardless of radiographic
features
 If a biopsy is non-diagnostic, whether to observe, re-biopsy or resect will depend on
the level of suspicion
o PET scan not yet routine but can help distinguish benign from malignant nodules
Watchful waiting: repeat CXR and/or CT scan at 3, 6, 12 months
Algorithm: Evaluation of a Solitary Pulmonary Nodule; check previous CXR
o Looks benign or unchanged  repeat CXR q 3-6 months for 2 years
o Significant risk factor on history or looks malignant or changed  CT chest
 Cause (infection or cancer)  stage and treat
 Calcification  observe
 No diagnosis  trans-thoracic needle biopsy
Inflammatory  treat the cause
Cancer  stage and treat
Still NO diagnosis  resect for diagnosis

OSCE-guide-III.doc Page 64 of 255


History taking Medicine

 Past Medical History


 Family history: of T.B. or Lung cancer
 Social History

Evaluation of a Solitary Pulmonary Nodule:

OSCE-guide-III.doc Page 65 of 255


History taking Medicine

High Creatinine

 Introduction:
Why the test was done?
When was the last normal test?
Any idea about the meaning of the test

 Give the test result:


 Consequences: manifestations of renal failure


Nausea and vomiting / stomach pain
Itching
Pallor
Fatigue
S.O.B with exertion
Swelling (ankle, around eye)
Bone pain

 Causes:
Renal:
o Hypertension
o Diabetes
o Repeated kidneys infection
o Poly-cystic kidneys
o Medications: NSAIDs / gold / penicellamine / ACEIs
Post-renal:
o Kidney stones
o Bladder cancer
o Prostate problem

 Past Medical History:


o Kidney disease
o Previous hospitalization
o Nephrectomy
o Allergies
 Family history: of renal problems
 Social History
o Smoking
o Alcohol
o Drugs
o Work
o Home
o Support

OSCE-guide-III.doc Page 66 of 255


History taking Medicine

Impotence / Erectile Dysfunction

 Introduction:
ED is a common problem in men, with a broad DD, encompassing organic & psychogenic causes.
This is often a difficult topic for men to discuss with their doctor. Confidentiality.
Penile erection is a multi-factorial process dependent on integration of neurologic, hormonal,
vascular and emotional factors.
 Analysis of the CC:
Primary vs. Secondary
o Chronology (frequency, onset, duration, course)
 Onset: acute (more likely psychogenic) or gradual (organic)?
 Course: intermittent (more likely psychogenic)? Libido affected?
o Severity or amount? All the time?
o Aggravating / precipitating and alleviating factors
Organic vs. Psychogenic
o Do you have early morning erection?
o Do you have night time emissions?
o Do you have desire?
o Are you able to masturbate to an erection or climax?
o Situational dysfunction; does function vary depending on the setting? Partner / Place /
Time?
 Consequences: How does this affect your life? Your relationship?
 Causes:
Many endocrine disorders and systemic diseases cause ED by influencing libido, autonomic
pathways and/or blood flow.
Organic causes:
o Medical causes: history of DM, HTN, hyperlipidemia, peripheral vascular disease,
intermittent claudication
o Neuro: back trauma / constitutional symptoms (back metastasis) / back pain / weakness,
numbness / history of MS,
o Low testosterone: changes in secondary sex characteristics, e.g. hair pattern changes /
history of gynecomastia / galactorrhea / history of thyroid disease / pituitary disease (
visual defect, headache)
o Medications; e.g. anti-depressants, hormonal treatment, opioids, MAO inhibitors
o SAD: smoking / alcohol / recreational drugs
Psychogenic causes:
o Any problems with their partner(s)
o History / screening of depression
o Any recent changes in life (home, work, socially) / anxiety attacks? Any stress? Past life-
background, upbringing,

Counselling:
Normalize patient feelings
ED can often be improved with:
o Life style modifications: exercise / weight loss / improved diet / DM control / smoking
cessation / alcohol / stress management / anxiety / sleep hygiene
o Improvement of patient relationship with partner: marital counselling / address sexual
boredom / refer to specialist in sexual education and therapy
Unfortunately, many organic causes are irreversible, but we have treatment options:
o Testosterone preparations (if low testosterone)
o Viagra or Cialis
o Penile self-injection
o Vacuum rubber ring device
o Penile prosthesis
Follow-up appointment for BOTH partners

OSCE-guide-III.doc Page 67 of 255


History taking Medicine

Rheumatology History Taking

 Chief complaint: pain / stiffness / weakness / deformity / limitation of movement / joint


clicking
OCD / Acute (< 6 weeks) versus chronic (> 6 weeks)
PQRST /

 Associated Symptoms:
Morning stiffness
o Inflammation: morning stiffness (>30 min), better with use, constitutional
symptoms
o Non-inflammatory: worse with use, worse at end of day, can have some stiffness
but usually not prolonged
Joint swelling / redness
Other Joints / Pattern of joint involvement:
o Mono-arthritis, oligo-arthritis (4 or less), poly-arthritis (5 or more)
o Symmetric vs. asymmetric
o Peripheral joints versus axial involvement (spine, SI joints)
o Small joints (hands / feet) versus large joints (hips / shoulders)
o Additive joints vs. migratory joints
o Tendon involvement
Constitutional symptoms
Extra-articular features:
Seropositive (e.g. RA, SLE, Sjogrens, scleroderma, inflammatory, myositis)
Seronegative (Ankylosing spondylitis, psoriatic arthritis, enteropathic arthropathy,
reactive arthritis)
o Eyes: iritis, scleritis, conjunctivitis, dry eyes
o Oral ulcers
o Respiratory: pleural effusion, pleuritis, pulmonary fibrosis, pulmonary nodules
o Cardiac: pericarditis, pericardial effusion, conduction defects
o GIT: GERD, inflammatory bowel disease, malabsorption, bloody diarrhea
o Dermatology: malar rash, discoid, nodules, telangiectasias, sclerodactyly,
calcinosis, alopecia, periungal erythema, psoriasis, nail pitting, onycholysis,
erythema nodosum, pyoderma gangrenosum
Crystal arthropathies
o Mono-arthritis (red, hot), chronically can be poly-arthritis: gout (tophi, alcohol
history, renal failure, drugs)
o CPPD (hyperparathyroidism, hypomagnesemia, hemochromatosis, Wilsons
disease, hypothyroidism)
Septic arthritis: usually mono-arthritis, fever, red, hot. Gonococcal arthritis can be
migratory, with tenosynovitis and skin pustules

 Disability and adaptation


How does it affect your life? Ask about daily activity!
Effects on ability to work! Support!

OSCE-guide-III.doc Page 68 of 255


History taking Medicine

Multiple Sclerosis

Middle aged man (or woman) with episodes of numbness in one leg.
History: Review of systems
Diagnosis: MS
Investigations: MRI / CSF

 History of present illness (neurological screening)


Ethnic background.
History of weakness: MS can mimic ANY neurological disease process.
o Determine which groups of muscles are involved: proximal vs. Distal, symmetric
vs. Asymmetric
o Pattern of weakness: stepwise decline, relapse and remitting
Dysphagia, dysarthria
Dry eyes, dry mouth
Numbness in upper or lower limbs
History of previous vertigo, loss of vision (optic neuritis), diplopia,
Clumsiness, loss of balance, falls
Any associated sensory symptoms
Bowel or bladder dysfunction (difficulty fully emptying bladder, urgency, incontinence)
Fatigue, arthralgia, depression, behavioural changes
Constitutional symptoms
Impact on daily functioning

Lhermitte's sign: electrical sensation down back on neck flexion


Uhthoff's phenomenon: worsening of symptoms in heat (hot bath, summer)

 Past Medical History


Previous transient focal neurological deficits: vertigo, sensory deficits, etc.
History of headache
Sarcoidosis
HIV status
Systemic inflammatory disease

 Family History: MS, SLE, weakness NYD, visual deficits NYD

OSCE-guide-III.doc Page 69 of 255


History taking Medicine

Obesity

 Analysis of chief complaint:


Weight analysis: now and how about one year ago? Maximum and minimum weights?
Weight loss history:
o Attempts, medically vs. surgically
o What weight achieved, what caused failure
o Assistance: dietition, exercises programs
o Explore why patient want to do it again
o Admire and encourage the pt
General health now:
o HTN, IHD
o DM, cholesterol
o Sleep apnea, asthma
o OA, functional impairment, disabilities, social activities involvement
o Psychological assessment: embarrassment, low self esteem, depression, anxiety
Diet: detailed history; how many meals, how much, bedtime meals,
Exercises: how often, type

 Past medical history: AMPLE,


 Social history: SAD, social activities, home and work environments
 Family history: obesity in the family

Counselling:

 Encouragement: admire patient, it is important for your general health, requires a lot of effort;
it is very difficult process, very common multiple tries.
 Methods:
Set up a goal first, start slowly
Diet: can refer you to a dietition
o Type of food: high fibre, vegetables and fruits, less fat/cholesterol, low
carbohydrate,
o Caloric intake should be calculated /+/ does not exceed 1800 Cal/d
o I will give you tables and graphs to show you the ideal meal composition, but
generally, lunch and supper must be formed of: 50% vegetables and fruits /+/
25% protein /+/ 25% carbohydrates
Exercises:
o Program: 3-5 times per week /+/ 30-50 min each time
o Set up personal instructor to guide
Medications: locally to absorb fats or centrally working on the satiety centre; do not like
to start with
Surgical procedures, in very advanced cases and there is medical impairment, we can
discuss it later.

Avoid: smoking / alcohol /+/ Healthful life style.


Educational information: group program / booklet and brochures
Follow up on regular basis

OSCE-guide-III.doc Page 70 of 255


History taking Medicine

Epilepsy Counselling

 Young 16 yr old male for driving license counsel

Introduction  Why does the patient want a note from doctor for a drivers license?
Usually Dr does not give such note unless there is underlying condition!
Analyze  Age of onset? / When was the diagnosis? / What was the diagnosis?
epilepsy history  How frequently do the attacks occur?
 How long does each attack last? LOC
 Aura prior to attack?
 When was last attack? Similar to previous ones?
 What happens during an attack? Does the patient shake / all over / partly /
roll up eyes/ bite tongue?
 How do you regain consciousness / how do you feel after the attack
Triggers  Which medication does the patient take? Compliance? When was the drug
level checked?
 Any other medications that might interact with epileptic drugs?
 Sleep deprivation / Long screen time before sleep?
 Alcohol? Stimulants?
 Are you under stress
MOAPS Scan the mood and anxiety
HEAD SSS Home / Education / SAD (do you take stimulants)

 Counselling: What is your understanding of seizures?


It is a common condition due to increased electrical activity in the brain, causing abnormal body
movements. Some people lose consciousness, and some do not. Usually, it does not cause learning
disability or brain damage
The diagnosis of epilepsy requires the occurrence of at least 2 unprovoked seizures 24 hours apart
Most of patients have no clear reason to explain why they are having this, but it tends to run in the
family (idiopathic) or structural brain damage (post-meningitis)
Other conditions that should be considered include: Syncope (arrhythmias), Vascular (TIAs),
Metabolic (hypoglycemia), Psychiatric (conversion, panic attacks, malingering)
Prognosis: The patient's prognosis for disability and for a recurrence of epileptic seizures depends
on the type of epileptic seizure and the epileptic syndrome in question. Regarding morbidity,
trauma is not uncommon. Regarding mortality, seizures cause death in a small proportion of
individuals. Most deaths are accidental due to impaired consciousness

 Plan:
Diagnostic workup
Patient education
Treatment
Pregnancy

Diagnostic workup:
o Two imaging studies must be performed after a seizure. They are neuro-imaging
evaluation (MRI or CT) and electroencephalography (EEG).
o Lumbar puncture for CSF examination has a role in the patient with obtundation or in
patients in whom meningitis or encephalitis is suspected.
o Metabolic screen
o Serum studies of anticonvulsant agents (e.g. phenytoin); if therapeutic level but side
effects or poor seizure control  add another drug (carbamazepine / valproic acid)

OSCE-guide-III.doc Page 71 of 255


History taking Medicine

Patient education:
o Dangerous activities: to prevent injury, educate patients about seizure precautions. Most
accidents occur when patients have impaired consciousness. Restrictions apply on:
 Driving (report to ministry of transportation), must be seizures-free for more than 1
year
 Diving, swimming, hiking, mountain climbing
 Taking unsupervised baths, better take shower not bath, with open door
 Working at significant heights, operating machines and the use of fire and power
tools.
o Avoid the triggers for seizure attack:
 Alcohol will exacerbate (chronic alcohol: blood level of anti-epileptics due to
metabolism / excess alcohol: seizure threshold)
 Stress; if the patient is having stress / anxiety / alcohol issues: counsel and offer
social support
 Sleep deprivation / long screen time before sleep
 Head trauma,
 Forgetting to take medication on time
 Taking other medications that interact with the treatment
o Life style:
 You have to take the treatment almost for your whole life
 Talk with your physician about any new medication you want to take
 Medications are teratogenic, females to take proper contraceptive measures
 Patient might choose to wear a bracelet indicating he has epilepsy
 If a seizure will happen: go to the ER
 Regular follow-up visits and monitoring of anti-convulsion level in blood
Treatment:
o The mainstay of therapy for people with recurrent unprovoked seizures is an
anticonvulsant. If a patient has had more than 1 seizure, administration of an
anticonvulsant is recommended. However, standard of care for a single, unprovoked
seizure is avoidance of typical precipitants (e.g. alcohol, sleep deprivation); no
anticonvulsants are recommended unless the patient has risk factors for recurrence
o Medications will be taken for long term, there are many options, will start with one
medication, if no full control, we may increase the dose and/or add another drug
o Side effects of medications: movement disorders (ataxia, dysarthria), teratogenic, liver,
kidney, drowsiness, poor concentration
o Discontinuation: After a person has been seizure free for typically 2-5 years, physicians
consider discontinuing the medication. About 75% of relapses after discontinuation occur
in the first year, and at least 50% of patients who have another seizure do so in the first 3
months. Therefore, patients to observe strict seizure precautions (including not driving)
during tapering and for at least 3 months after discontinuation. Authors recommend that
anticonvulsants be gradually discontinued over 10 weeks
Pregnancy:
 Are you sexually active?
 Do you take use contraception?
o No  Are you planning to get pregnant? Yes! Let us talk about pregnancy and the meds
you will start. Can you postpone the pregnancy for a while? It is better to have good
control of seizures for a while; to get any seizure during pregnancy will pose great risk
for both of you and baby. And the medications can cause serious malformation to the
baby
o Yes is it OCPs? Yes! There might be drug interaction, so for the time being you need
to continue to use your pills and add another method (mechanical) till you contact your
gynecologist

OSCE-guide-III.doc Page 72 of 255


History taking Medicine

Medical note

 For past date!


 For headache

1. What type of doctors note pt wants; reason for the note?

2. Analyze the case  headache case  full history

3. Deal with the patient request:


 Mention that patient is probably suffering from migraine or tension headache.
 Mention that you cant write note for previous visit; but can give copy of todays visit and
notes about what happened;
 Encourage the patient to talk to the tutor (or supervisor) frankly about what happened.
Mention you believe the professor would be reasonable; ask about any possibility to make up
the class he missed;
 Mention you are willing to do any thing to help him out; and apologize that you cant lie or
write a note for the pt you didnt see at that time;
 Mention you are happy to see this patient again to see how things going after he talks to
supervisor and for follow-up regarding his headache.

OSCE-guide-III.doc Page 73 of 255


History taking Medicine

Pre-diabetes Counselling
 What is DM?
o Fasting blood sugar (FBS) > 7 mmol/L
o Random blood sugar (RBS) > 10 mmol/L + symptoms
o Glucose tolerance test (GTT) > 11.1 mmol/L
 What is pre-diabetes? Impaired glucose tolerance
o Fasting blood sugar (FBS) 6.1 6..9 mmol/L
o Glucose tolerance test (GTT) 7.8 11 mmol/L

Introduction  Pre-diabetes: does not mean that you have diabetes, but it shows that you have an
increased chance of having it, about 15% per year. It also shows increased risk of
you having complications in the large blood vessels causing heart diseases, strokes
and peripheral vessel diseases
 Diabetes:
Increase of blood sugar in our blood due to deficient or ineffective insulin.
Explain the role of insulin in helping cells to utilize glucose, two types of DM,
type I and type II.
With one reading we can not say that you are prone or have DM, so let me ask
few questions, to see if you have the symptoms of DM!

Impact Symptoms of  Eat more, drink more, pee more even at night
hyperglycemia  Blurred vision
 Tired / weight loss
 Yeast infections, are there itching / rashes in your groins, in the
toes and finger webs?
 Do your wounds get long time to heal?
Symptoms of N/V, abdominal pain, dehydration, LOC
Ketoacidosis
Symptoms of If patient is on insulin: sweating, shaking, palpitation, fatigue,
hypoglycaemia headache, confusion, seizures
Complications of  Micro-vascular: nephropathy / neuropathy / retinopathy
high blood sugar  Macro-vascular: CAD / peripheral arterial dis / impotence
Red flags  Lifestyle: too much simple sugars, lack of exercises, overweight, family history
 Medications: steroids / beta blockers (-blockers are contraindicated in DM: it causes
hyperglycemia / and it masks hypoglycemia)

PMH Medications: used long term steroids, thiazides, phenytoin, clozapine or other anti-
psychotics, HTN, Cholesterol, CAD, CVD, kidney, hospitalization
FH DM in first degree relatives
SH  Sexual function: any concerns
 Smoking
 From the conversation we had, it looks like you are likely to get DM. However I am going to examine
you and do blood tests (FBS, Hb A1C which shows your blood sugar level over the past 3 months,
lipid profile, micro albumin / Cr ratio, ECG).
 I strongly recommend you to work on lowering your chance of having diabetes by half by: watching
your diet (healthy balanced diet, avoid saturated fats and simple sugars, choose low glycemic content
foods), exercising (30 -45 min of moderate exercise for 4-5 days/wk) and life style changes (limit Na,
alcohol, caffeine, stop smoking).
 I can refer you to diabetes educational program if you wish.
 Treatment targets: Hb A1C < 7 FBS 4 6
Lipids: LDL < 2, Triglycerides < 1.5 or TC/HDL < 4 BP < 130/80

OSCE-guide-III.doc Page 74 of 255


Emergency Medicine

Emergency Medicine

OSCE-guide-III.doc Page 75 of 255


Emergency Medicine

Emergency Room

Trauma Non-trauma

ATLS: Advanced Trauma Cardiac Non cardiac


Life Support protocol
Primary survey: Chest pain: - Acute abdomen for 24 hrs,
- Any trauma (kicked, car - MONA -blockers BP, HR
accident, thrown from - STEMI: catheterization, - Upper / lower GIT
height, ) thrombolytics bleeding
- Manage over the phone: - No ST elevation: heparin, - Severe headache for 2
trauma or meningitis angiography hours
- Secondary survey: patient Arrhythmias: - Seizure for the last 20
in the ER after car - V Fib minutes
accident, primary survey - V Tachy - 16 years brought
was done, do the ACLS: Advanced Cardiac unconscious to the ER
secondary survey Life Support code - Patient receiving blood,
N.B. if knife: leave it in place, Heart block: old patient counsel him
fix with gauze - DNR
- Advanced directive

Management:

Trauma Medical
I I
A A
B
B C
D
OCD
C PQRST

D Associated symptoms
Risk factors
AMPLE PMH
Head to toe Focused physical exam
Management Management

OSCE-guide-III.doc Page 76 of 255


Emergency Medicine

Trauma

I: introduction:
- Because it is a trauma case, I would like to activate the ATLS protocol
- I would like also to get protection for me and my team; gloves, gowns, goggles and
masks

- Mr I am Dr , the physician in charge in the emergency room;


o If the patient is wearing a neck collar, proceed
o Pt is not wearing a neck collar: Mr please do not move your head, nurse can
you please fix his head, we need to put neck collar

- I understand that you are here because you had a car accident
- How are you feeling / doing right now?
o I would like to make sure that you are stable, I will check with the nurse and we
will start the management then I will be asking you more questions.
o I can see that you are in a lot of pain, please bear with me for few minutes, and I
will give pain killer as soon as I can.
o Doctor, where is my wife? How is she doing? Was she with you? I can see that
you are concerned about your wife, I will look for her and I will get back to you
as soon as I can, meanwhile my first concern is to make sure you are stable

We will start the primary survey now:


Airways / Breathing
Mouth / O2 / inspect chest, neck / listen to lungs, heart

Can you please open your mouth? Mouth is clear; no FB, no dentures, no vomitus
Pt is talking to me that means airways are patent
Nurse, what is O2 saturation, plz? Can you give him O2 4 L with a nasal canula
Any change in saturation? Can you plz let me know if
any change in saturation happens!
Inspect the chest By inspection, chest is symmetrical, no bruises, no
open wounds, no paradoxical movements of the chest,
no use of accessory muscles for breathing
Open the collar window, or fix pt head Trachea is central, JV not engorged, bilateral air entry,
and remove anterior part: normal heart sounds (HS) S1 and S2
Trachea Trachea JV Air HS Diagnosis
Jugular veins (JV) entry
shifted Engorged same normal tension
away side pneumo-
Listen to lungs thorax
shifted depleted same normal Hemo-
Listen to heart sounds same side thorax
side
central engorged bilatera muffle cardiac
l d temponad
e
Usually no cardiac temponade in the exam

OSCE-guide-III.doc Page 77 of 255


Emergency Medicine

+ If BP and HR / other signs of tension pneumothorax  nurse, I need to put a large needle
(16 / 14 G) in the 2nd intercostal space at MCL (upper border of the 3rd rib);
Is there any gush of air?
Check the trachea centrality and air entry
We need to put a chest tube in the 5th intercostal space
+ If BP and HR / other signs of hemothorax  nurse, I need to put chest tube in the 5th
intercostal space at anterior Axillary line;
What is the amount of blood?
If > 1.5 L  stat surgery
Otherwise, monitor; if > 200 ml/hr  surgery

Circulation
Vital signs / fluids / withdraw blood samples / look for source of bleeding
Can I get the vital signs please Comment, patient is hypo- / hyper- / tension,
comment on HR, pt is stable / unstable
I would like to have two large IV lines, 16 G in both anti-cubital fossae:
One to start fluids: bolus 2 L ringer lactate or normal saline
The other line is to withdraw samples for: CBC/differential/lytes /+/ blood grouping and
cross matching / and prepare 6 units of blood (4 matched and 2 O) /+/ stat glucose /+/
INR/PTT/LFT /+/ Bun/creatinine /+/ toxic screen/alcohol level /+/ continuous cardiac
monitoring/cardiac enzymes and ECG
Can you please inform me with the vitals; after the bolus fluid is
done and every 5-10 minutes or if there is a change in the vitals
Look for the source of bleeding
Abdomen:  Inspect the abdomen  bruises
I am going to look at and feel your  Palpate the abdomen  rigidity and guarding
abdomen  If positive; I am suspecting intra-abdominal
bleeding, I would like:
To get stat surgery consult
To arrange for FAST (focused abdominal
sonogram for trauma)
To do DPL (peritoneal lavage)
I am going to press on your pelvis  Press from the sides
 Press open book
 If positive; I am suspecting pelvic fracture:
Cut pt sheet and wrap around the pelvis to
support, and check blood on penile meatus
Stat orthopedics consult
Lower extremities By inspection, patient lower extremities are
symmetrical, no abnormal posture or deformity. No
inequality in length, no pain, no deviation
 If positive: I am suspecting femur fracture;
Check the pulses
Thomas splint
Stat orthopedics consult
Log rolling I need more team members to roll the patient on his left
side:
 To check for external source of bleeding
 To press on the spinal processes
 To perform digital rectal exam

OSCE-guide-III.doc Page 78 of 255


Emergency Medicine

I would like to get trauma X-ray series: for neck, chest, LSS and pelvis

D:
D1: Deficits / Disability D2: Detoxification D3: Drugs
Neuro screen /
I am going to shine light in your eyes? Pupils are round, symmetrical and reactive
Can you please squeeze my fingers, do Patient is gross neurologically free
not let them go
Can you wiggle your toes?
Do you feel me touching you here,
here, and here
Glasgow coma scale eyes Alert 4 Pain 2
AVPU Verbal 3 Unresponsiveness 1

AMPLE
A Do you have any allergies?
M Do you take any medications on regular basis?
P PMH, any history of HTN, heart attack, stroke, DM, any long term disease
L  Last meal
 Last tetanus shot
 LMP
E Event:
 Can you describe to me want happened?
 Car accident! Were you the driver or passenger / front passenger?
 Were you wearing your seat belt?
 Did you hit your head? Did you lose your conscious?
 Do you remember what happened, before and after the accident?

Conclusion:
I am suspecting an intra-abdominal bleeding; we are waiting for (surgeon, orthopedics surgeon)
to intervene
Summary:
Introduction to examiner If you are done  go for secondary survey:
Hello  Expose the patient
Neck collar  Examine him head to toe, looking for fractures,
Introduction to patient more detailed neurological examination
A/B / C / order x-rays / D / AMPLE

OSCE-guide-III.doc Page 79 of 255


Emergency Medicine

NOTES:
FLUIDS:
- Trauma / GIT bleeding: we always start with 2 L bolus
o If the patient is stable for the beginning  do not give anything more
o It the patient was not stable, but becomes stable after the first 2L bolus  give
maintenance fluids
o If patient was not stable, and remains unstable  start bld transfusion: 1 unit of
packed RBCs for every 3 units of fluids, and continue till you find source of
bleeding
Stable 2 L bolus Stable Give nothing
Unstable 2 L bolus Stable Give fluids for maintenance
Unstable 2 L bolus Unstable Start blood transfusion 2 RBCs
Then continue 1 (RBCs) : 3 (NS)

- Anaphylactic shock:
o 0.5 L bolus
o Give epinephrine / steroids / anti-histaminics (Benadryl)
- Acute abdomen (pancreatitis / DKA):
o 1-2 L bolus
o Followed by 1 L / hour till the urine output improves
- Heart attack:
o KVO (keep vein open) 100 cc / hour
- If trauma, BP, HR with warm extremities  neurogenic shock (spinal cord injury) 
give only 2 L of fluids then give vasopressors

For the exam:


 Tension pneumothorax
 Hemothorax
 Rupture spleen
 Fracture pelvis
 Fracture femur

OSCE-guide-III.doc Page 80 of 255


Emergency Medicine

Unconscious Patient Neuro


1- Introduction:
- Mr I am Dr I am the physician in charge in the emergency room now,
- Mr Mr if you hear me open your eyes please. Tap on the shoulder, do you hear me
 I would like to activate the ACLS code please / start primary survey
A  Check the mouth, listen for patent air way  Give 4L O2 via nasal
B  What is the O2 please canula
 Trachea central, chest is moving  Monitor O2 for need
 Listen to lungs, heart to intubation
C I would like to get the vital signs please:  2 large IV lines; for
 Based on vitals, no in BP and no in HR (as seen IV fluids and to
in Cushing triad) withdraw samples
 No fever
 No abnormal breathing patterns
D  Mr I will open your eyes, and shine light in it, pupillary reaction  both
pupils are round, symmetrical and reactive
 Mr if you hear me can you please move your eyes up and down  NO
locked-in syndrome
 D3: universal antidotes: thiamine 100 mg
2- Glasgow Coma Scale  if < 8  intubate (ask about DNR)
1 2 3 4 5 6
Eyes No Pain Voice Spontaneou N/A N/A
response s
Verbal No sounds Sounds Words Confused Normal N/A
Motor No Extension Flexion Withdrawal Localizes Obeys
movements (decerebrat (decorticat commands
e response) e
response)
3- Cranial nerve exam: excluding the motor and the sensory:
- Inspection:
o Face is symmetrical, no deviation of the angle of the mouth
o Both eyes are symmetrical, no nystagmus
- I would like to do a fundoscopic examination, to look for disc edema, or retinal
hemorrhage
- Reflexes: Pupillary reflex (II, III) / Corneal reflex (V, VII) / Gag reflex: (IX, X)
4- Upper extremities:
- Inspection: no abnormal posture or contractures
- Tone: check the wrist and elbow in both sides  No cog wheel, no lead pipe rigidity, no
clasp knife spasticity
- Reflexes: Biceps / Triceps
- Check the radial pulse
5- Lower extremities:
- Inspection:
- Tone: check by leg rolling  rigidity, elevate the knee rapidly  spasticity
- Reflexes: Knee reflex / Ankle reflex / I would like to do Babinski reflex / Clonus
- Check dorsalis pedis pulse
- If the neck is cleared (by CT), I would like to do nuchal rigidity, Kernig's sign,
Brudzinski's sign
6- To check for brain death: I would like to arrange for: dolls eyes, caloric reflex test

OSCE-guide-III.doc Page 81 of 255


Emergency Medicine

Unconscious Patient Diabetic

16 years old female found unconscious in her class, next 10 minutes manage and counsel

Introduction:
- Ms I am Dr I am the physician in charge in the ER,
- Ms if you hear me; can you open your eyes please? Tap on the shoulder, do you
hear me  I would like to activate ACLS code please / start primary survey
A  Check the mouth, listen for patent air way  Give 4L O2 via nasal
B  What is the O2 please canula
 Trachea central, chest is moving  Monitor O2 for need to
 Listen to lungs, heart intubation
C  I would like to get the vital signs please: BP and HR
 2 large IV lines; for IV fluids5 and to withdraw samples6
When you ask the nurse for stat glucose by finger prick:
Hypoglycemia Hyperglycemia
 Stat 100 mg thiamine IV  Stat insulin 10 units IV
 Stat 50 ml D50 (Dextrose 50%) IV  Stat 100 mg thiamine IV
 If no IV line  glucagon IM  2 L fluids
At that time, the patient will Orient her; your blood sugar was low, your class-mates
start to regain her conscious brought you here, you are in the ER in hospital, you are
doing well now, how do you feel right now?
Patient states that she is  Reassure her
worried she will lose her  I can help by giving you a doctors note
exam / or other important  This is a very serious condition, you need medical
appointment! attention for some time it is not safe to leave
D D1: Brief neurology  Start D5 (Dextrose 5%): 250 ml / hr
D3: Dextrose  Nurse, I would like to monitor her blood glucose
every 5-10 minutes

+ In case of DKA and physical exam:


History  The same as in hypoglycemia (see below)
 The causes are (5 Is): insulin missed / infection / intoxication / ischemia /
infarction
Physical exam  Brief neuro exam
 Brief DM exam

+ In case of hypoglycemia:
History Are you diabetic?
Analysis Diabetic  When were you diagnosed? And how?
history  Do you take insulin?
 Have you had coma (DKA or hypoglycemia) before?
 When was your last DM follow-up visit? Any reason?
 At that time; were you controlled? Symptoms free?
 When was your last Hb A1C test? What was it?
 How about last few days, were you measuring your glucose?
EVENT  This morning, did you get breakfast, your insulin? Did you check your glucose?
Did you exercise?
 Before you lost conscious, hoe did you feel? Hungry / shaky / dizzy / sweating?

5
If the HR is normal and other VS are normal, you can give only 50 ml/hr to keep vein open (KVO)
But if HR  give 2 L fluids for follow-up
6
For any female patient: -HCG with the blood works you will order

OSCE-guide-III.doc Page 82 of 255


Emergency Medicine

Impact  If pt dx < 5 years  no worry about complications


 If pt dx > 5 years screen for complications (urine changes / visual changes /
numbness and tingling in her feet)
Red flags  Constitutional symptoms
DD  How much insulin do you take? Do you make sure you eat after your insulin
dose?
 Any or in your weight? Stress?
 Did you start new medication?
PMH HEAD SSS /+/ Pregnancy

Counselling:
- What is your understanding about diabetes mellitus?
Pathophysiology:
- It is a condition related to our blood sugar. Whatever we eat, the food contains different
components, including sugar. The food travels through the food pipe to our stomach, to our
bowels where it is absorbed and goes to all our body. Our organs (brain / muscle) use this sugar as
source of energy. In order for muscles to use this sugar, it needs a key to enter into cells, this key
is the insulin.
- We have two types of DM, type I and type II.
- Patients with DM type I, their body does not produce insulin, so we need to compensate for that
by giving it from external source.
Complications:
- High blood sugar is harmful for our bodies, because it affects all our blood vessels, the small and
big ones, and may give a lot of complications! It might cause kidney, eye, or nerves injury and
harm on the longer term.
- On the other hand, low blood sugar is even more dangerous; do you know why? Because our brain
can not survive without blood sugar for more than 5-7 minutes, it is the only source of energy to
our brains.
Prevention:
- What happened to you is a very serious condition, and it might happen again. The best way to treat
is to prevent this from happening; by:
o Make sure that you always eat after your insulin dose
o Monitor your blood sugar frequently
o If you exercise, adjust your insulin dose based on your blood sugar level
- Now, if this happens again, do you know how to identify it before you totally lose your conscious?
o Whenever you feel hungry / sweating / shaky / dizzy / heart racing
o You need to stop, and immediately eat a candy / chocolate / juice
o So, you need to keep glucose tablets in your bag, to take it in case of emergency
If you are at home; keep monitoring your blood sugar,
If you are out; reach to the nearest ER
Emergency measures:
- If you exercise, there is a special type of injections (glucagon emergency kit); if your blood sugar
drops suddenly, use it, or other people can use it to inject you.
- That is why it is important that you have a bracelet that mentions you are diabetic, so if you lose
conscious and some one finds you, they can identify the situation and provide help.
Follow-up:
- You should see your family physician within few days, and he can refer you to diabetes clinic,
for more education and assessment.
- I will still give you some brochures and web sites in case you would like to know more.

Notes: If you are the family physician, what referral will you do for a diabetic patient?
- Diabetes clinic / Foot specialist / Dietician
- If DM type I > 5 years, OR type II at any time: Ophthalmologist / Nephrologist / Neurologist

OSCE-guide-III.doc Page 83 of 255


Emergency Medicine

Unconscious Seizing Patient DT / Epilepsy / Brain tumour /

Patient arrives to the ER with his wife, on the way he had attack of seizures, and received 1 dose
of diazepam, he is unconscious now. In the next 10 minutes; manage.

Introduction Very brief introduction to wife, I will make sure he is stable then I will ask
you more questions
Mr ; Patient is unresponsive, I will start my primary survey:
can you hear me  A: can you open your mouth (open and comment) / trachea central / JV
not engorged
STABILIZE  B: listen to lungs and apex / normal air entry on both sides / normal
heart sounds
 C: can I get the vitals please! Normal! 2 large IV lines please
One to give IV fluids 50 ml/hr to keep vein open (KVO)
The other one is to withdraw samples
 D1; deficits: pupillary reaction
 D3; drugs: universal antidotes  thiamine 100 mg / if O2 is ok, no need
for naloxone, if blood sugar is ok, no need for dextrose
If at any time, the patient starts to seize, give ativan 2 mg IV and reassess ABCD
History Event  First time to seize?
(wife)  Can you describe what happened? Did he fall to the ground?
 Before he seized; did he shout? Starred at the wall? Complained of
strange smell?
 Was all his body seizing or part of it? For how long? Did he bite his
tongue? Rolled up eyes? Did he wet himself? Was breathing?
 Did he regain conscious alone or with intervention?
Cause  History of epilepsy? Medications for epilepsy?
 And mood stabilizers medications?
 RECENTLY, did he complain of: Neurological / Constitutional
symptoms
 Any history of trauma / head injury?
 Recent ear infection?
 SAD: sweating / shaking
 Any medications / blood thinners
PMH  Long term disease; e.g. HTN, DM, kidney, lung, or heart disease
 Previous hospitalization / surgeries?
FH  FH of epilepsy
Examination  Vitals from the examiner
 Glasgow coma scale (if < 8  arrange for intubation)
Neurological examination:
 Cranial nerve examination
 UL and LL: tone and reflexes
Management  Stat neurology consult
 Stat CT brain

OSCE-guide-III.doc Page 84 of 255


Emergency Medicine

N.B. Causes of seizing:


- Epilepsy relapse
- Brain tumour / brain hemorrhage
- Infection / meningitis
- Organ failure
- Electrolytes imbalance
- Delirium Tremens
- Withdrawal from sleeping pills
- Cocaine overdose

How to identify if this is narcotics overdose?


- While doing AB; if O2 saturation is low even after you give O2, and respiratory rate is
low; jump to shine light in the pupils  if pin-point-pupil  give stat naloxone 0.2 2.0
mg IV
- Then back to ABCD

Any unconscious patient:


- A; if fluids  suction
- B; if not breathing  mask and bag
- C
- D
- Then assess Glasgow coma scale, if < 8  intubate (ask for DNR here)

Common 5 causes for unconscious patients:


- Hypoglycemia / DKA
- Epilepsy
- Meningitis
- Stroke lateralization
- Heart block

OSCE-guide-III.doc Page 85 of 255


Emergency Medicine

Heart Attack Chest Pain (MI or Heart Block)


4 cases:
- Chest pain for 45 minutes, with normal ECG
- Chest pain for 45 minutes, with ST elevation
- V. fib: chest pain and the patient is intoxicated
- V. fid: chest pain and the patient is calm

Introduction How are you feeling right now?


Brief history  Os Cf D
 PQRST
 ... position or deep breathing
ABCD  Patient is talking to me  air ways ok.
 Listen to chest to exclude pneumothorax  breathing is ok
 C and D briefly
MONA Oxygen  4 L O2 via nasal canula
Aspirin  Aspirin 325 mg
 Plavix 300 mg
Nitrates  0.3 mg puff X 3 times; 5 minutes apart
- If no low BP
- If the pt did not consume Viagra recently
Morphine  If patient is in pain after 3 puffs of nitrates and there is NO
hypotension or bradycardia  morphine 2 mg
- If antero-lateral infarction (V2/3/4/6 & aVL): 5 mg morphine
- If (II, III, aVF)  usually involves the right ventricle, and the
morphine increase the pre-load  2 mg morphine only
-blockers Given to all patients; EXCEPT:
 History of asthma
 BP, HR, heart block, and inferior MI (II, III, aVF)
 Cocaine (unopposed alpha  blood pressure)  labetalol 20 mg
(2 mg/min for 10 min)
You give metoprolol 2 mg IV
No ST elevation - Stat cardiology consult
- Catheterization lab  angiography
 Heparin (ask about the hospital protocol for heparin usually 5000
U IV then 1000 U/hr by infusion) or give enoxaparin 1 mg/ kg)
ST elevation - Stat cardiology consult
- Catheterization lab  angioplasty
 Heparin (enoxaparin 1 mg/ kg)
 If the cardiologist is not available  start thrombolytics, but first
rule out the contra-indications for thrombolytics:
- 2 recent (< 2 weeks): Recent surgery / Recent bleeding
- 2 brain: Brain tumour / Bleeding in brain stroke
- 1 heart: Aortic dissection

 Chest pain presents with heart racing / SOB / nausea / vomiting / sweating
 History will be: chest pain analysis / cardiac symptoms / risk factors
 If blood pressure is low: we only give oxygen / aspirin / and plavix
 If inferior MI (II, III, aVF)  I need 15 lead ECG / do not give -blockers
 Risk of bleeding with thrombolytics is 1%, but being serious, this needs consent

OSCE-guide-III.doc Page 86 of 255


Emergency Medicine

Case 1: Chest pain with initial normal ECG

Chest pain for 45 minutes, manage

Introduction How are you feeling right now?


Pain history  Os Cf D
 PQRST (R: how about your jaws / back / arms)
 ... position or deep breathing
ABCD  Start O2 4 L by nasal canula
 vitals please: comment and ask for regular VS check
 2 IV lines: KVO / take samples and serial ECG
- ECG normal: order Serial ECG every 5 minutes / if any VS / any
change in pain
- How did you come here today?
- Did you take any aspirin? aspirin 325 mg / plavix 300 mg
- Have you recently used Viagra? If no  nitroglycerine puffs 0.3 mg X
3; 5 minutes apart
 D1: brief neuro exam  pt is grossly neurological free
 Any results back?
 Order chest x-ray?
Brief history  Cardiac symptoms: heart racing / SOB / nausea / vomiting / sweating /
dizziness / LOC
 Chest symptoms
 Constitutional symptoms
 DD: GIT (difficulty swallowing / acidic taste / GERD / heart burn / hx of
PUD) / Trauma / relation to respiration / calf pain / swelling
 PMH: medications / blood pressure / blood glucose / cholesterol / SAD
(especially for smoking and cocaine) / stress
 FH: heart diseases / heart attacks

Another ECG

Normal ECG Do physical exam:


 Vitals / compare BP in both arms ? aortic dissection
 General status of the patient
 Eyes / mouth
 Heart and chest examination
 LL edema

OSCE-guide-III.doc Page 87 of 255


Emergency Medicine

ST elevation  Morphine 2 mg (up to 5 mg in antero-lateral MI)


 -blockers: metoprolol 2 mg IV (if NO contraindications: history of
asthma, BP, HR, heart block, inferior MI, cocaine)
Mr , based on your symptoms and your ECG, it seems that you
have a heart attack. (Am I dying Dr?). This is a serious condition,
however you are stable and doing well, you are in a safe place and
we have experience in dealing with that. Heart attack means one or
more arteries providing blood supply to your heart are blocked by a
clot, we need to reopen this clot.
 Stat cardiology consult
 Do we have catheterization lab?

What we need to do now is to use medication called thrombolytic


or clot buster. I would like first to make sure that you are a good
candidate; recently have you had any surgery or bleeding? Have you
ever had stroke before? Were you diagnosed or told that you have
brain mass? Lesion? Based on the ECG, I do not see signs consistent
with cardiac aneurysm, and based on the BP, it does not look like
having dissection. Based on this, I can say that you are a good
candidate for thrombolytics, it is a life saving procedure, however
like any other medical intervention, it has side effects; it might cause
bleeding in 1% of cases, this includes stroke, so we need your
consent if you would like to have it!
 Heparin (ask about the hospital protocol for heparin usually 5000 U IV
then 1000 U/hr by infusion) or give enoxaparin 1 mg/ kg)
 Can you get the thrombolytics kit please

ST elevation:
 Lateral MI
 Inferior MI

The patient wants to leave the hospital:


- I would like to make sure he is competent, not under influence of alcohol or any
substance, and to rule out suicidal ideation
- I would explain to the patient: diagnosis / treatment / side effects of treatment /
complications of not receiving treatment / alternatives
- I will document this, and I will ask the patient to sign a LAMA (leaving against medical
advice), and I will let him go

OSCE-guide-III.doc Page 88 of 255


Emergency Medicine

Case 2: Chest pain with STEMI


Introduction How are you feeling right now?
Pain history  Os Cf D
 PQRST (R: how about your jaws / back / arms)
 ... position or deep breathing
ABCD  Start O2 4 L by nasal canula
 vitals please: comment and ask for regular VS check
 2 IV lines: KVO / take samples and serial ECG
- ECG: ST elevation in V 2/3/4, aVL  ant-lat MI
- How did you come here today?
- Did you take any aspirin? aspirin 325 mg / plavix 300 mg
- Have you recently used Viagra? If no  nitroglycerine puffs 0.3 mg X
3; 5 minutes apart
 D1: brief neuro exam  pt is grossly neurological free
 Any results back?
 Order chest x-ray?
Brief history  Cardiac symptoms: heart racing / SOB / nausea / vomiting / sweating /
dizziness / LOC
 Chest symptoms
 Constitutional symptoms
 RISK factors
 PMH: medications / blood pressure / blood glucose / cholesterol / SAD
(especially for cocaine) / stress
 FH: heart diseases / heart attacks
ST elevation  Morphine 2 mg (up to 5 mg in antero-lateral MI)
 -blockers: metoprolol 2 mg IV (if NO contraindications: history of
asthma, BP, HR, heart block, inferior MI, cocaine overdose)
Mr , based on your symptoms and your ECG, it seems that you
have a heart attack. (Am I dying Dr?). This is a serious condition,
however you are stable and doing well, you are in a safe place and
we have experience in dealing with that. Heart attack means one or
more arteries providing blood supply to your heart are blocked by a
clot, we need to reopen this clot.
 Stat cardiology consult
 Do we have catheterization lab?
What we need to do now is to use medication called thrombolytic
or clot buster. I would like first to make sure that you are a good
candidate; recently have you had any surgery or bleeding? Have you
ever had stroke before? Were you diagnosed or told that you have
brain mass? Lesion? Based on the ECG, I do not see signs consistent
with cardiac aneurysm, and based on the BP, it does not look like
having dissection. Based on this, I can say that you are a good
candidate for thrombolytics, it is a life saving procedure, however
like any other medical intervention, it has side effects; it might cause
bleeding in 1% of cases, this includes stroke, so we need your
consent if you would like to have it!
 Heparin (hospital protocol or give enoxaparin 1 mg/ kg)
 Can you get the thrombolytics kit please
Have time Physical exam: listen to lung bases / S1 and S2 / compare BP both arms

OSCE-guide-III.doc Page 89 of 255


Emergency Medicine

Case 3: Chest pain v fibrillation / v tachy

- I would like to initiate code blue ACLS


- Is this is the patient ECG? Is it the last one? We need to start defibrillation!
o Done.
o Can you give me the last ECG please?  NORMAL ECG
- How do you feel? Do you feel drowsy? Because you have just had cardiac arrest, we
would like to make sure you are stable.

- Manage as the first case  the chest pain with normal ECG

Case 4: Chest pain v fibrillation intoxicated patient

- I would like to initiate code blue ACLS


- Is this is the patient ECG? Is it the last one? We need to start defibrillation!
o Done.
o Can you give me the last ECG please?  NORMAL ECG
- How do you feel? Do you feel drowsy? Because you have just had cardiac arrest, we
would like to make sure you are stable.

- The patient will be rude / angry / aggressive. ? intoxicated


o If the nurse gave you cocaine and said it was found with the patient; ask her to
keep it with the hospital security
o Mr you have just had a cardiac arrest, this is a very serious condition, and
might happen again, you should not leave, it is important to stay; we want to
make sure you are stable.
o If agitated  1 mg lorazepam injection

Cocaine (arrhythmias / HTN / neurological manifestations):


- What were you doing in the party? Any alcohol? Drugs?
- We will give you a medication that interacts with cocaine, it is very important to tell us if
you took any cocaine recently! (if cocaine  Ca channel blockers)
- Any weakness / numbness / tingling / vision problems / hearing problems?
- Do you hear voices / see things? Do you think someone want to hurt you?

- Manage as the first case  the chest pain with normal ECG

OSCE-guide-III.doc Page 90 of 255


Emergency Medicine

Heart Block

2 cases:
- One of them is DNR (must be dated, valid, and signed)
- The other case is: do not intubate / do not defibrillate. You can still pace maker

1- Introduction:
- Is this is the last ECG for this patient? I do not see any signs of V. fib or V. tachy. I
would like to see the patient first to make sure he is stable, and then I will look at the
ECG.
- Mr I am Dr working in the ER, do you hear me?
- I would like to activate the ACLS code please / start primary survey
A  Check the mouth, listen for patent air way  Give 4L O2 via
B  What is the O2 please nasal canula
 Trachea central, chest is moving  Monitor O2 for
 Listen to lungs, heart need to intubation
C  I would like to get the vital signs please;
 2 large IV lines; for IV fluids and to withdraw samples

2- Glasgow Coma Scale  [if < 8  intubate (ask about DNR)]


- This patient Glasgow coma scale is
- What is this patient code status? Any advanced directives?
o Is it signed, dated and valid?
o What does he have?
- We will respect his wishes, we will not (if DNR  do nothing)
- If no DNR:
D2 Third degree heart block:
 1 mg atropine  any changes?
 Pace maker:
- Rate: 20 more than his base heart rate
- Leads on sternum and apex
D1 Brief neurological scan, pupils,
Collateral  Do we have his file?
history  Can we contact his family physician? Or family member?
 Does he have a med alert?
 Do we have paramedics report?
 Can we check his belongings? He is taking , thiazides and digoxin
+
 I need to get his K level / digoxin level and I would like to check the dose for
digi-bind and digi-fib
Physical  Neuro exam
exam  Cardiac exam

Notes:
- For any unconscious patient: ask about advanced directives or DNR! What is this patient
code status?
- Whenever the examiner or the nurse tries to give you an ECG at the room entrance,
assess for V. fib or V. tachy and report: there are no signs of V. fib or V. tachy. I would
like to see the patient first to make sure he is stable.

OSCE-guide-III.doc Page 91 of 255


Emergency Medicine

Headache

Introduction
CC Headache for 2 hours (very acute very serious)
Analysis of CC Os Cf D
 Is this your first time
 Did you get any trauma?
 Would you describe it as the worst headache in your life? Thunder
clap?
- Can you please lie down?  Put the bed 45, I would like to make
sure u r stable!
- ABCD: IV lines / D1: Pupils
 PQRST
 ... position or coughing
Associated  Acute neuro: fever / neck pain / stiffness / vision / hearing / gait / falls
symptoms / weakness / numbness
 The patient says: I am diabetic  stat blood glucose (prick)
PMH HTN / blood thinners / kidney diseases
FH Kidney cysts / disease / aneurysm
SH Cocaine

If the patient loses his conscious  manage as unconscious patient


ABCD  If BP and HR  I am suspecting Cushing triad, put the bed in 45
 D1: Pupils, D3: thiamine 100 mg
Secondary  Glasgow coma scale As in the case of the
survey unconscious patient

Physical exam  Orientation (if conscious) ? subarachnoid


 Quick cranial nerves hemorrhage
 Quick neuro (tone/ motor / sensory / reflexes)
 Nuchal rigidity, Kernig's, Brudzinski's signs ? meningitis
(FEVER)
Management  CT scan stat LP
 Others: Chest x-ray / Urinalysis
 Blood works (CBC / differential / lytes) Treatment of
 Septic workup (samples / C&S) meningitis:

 Stat neurosurgery consult vancomycin 1 g IV


 Empiric antibiotics q12h + ceftriaxone
 IV mannitol 20% 1 gm / kg 2 g IV q12h
 Intubate:
- To keep air ways patent ampicillin 2 g IV
- To intra-cranial pressure q4h (if >50 years or
hx of alcohol use or
immunocompromise
)

OSCE-guide-III.doc Page 92 of 255


Emergency Medicine

Acute Abdominal Pain

OsCfD /+/ PQRST  will give you an idea


In abdominal pain cases:
 If you suspect certain diagnosis  scan for the risk factors for that disease
 If no diagnosis  continue GIT symptoms and general differential diagnosis

You suspect obstruction  nausea / vomiting (COCA+B / coffee ground material) AND bowel
movements  if vomiting  screen for dehydration
 If you dx obstruction  check risk factors of obst  then rest of GI symptoms
 If not obst  scan GIT  near-by systems  PMH for systemic disease
If you suspect kidney stones  screen with renal symptoms
 If you dx renal stone  check risk factors (diet, medications, hx of renal stones, uric
acid, bone pains / fractures)  then rest of urinary symptoms

Intestinal obstruction

Intro But first I would like to ask you, how do you feel now?
Analysis of Analysis: OsCfD: gradual, started colicky, and now continuous dull pain /

CC PQRST / What or (position / eating / bowel movements / vomiting)
 Screen for obstruction:
Nausea/ vomiting
o Relation to pain, which started first, does it relief pain
o COCA + Blood (coffee ground material)
Impact  Screen dehydration (dizziness / light headedness / thirsty / LOC)
 Bowel movements
How about any blood? Any time?
Still passing gas?
Red flags Risk factors for intestinal obstruction:
 Previous surgery? What? When?
 Fever/ night sweats/ chills / appetite / loss of weight / lumps & bumps
 PMH or FH of cancer or benign tumour
 Hx of Crohns disease (hx of abd pain/ bloody diarrhea) / family hx
 Hx of hernia / groin mass
 Gall bladder stones / right upper quadrant pain
Differential  Gastroenteritis:
diagnosis What did you eat yesterday? Place that you are not used to?
Diarrhea / blood in stools?
Anybody else ate with you and suffered from the same problem
 Renal: flank pain / burning sensation / going more to washroom / stone
 Liver: yellowish discoloration / itching / dark urine/ pale stools
 Hx of HTN / SOB / cough / phlegm (aortic dissection)
PMH / FH / SH

 X-ray findings of small intestinal obstruction: (1) Multiple air/fluid levels, (2) Dilated loops
of small intestine, (3) No air under the diaphragm.
 Management: (1) NPO / NG tube, (2) Oxygen mask, (3) IV fluids, (4) Stat surgical consult,
(5) Foleys catheter, (6) Correct electrolytes.

OSCE-guide-III.doc Page 93 of 255


Emergency Medicine

Right / Left lower quadrant abdominal pain middle aged female


 History: nausea / vomiting / fever / LMP / vag discharge / risky behaviour
 Physical exam: MUST mention pelvic and bimanual exam
 DD: (Left side): ectopic / PID / ovarian cyst / torsion / kidney stone
 DD: (Right side): ectopic / PID / ovarian cyst / torsion / kidney stone / appendicitis / intest
obstruction
 Investigations: -HCG / CBC / differential / abd-pelvic US / culture of the vag and cervical
secretions
 Long term complications: abscess / ectopic / infertility / intestinal obstruction / peritonitis
 Signs at PV exam: left adnexal mass / cervical motion tenderness

Acute abdomen in a female  missed period (ectopic), bleeding (abortion), discharge (PID)

PID
 Diagnostic plan:
Pregnancy test -HCG
CBC / ESR
Cervical culture (for Gonorrhea and Chlamydia)
Syphilis serology
 What is the treatment of pelvic inflammatory disease?
Cefoxitin 2 g IV every 6 hours X 2 days (covers anaerobic bacteria)
Doxycycline 100 mg orally BID X 2 weeks
Remove any IUD (if present)
 What are the indications of hospitalizing the patient?
(1) Pregnancy, (2) Pelvic abscess on U/S scanning / high fever (> 38.5 C), (3) PID at young
age, (4) Recurrent PIDs, (5) Failure to respond to outpatient management, (6)
Immunodeficiency (patients with HIV infection) or severe illness
 Complications of PID: abscess / ectopic / infertility / intestinal obstruction / peritonitis

Left lower quadrant abdominal pain elderly  Diverticulitis


 Signs:
Fever
Peritoneal signs: +ve cough tenderness / percussion tenderness / tenderness / rebound
tenderness / DRE: severe pain
 DD: diverticulitis / abscess / cancer
 Investigations:
AXR, upright CXR:
o May be normal
o Localized diverticulitis (ileus, thickened wall, SBO, partial colonic obstruction)
o Free air may be seen in 30% with perforation and generalized peritonitis
CT scan (optimal method of investigation). 97% sensitive, very useful for assessment of
severity and prognosis. Very helpful in localizing an abscess
Hypaque (water soluble) enema SAFE (under low pressure):
o Saw-tooth pattern (colonic spasm)
o May show site of perforation, abscess cavities or sinus tracts, fistulas
Barium enema: contraindicated during an acute attack: risk of chemical peritonitis
(perforation)
Sigmoidoscopy/colonoscopy:
o Not during an acute attack, only done on an elective basis
o Take biopsies to rule out other diagnoses (polyps, malignancy)

OSCE-guide-III.doc Page 94 of 255


Emergency Medicine

 Management: (1) NPO / NG tube, (2) Oxygen mask, (3) IV fluids, (4) Stat surgical consult
(5) IV antibiotics (IV ciprofloxacin 500 mg BID / IV Metronidazole 500 mg TID)
 Indications for surgery for diverticulitis:
Unstable patient with peritonitis
Hinchey stage 2-4 (large abscess / fistula / ruptured abscess / peritonitis)
After 1 attack if: (a) immuno-suppressed, (b) abscess needing percutaneous drainage
Consider after 2 or more attacks, recent trend is toward conservative management of
recurrent mild/moderate attacks

Diabetic ketoacidosis  acute abdominal pain

Analysis  Pain: OCD / PQRST /


 Vomiting: number of episodes + COCA
 AS: fever / malaise / cough / urinary symptoms / diarrhea /
gastroenteritis
 Diabetic history
Impact  Dehydration: feel weak / pee less / dry tongue and skin
Red flags  Hyperglycemia symptoms: thirst, polyuria, polydipsia, and nocturia
DKA  Acidosis: shallow rapid breathing or air hunger (Kussmaul or sighing
respiration), abdominal tenderness, and disturbance of consciousness
 Complications; of diabetes mellitus
DD causes of  Non-compliance OR wrong dose
DKA  Recent surgery
 Pregnancy
 Trauma
 MI
 Infection

Management of  IV fluids NS (1 L/hr x 2 hrs then 500 ml/hr x 2 hrs then 250 ml/hr
DKA x 4 hrs)
 Foleys catheter
 Insulin drip 2 units / hour check glucose and lytes every 2 hours
 When glucose reaches down to 15  fluids will continue as
maintenance, 2/3 : 1/3 of D5W : NS + 20 mEq KCl/L. 4:2:1 rule: 4
ml/kg/hr for the first 10 kg, then 2 ml/kg/hr for the next 10, then 1
ml/kg/hr for the next whatever
 Serial blood glucose
 ABG / serum ketones
 CBC / lytes
 Septic workup (chest x-ray / blood cultures / urinalysis)
 ECG (for the in K+)

OSCE-guide-III.doc Page 95 of 255


Emergency Medicine

Acute Abdomen

Patient came to ER with abdominal pain / vomiting / diarrhea / BP 90/60 / HR 120


Acute pancreatitis / dissecting aortic aneurysm (no vomiting) / perforated peptic ulcer

Introduction  I can see that you have a lot of pain, bear with me for few minutes and I will
give you a pain killer as soon as I can.
 In the moment, I would like to make sure you are stable
 What are the vitals pleas?
Stable Unstable
Proceed to I am going to start my primary survey  ABCD
history  When you send blood works: add lipase / amylase
Did you vomit blood? How about coffee ground? (if yes: order
blood)
Analysis Os Cf D / PQRST / / relation to position / breathing / eating
 Vomiting  COCA + Blood
 Change in the bowel movements
Impact  Dehydration
 How do you feel right now? What are the vitals please?
Red flags  Constitutional symptoms
DD Liver / GB  Yellowish discoloration / itching / dark urine / pale stools?
 Recent flu-like illness?
 Do you have hx of gall bladder stones? Repeated attacks abd
pain?
Stomach  Hx of PUD? GERD? Acidic taste / heart burn?
 Alcohol? How much? When was the last time? Did u drink >
usual?
 Gastroenteritis (What did you eat yesterday? Place that you
are not used to? Diarrhea / blood in stools? Anybody else ate
with you and suffered from the same problem?)
Medications  If vomited blood: Do you take steroids / NSAIDs / blood
thinners?
Kidney  Flank pain? Burning sensation? Dark urine? Frequency?
Aorta  Hx of HTN / atherosclerosis / DM / cholesterol / smoking /
SOB
Trauma  Did you have trauma?
PMH  Medications / allergies / long term disease?

Physical exam: abdominal exam


Face Eyes for jaundice / mouth for dehydration / ulcers
7 8
Inspection  No Cullens sign / no Grey Turner sign
 Cough tenderness

7
Cullens sign: peri-umbilical ecchymosis. It arises from spread of retroperitoneal blood associated with: pancreatitis / ruptured
ectopic preg / ruptured aortic aneurysm / ruptured spleen / perforated duodenal ulcer
8
Grey-Turner sign: ecchymoses of the skin of the flanks, also with retroperitoneal bleeding

OSCE-guide-III.doc Page 96 of 255


Emergency Medicine

Auscultation  Patient in severe pain:


Percussion I can see that you are in pain, can you bear with me for few minutes
Palpation then I will give you pain killer as soon as I can.
In order for me to examine you properly, I need to get good look at your
abdomen, for that reason, can you please lie on your back!
Do you feel relieved like that, I understand. It will be only few minutes,
do you want to give it a try? Slowly! Do you want me to help you! It is
crucial to reach a proper diagnosis!
If still refusing  offer 2 mg morphine S.C. Finally she will lie down.
 ASK FOR X-RAY (3 view x-ray abdomen)  ? perforation
Perforation  severe guarding, will not be able to proceed
Obstruction

Patient is obviously in severe pain, I will not be able proceed with examination

Management I am suspecting acute peritonitis I am suspecting pancreatitis


(? perforated peptic ulcer)
 NPO / nasogastric tube  NPO / nasogastric tube
 O2 mask  O2 mask
 IV fluids (250 ml/hr) / Cross  ICU admission
match blood

 Pain killer: Meperidine 1 mg / kg  Pain killer: Meperidine 1 mg / kg


or Fentanyl or Fentanyl
 Stat SURGICAL consult  Stat GIT consult

 3 view abdominal x-ray  3 view abdominal x-ray


 Abd U/S  Abd U/S for gall stones
 CT abdomen  CT abdomen

 Lab: CBC / blood sugar / calcium  Lab: CBC / blood sugar / calcium /
/ amylase / lipase amylase / lipase
 Albumin level / serum Ca

DD:
 Perforated PUD: vomiting coffee ground material
 Aortic dissection: NO vomiting / severe pain shooting to the back
 Acute pancreatitis:
NO upper GIT bleeding
Fever (due to chemical irritation not infection)
Pain improves when leaning forward
Paralytic ileus
Tetany

Ethical question:
 The patient girl friend is on the phone, she is asking about his condition?!
I am still doing my examination,
I can assure you that he is well taken care of, and we will do our best to help him,
All the details of his medical information is absolutely confidential, and I can not release

OSCE-guide-III.doc Page 97 of 255


Emergency Medicine

Upper GIT bleeding

Patient vomited blood; esophageal: varices / gastric: perforated PUD

Introduction  Because this is a case of GI bleeding, protection


 Mr , make sure that you are stable, then I will ask you
 How do you feel right now?
ABCD Be aggressive with fluids 2 L IV
Analysis  Os Cf D / COCA + dehydration
 Associated with pain (painless: esophagus / pain: stomach)  PQRST
 Nausea / vomiting / diarrhea
 Recently; any blood in stools / dark stools?
Impact  Weak / drowsy /
Red flags  Constitutional symptoms cancer stomach
DD Liver  Yellow / itchy / urine / stool
 Alcohol; how much / for how long / more than usual? SAD?
 Long term liver disease? Abdomen size? Bruises?
Memory? Concentration? Numbness LL?
 Risk factors for hepatitis: piercings, bld transfusion,
unprotected sex
Stomach  History of PUD / pain / GERD, heart burn
 Risk factors for ca stomach: smoking
Medications  Do you take any blood thinners? Aspirin? Steroids? NSAIDs?
PMH Any medications / allergies
FH Family history of gastric cancer
SH

Vitals or fluids (if stable: fluids 250 ml/hr) and monitor vitals
General General appearance of the patient: cachectic / distressed /

I would like to check if there is any postural drop in the blood pressure

Exam Liver exam: extra-hepatic signs of liver cell failure /+/ Bruises

Abdominal exam: epigastric mass / pain / liver / ascites

If painful: manage as acute abdomen case (perforated PUD)

Management STAT GIT consult for UPPER GIT endoscopy

IV pantoprazole (80 mg bolus then 8 mg/hr)

IV octereotide (25 mcg/hr)  portal circ VD  portal pressure

Abdominal x-ray

Admission to ICU

Longer term management:
 If portal HTN: non-selective -blockers
 Advice on cutting down the alcohol
 Advice to follow-up with the family doctor

OSCE-guide-III.doc Page 98 of 255


Emergency Medicine

Lower GIT bleeding


Introduction  Because this is a case of GI bleeding, protection
 Mr , make sure that you are stable, then I will ask you
 How do you feel right now?
ABCD Be aggressive with fluids 2 L IV
 D1: you might find some deficits  previous cerebro-vascular accident (may
be history of weakness and numbness)
Analysis  Os Cf D / COCA + dehydration
 Associated with abdominal pain  PQRST / / related to meals
 Vomiting blood? Nausea / vomiting?
Impact  Weak / drowsy /
Red flags  Constitutional symptoms cancer colon
DD Liver  Liver: yellow / itchy / urine / stool
Colon  Hx of dx / screening / for Ca colon? When? What was the
result?
 Recent change in bowel movement?
 Recent changes in calibre of stools?
 Risk factors for Ca colon: Fm hx of ca colon / diet ( fibres
fat)?
 Risk factors ischemic colitis? Atherosclerosis (similar to
CAD)?
Medications  Do you take any blood thinners? Aspirin? Steroids? NSAIDs?
PMH Any medications / allergies
FH Family history of colon cancer
SH SAD
Vitals or fluids (if stable: fluids 250 ml/hr)
General  General appearance of the patient: cachectic / distressed /
 I would like to check if there is any postural drop in the blood pressure
Exam  Abdominal exam:
If soft abdomen, and no findings (PE disproportionate to the pain) 
ischemic colitis
Management  STAT SURGERY consult
 NPO / IV fluids / Foleys catheter
 Admission to ICU
 Abdominal x-ray
 CT angiography

DD With pain Painless


Cancer colon: chronic intermittent Diverticulosis
Ischemic colitis: acute / sudden Angiodysplasia
Upper GIT bleeding
Aspirin
Hemorrhoids

Colitis: Radiation / Infectious / Ischemic / IBD (UC > CD)


Abdominal x-ray findings:
Intestinal ischemia: thumb print sign
Ischemic colitis: pneumatosis intestinalis (coli)
N.B. ischemic bowel  metabolic acidosis

OSCE-guide-III.doc Page 99 of 255


Emergency Medicine

ECG
 Normal
 V fib /+/ V tachy /+/ Torsades du pointes
 A fib /+/ Atrial flutter
 ST elevation:
o Pericarditis: all leads
o MI:
 V 2/3/4 V5/6, aVL: antero-lateral MI (left coronary)
 II, III, aVF: inferior MI (right coronary, posterior and inferior surfaces)
 Hear block third degree /+/ Bundle branch block
 Hyperkalemia /+/ Hypokalemia /+/ Hypercalcemia
 Digitalis toxicity

OSCE-guide-III.doc Page 100 of 255


Emergency Medicine

1. Rate:
 Regular: 300/number of big squares (R-R)
 Irregular: Number of Rs x 6

2. Rhythm  check for P wave in lead II


Tachyarrhythmias:
 Sinus tachycardia
 Irregular irregularities: A. Fibrillation
 Saw teeth (regular irregularities): A. Flutter
 Rapid SVT
 Ventricular arrhythmias: Premature ventricular beats / V. Tachy / V. Fib / Torsades de
pointes
Bradyarrhythmias: (< 60/min)
 Sinus bradycardia
 Heart block:
- 1st degree: P-R intervals increasing, but every P  QRS.
- 2nd degree:
 Mobitz type I: P-R intervals increasing, with missing QRS
 Mobitz type II: P-R intervals constant, with missing QRS
- 3rd degree: P-P has a rate, and the R-R has another rate

3. Axis
Normally, QRS in leads I, II, III are positive (upwards ).
 Right axis deviation: QRS in I is negative (downwards ); I and III facing.
 Left axis deviation: QRS in II, III is negative (downwards ); I and III opponents.
Diagram showing how the polarity of the QRS complex in leads I, II, and III can be used to
estimate the heart's electrical axis in the frontal plane:

Lead I negative and aVF positive: Rt axis deviation / Lead I positive and aVF negative: Lt axis
deviation.

4. Bundle Branch Block:


 Normally QRS in V1 is downwards , if in V1: QRS is upwards & wide: RBBB.
 Wide QRS in V6 (M mountain): LBBB.
A mnemonic to remember ECG changes is WiLLiaM MaRRoW, i.e. with LBBB there is W in
V1 and M in V6 and with a RBBB there is M in V1 and W in V6

OSCE-guide-III.doc Page 101 of 255


Emergency Medicine

5. ST segment:
Angina STEMI AND Non-STEMI

6. Others:
Hypokalemia Hyperkalemia
ST segment depression, inverted T waves, 1- Flat P wave
large U waves, and a slightly prolonged PR 2- Wide QRS
interval. 3- Spiked T wave

OSCE-guide-III.doc Page 102 of 255


Emergency Medicine

Phone calls

THE SEIZING CHILD PHONE CALL

The mother is on the phone, panicked as her child is seizing for 3 minutes

Introduction - Good evening, I am Dr the physician in charge at the (clinic/ER), may


+ Reassurance I know who is on the line?
- Before I proceed, I would like to know the name of your child, your
phone number and your address please! In case the line is disconnected,
hang on, and do not call me, I will call you back.
- What is your child weight?
- I know this is stressful time for you; however, I need your help here. I
will ask you some questions and give you some instructions.
Stabilize - Is this his first time?
- Did you call the ambulance? I will ask the nurse to call for you
- Is your child alert/conscious? Is he breathing? What is his color?
- Can you support and monitor him, remove any object near him?
- Can you put him on his side?  analyze during
- Do not put anything in his mouth, do not fix him
- Monitor till he stops seizing  assess for focal symptoms  after
Analyze the event - Can you describe the seizure for me please!
- Is it the first time? Or did it happen before?
During - Did you watch that?
- Did he lose conscious? Which happened first, seizing or LOC?
- Did he fall from height? Did he hit his head?
- Was he shaking? Certain part of the body or whole?
- Is he breathing? Did he turn blue?
- Any tongue biting? Did he wet himself? Roll up his eyes?
After - How long did it last? How did it stop? Spontaneously?
- Did he regain consciousness? After he regains consciousness; is he
able to recognize you? Able to talk? To move? Confused?
- Does he feel any weakness, numbness?
Fever - Does he have fever? When did it start? Any medical advice? Any
diagnosis? Treatment? Is he taking the medications? Any reason?
- Constitutional symptoms
Rule out Meningitis / pneumonia
BINDE
PMH Neurological deficits / diabetes mellitus
FH Febrile seizure / epilepsy
Counselling The seizing child counselling

Notes:
 Febrile seizure vs. meningitis: 1st time send the ambulance, 2nd time: send the ambulance if:
the seizure is > 15 minutes or > 2 attacks in 24 hours

OSCE-guide-III.doc Page 103 of 255


Emergency Medicine

THE CHILD SWALLOWED MEDICATIONS / CAUSTIC MATERIAL

The mother is on phone, panicked as her child swallowed medication / caustic material at home

Introduction - Good evening, I am Dr the physician in charge at the (clinic/ER), may


+ Reassurance I know who is on the line?
- Before I proceed, I would like to know the name of your child, your
phone number and your address please! In case the line is disconnected,
hang on, and do not call me, I will call you back.
- What is your child weight?
- I would like to ask you to take a deep breathe to calm down; I will need
your help here. I will ask you some questions and give you some
instructions, this is important for the sake of your child, ok.
Primary survey - Is your child alert?
+ Stabilize Yes NO
- Good to hear that - Do you know how to do CPR?
- Yes, start it now
- No, guide her:
- If < 8 yrs 5 / 1
- If > 8 yrs 15 / 2
ABC - Is he breathing? Talking to you?
- Can you check the child lips color?
- Does he recognize you?
- Can you check his mouth, and if there is any of the medications can
you take it out?
- If there are any remnants of the chemical on his mouth or face, can
you wash it with some water?
- Did you call the ambulance? I will ask the nurse to call the ambulance
and we will send it to your home now
- You need not to give him/her any thing by mouth
- You do not need to do anything; you can just hold him/her?
Analyze the event - Can you tell me what happened?
- Which medication did he take? If does not know; who is taking this
medication? Why for? Was it recently renewed? Is there vitamins?
Sleeping pills? Aspirin? When ambulance arrives, please give them any
bottle that the child might have taken medications from?
- How much did he take?
- For how long was he/she left alone? Any reason?
Is it the first time? - If yes, analyze the event
Or did this happen - Are there other children at home? Any history of repeated illnesses at
before? home? Repeated visits to ER?
BINDE - B: Was he a full term? Did he need special attention? Any congenital
anomalies?
- I: Are his/her shots up-to-date?
- N: Is he a fussy baby?
- D: Is he a difficult baby?
PMH

OSCE-guide-III.doc Page 104 of 255


Emergency Medicine

EMERGENCY TRAUMA PHONE CALL

A nurse is calling you from a remote rural medical center; she has a patient of trauma after a car
accident, BP 90/60 and HR 120. Manage over the pho ne.

A/B: Mouth / O2 / inspect chest, neck / listen to lungs, heart


- I am Dr , the physician in charge in the emergency room; may I know who is on the
line?
- Can you describe the case / situation for me!
- Now, I would like to get your phone number (and address if reasonably far), and patient
name, and if by any chance the line gets disconnected, do not try to call me, just hang the
line and I will call you back, ok.
- Are you a nurse? Do you have someone else with you to help?
- Is the patient alert?
- Does he have a neck collar?
- Open his mouth, check it for any FB or blood, and make sure it is clear. Can you feel air
coming from patient mouth?
- What is his O2 saturation? Can you give him O2 please, 4 L/min via a nasal canula
- Can you open the neck collar window and check his trachea and neck jugular veins, is the
trachea central or not? Are the JV engorged?
- Can you look to his chest; is it bilateral symmetrical movements of the chest wall?
- Can you listen to air entry on both sides?
- I am suspecting tension pneumothorax, I would like you to insert a wide bore needle (16
or 14 G) into the second intercostal space, in the MCL (on the upper margin of the third
rib)
o If she said, I did not do it before, or I am afraid I can not do it, or who is
responsible for this: encourage her and tell her that you are responsible
- Did you get a gush of air?
o Yes, that is great, you have just saved the patient life
- Can you re-check the trachea centrality, jugular veins and HR again for me please
o If stable: can you please fix a chest tube in the 5th intercostal space, anterior
Axillary line
C: Vital signs / fluids / withdraw blood samples / D:
- Can you please insert two wide bore (16 G) IV lines in the patient ante-cubital fossae,
and give fluids (2 L NS bolus) and then withdraw samples for
- Can you find any source of bleeding?
- D3: Give universal antidotes:
o Give 100 mg thiamine
o If RR  naloxone
o If blood sugar dextrose
- Can you tell me his Glasgow coma scale! Check his papillary reaction (D1).
- I would like also to intubate the patient

What are the requirements to transfer patient from a center to another center?
- Accompanied by two trained medical personnel (paramedics, nurses, physicians)
- Intubated and on ventilator
- Secured IV lines and fluids
- pre-arrangement with the place that will receive the patient

OSCE-guide-III.doc Page 105 of 255


Physical Examination

Physical Examination

OSCE-guide-III.doc Page 106 of 255


Physical Examination

Medical Physical Exam

Always be nice and smile


Use the alcohol anti-septic
Introduction
Drape / cover the patient properly
Warm your hands before touching the patient
Warm your stethoscope
If you are going to make painful manoeuvres: explain before / apologize for the pain / do
not repeat
Make sure you put the patient in the proper position at ease
Whenever you notice the patient is in pain, tell him that you will give pain medication as
soon as you can
If you ask the patient to walk, make sure he will not fall, surround him
Make sure you talk to the patient: I am going to look, feel, press, tap, listen (use
simplified non-medical language)
Report to the examiner every thing in medical terms, by inspection , by palpation,
percussion,
Cover the patient after you finish
Thank the patient and the examiner

Introduction:
- Good evening Mr , I am Dr I am the physician working in the clinic today / I am the
physician in charge in the ER now. I understand that you are here because you have been
having For the next few minutes I am going to do physical exam for your and I will
need to ask you questions during my exam. Also, I will be asking you to do some
movements and manoeuvres, if you feel any discomfort or pain, please do not hesitate to
let me know and stop me
- If you have any questions or concerns please feel free to ask me / to bring it up
- If SOB: during my exam, if you feel that you can not continue, please stop me

If there is history taking and then physical exam:


- I will be asking you some questions, then I will do physical exam. Hopefully towards the
end, we reach a working plan

Vital signs:
- If vitals are given: based on the vitals, the patient is stable, I would like to proceed. Or
the patient is unstable! Or comment: with mil fever
- If the vitals are missing one; e.g. the temperature: ask about it specifically
- Vitals are not gives:
o I would like to get the vitals before I start!
o I am going to start my exam by measuring your vital signs that is your blood
pressure, heart rate. And I will start by measuring your heart rate

OSCE-guide-III.doc Page 107 of 255


Physical Examination

Abdominal examination:
 Introduction / Vital signs / General inspection of the patient: pt is sitting comfortably
- Inspection
- Auscultation: bowel sounds / bruits (aortic / renal / iliac)
- Percussion
- Palpation: superficial / deep / special tests
Respiratory examination:
 Introduction / Vital signs / General inspection of the patient: pt is sitting comfortably
- Inspection: face / hand / neck / chest / back
- Palpation: tenderness / tactile fremitus / chest expansion
- Percussion: dullness / percussion note / diaphragmatic excursion
- Auscultation: regular / special tests
o Then end with cardiology exam
Cardiac examination:
 Introduction / Vital signs / General inspection of the patient: pt is sitting comfortably
- Inspection: face / hand / neck / chest / heart (PMI)
- Palpation: apex / left para-sternal areas for heaves / valvular areas for thrills
- Auscultation: in Z format A-P-T-M
o Leg exam for edema
o Lung bases
- If full CVS exam  peripheral vascular assessment: abdominal bruits / legs pulses
palpation / chest exam
Musculoskeletal examination:
 Introduction / Vital signs / General inspection of the patient: pt is sitting comfortably
- Inspection: SEADS (scars / erythema / atrophy / deformity / swelling) / specific findings
(bulk of muscles / bony symmetry)
- Palpation: (TTC) tenderness / temperature / crepitus / effusion
- ROM: active (if normal, NO need to do the passive) / passive / against resistance
- Special test: mechanical (shoulder / elbow / hip / knee / ankle)
o To complete my exam, I would like to do:
 Check the pulses of the limb (upper or lower)
 Brief neurological examination of the limb
 One joint above and one joint below examination
 The other side joint
Neurological exams:
 Introduction / Vital signs / General inspection of the patient: pt is sitting comfortably
- Orientation: what is your name sir? Where are you? Time? Place?
- Cranial nerves
- Upper and lower extremities:
o Inspection
o Palpation / bulk
o Tone
o Motor power
o Sensory
o Reflexes
- Gait / Romberg test
- Cerebellar signs / Coordination
- Cortical sensations: two points discrimination

OSCE-guide-III.doc Page 108 of 255


Physical Examination

Abdominal examination:
- Introduction
- Vital signs
- General inspection of the patient:
o By general inspection, pt is lying down comfortably, no obvious distress
o Can I take a look at your eyes, would you please look downwards? No jaundice.
Upwards please? No pallor
o Would you please open your mouth: no signs of dehydration or vomiting
o Can I take a look at your hand?
 Temperature is fine / and skin is moist
 Normal capillary refill (< 3 seconds)
 No obvious clubbing
- I am going to drape you now!
o Bed flat
o Can you please put you hands to your sides
o Allowing the patient to bend his/her knees so that the soles of their feet rest on
the table will also relax the abdomen!

- Inspection; I will take a look at your abdomen:


o By inspection: abdomen is flat, not distended, normal contour, umbilicus is
inverted, abd is moving with respiration, no scars from previous surgeries, no
bruises, no obvious bulge or mass
o Would you please look to the left side, can you please cough
 Check pt face for tenderness  there is cough tenderness
 If no  cough another time  no herniation along the middle line.
Thank you, you can look back if you wish

- Auscultation; I am going to listen to your abdomen, warm my stethoscope:


o McBurneys point: normal bowel sounds (hyperactive / decreased)
o 2 inches above the umbilicus  no aortic bruits
o 2 inches above / 2 inches bilateral from umbilicus  no renal bruits
o 2 inches below / 2 inches bilateral from umbilicus  no iliac bruits

- Percussion: now, I am going to tap on your abdomen, can you point to your painful are.
I am going to start away from there:
o Percuss in 2 X 2 lines, and percuss to side  for ascites
o No percussion dullness / normal tympanic percussion note / no percussion
tenderness / no ascites

- Palpation:
o I am going to feel your abdomen. Start away from the painful area:
 I am checking (name the 4 quadrants or the 9 areas of the abdomen);
(NO) tenderness, guarding or rigidity
o I am going to apply more pressure now: no obvious masses, no organomegaly
o I am going to feel your kidneys now (bimanual)  no enlargement, no
tenderness of the kidneys
o I am going to do some special tests:
 Murphys sign (Rt costal margin)  can you take a deep breath
 Rebound tenderness: I am going to press and release my hand, can you
tell me which causes more pain! (any point except McBurneys)

OSCE-guide-III.doc Page 109 of 255


Physical Examination

 Press at McBurneys point  is it painful


 Press left iliac area, tell me if there is pain:
Not painful
Pain to the left side: left iliac tenderness / negative Rovsing's
sign
Pain to the right side  positive Rovsing's sign
/+/+/+/+/+/+/+/ COVER THE PATIENT ABDOMEN /+/+/+/+/+/+/+/
 Psoas sign: Rt LL extended; I am going to press on your thighs down, can
you press up against my hand (actively flex thigh at the hip)
 Obturator sign: Rt hip flexed, knee flexed; can you relax yourself, I will
rotate your leg (int and ext rotation), if pain  +ve sign

- Can you please sit up now; I am going to:


o Tap on your back, check the costo-vertebral angle for renal tenderness
o Listen to the back of your lungs  no crepitation
- Can you please lie down; thank you.
I would like to complete my exam by doing pelvic and vaginal exam / and digital rectal
exam
o Inguinal LNs
o Pelvic, vaginal and bimanual exam; looking for:
 Bleeding and/or discharge
 Adnexal masses / tenderness
 Cervical motion tenderness
 To obtain samples for culture and sensitivity
o Digital rectal exam; looking for:
 Bleeding
 Piles / haemorrhoids
 Fissures

How to differentiate between spleen and kidney by palpation:


- Spleen changes position with respiration (goes down with inspiration)
- You can feel the lower border of the spleen
- Spleen does not ballot

If there is history of Crohns disease:


- Inspection:
o General: truncal obesity / moon-like face
Red eyes
Mouth: ulcers
o Hands: nail changes / clubbing / skin rash / psoriatic changes / joint tenderness
o Abdomen: stria / scars
o Lower limbs: erythema nodosum
o DRE: fissures / fistulae
- When he sits up  press on the sacro-iliac joints  sacroiliitis

OSCE-guide-III.doc Page 110 of 255


Physical Examination

Liver Examination
Patient vomited blood 20 minutes ago, perform focused examination for liver cell failure
- Because the patient is vomiting blood, I would like to ask for protection for me and my
team please (gloves / gowns / masks / goggles)
- Introduction / Vital signs
- Orientation: I am going to ask you some questions which are part of my physical exam.
Do you know where you are now? What is the time? Do you know why you are here? 
Patient is oriented to people, time and place
- General:
o Patient is lying comfortably no signs of obvious distress
o Eyes  sclera: no jaundice
o Mouth: no fetor hepaticus / no clots / no vomitus
o Face: no parotid gland enlargement
o Hands: no clubbing / nail changes / palmar erythema / Dupuytrens contracture / atrophy
of thenar and hypothenar muscles / look for IV marks / stretch your hands please
(flapping tremors)
o Can you please lower your gown: no gynecomastia / no spider nevi
o Check lower limbs edema
- Inspection:
o No bruises / normal hair distribution / no caput medusa / no dilatation of collateral veins /
no obvious ascites / no scratch marks
- Auscultation
o Listen to liver: no hepatic rub / hum / or bruits
o Listen to spleen: no rub / no hum
- Percussion:
o 2 X 2 lines tap
o Liver: MCL (from above downwards and from down upwards) liver span
o Spleen: ant axillary line  last intercostal space / ask pt to take deep breathe in / then re-
percuss for the spleen
o Ascites: from midline to the side, no dullness  so there is no need to perform the
shifting dullness (to be clinically palpable: ascites > 500 ml)
- Palpation:
o Liver: start from the right iliac fossa and go upwards, while the patient is breathing in and
out (push during inspiration, do not move your hand from the patient)  margin of liver
is not palpable, not tender, and not nodular.
o Spleen: patient elevates his LEFT side 45, support from left back. Start from above the
umbilicus towards the spleen  spleen is not palpable
- I would like to complete my exam by doing:
o Digital rectal exam: for hemorrhage / piles
o Check for testicular atrophy

 DD of hepatitis: (alcoholic / viral / drug-induced) / cirrhosis / liver cancer


 Worst prognostic signs: ascites / encephalopathy
 Investigations: AST:ALT > 2:1, GGT, albumin, INR, CBC
 Treatment of encephalopathy: treatment of precipitating causes / Lactulose (15-30 ml tid) /
Rifaximin (550 mg PO bid)

 DD of ascites: liver cirrhosis / renal failure / heart failure / TB / malignancy /


Hypoalbuminemia
 Treatment of ascites: Sodium restriction (20-30 mEq/d) / Diuretic therapy / Therapeutic
paracentesis may be performed in patients who require rapid symptomatic relief for refractory
or tense ascites

OSCE-guide-III.doc Page 111 of 255


Physical Examination

Nasal bleeding  Hematological Examination

- Because the patient is bleeding, I would like to ask for protection for me and my team
please (gloves / gowns / masks / goggles)

- Introduction
- Vital signs

- General:
o Patient is lying comfortably no signs of obvious distress
 Nose: open the speculum antero-posteriorly
 Use the otoscope for ENT (nose / ear / mouth)
o Look for bruises / petichae  if you find them  continue hematological exam

- I ma going to feel your glands:


o Occipital / post-auricular / pre-auricular / sub mandibular / sub mental / anterior /
posterior cervical / supra-clavicular
o I would like to check the axillary LNs (ant / post / med / lat / apical)
o I would like to check the inguinal LNs:
 Superficial (transverse, along the groin crease)
 Deep:
Transverse: along the groin crease
Longitudinal: medial to the femoral artery
(Lat  Med: NAVAL Nerve Artery Vein LNs)
o I would like also to check the popliteal LNs
- Check for bony pains: sternum and 3-5 spots on the vertebral column (I am going to press
on your back)

CAN YOU LIE DOWN PLEASE?


- Listen to heart / lungs
- Abdominal exam:
o Inspection
o Auscultation
o Percussion: liver / spleen
o Palpation: liver / spleen
o I would like to: perform vaginal and pelvic exam

- Finally, I would like to do:


o Fundoscopic retinal exam
o Orientation
o Brief neurological exam:
 Hands: pull/push/ do you feel the same
 Legs: pull/push/ do you feel the same
Patient is grossly neurologically free
N.B. focal neurological deficits to DD between TTP and HUS.

OSCE-guide-III.doc Page 112 of 255


Physical Examination

Questions:
- Diagnosis:
o ITP (Immune Thrombocytopenic Purpura)
 Most common cause of isolated thrombocytopenia
 Diagnosis of exclusion (i.e. isolated thrombocytopenia with no clinically
apparent cause)
- Investigations:
o CBC: thrombocytopenia
o Peripheral blood film: decreased platelets, giant platelets
Bleeding time: increased / PT and aPTT: normal
Anti-platelets antibodies
o Bone marrow: increased number of megakaryocytes (critical test to rule out other
causes of thrombocytopenia for age > 60 years; e.g. myelodysplasia)
o Markers of hemolysis: increased unconjugated bilirubin, increased LDH,
decreased haptoglobin
o Kidney function tests (urea / creatinine for HUS)
- Treatment:
o Steroids (methylprednisolone 1 g/d for 3 days, then prednisone 1.5 mg/kg/day)
o Immunoglobulins (if low platelet count): IVIG 1 g/kg/d X 2 days
o Splenectomy
o Vaccination (pneumococcus, meningococcus, HIB)
- DD:
o ITP (Immune Thrombocytopenic Purpura)
o TTP (Thrombotic Thrombocytopenic Purpura)
o HUS (Hemolytic Uremic Syndrome):
ITP TTP HUS
 Remitting / relapsing  Predominantly adults  Predominantly children
course  Thrombocytopenia  Severe thrombocytopenia
 Mild fever  Micro-angiopathic  Micro-angiopathic
 Splenic discomfort hemolytic anemia hemolytic anemia
(mild engorgement) (MAHA) (MAHA)
 Renal failure  Renal failure
 Neurological symptoms
(headache, confusion,
focal deficits, seizures)
 Fever
Investigations  CBC and blood film: decreased platelets and schistocytes
(both TIP, HUS)  PT, aPTT, fibrinogen: normal
 Markers of hemolysis: increased unconjugated bilirubin.
increased LDH, decreased haptoglobin
 Negative Coombs' test
 Creatinine, urea, to follow renal function
 Stool C+S (HUS)
Management  Plasmapheresis steroids
(both TIP, HUS)  Platelet transfusion is contraindicated (increased micro-
vascular thrombosis)
 Plasma infusion: if plasmapheresis is not immediately
available
 TTP mortality 90% if untreated

OSCE-guide-III.doc Page 113 of 255


Physical Examination

Chest Examination
- Introduction
- Vital signs; especially: tachypnea / temperature
- General inspection of the patient; comment on respiratory distress
- Inspection:
o General:
Face: symmetrical, no nasal flaring / laboured breathing
Eyes: jaundice / pallor
Mouth / can you flip your tongue please: no central cyanosis / dehydration / exudates
or secretions
Hands: no nicotine staining / clubbing / peripheral cyanosis / normal capillary refill
o Neck; can you lower your gown please
Trachea is central, no accessory ms used for breathing / LNs
o Chest:
Chest is symmetrical, no accessory ms used for breathing, no intercostal retraction
Look from the side: no increase in the A/P diameter of the chest / no barrel chest
- Palpation; I am going to feel your chest
o Tenderness:
Check the ant chest wall: no tenderness
o Tactile fremitus: can you say 99 for 4-5 times, whenever you feel my hands on
your chest  normal tactile fremitus
Any lung pathology  tactile fremitus ( conduction)
Any pleural pathology  tactile fremitus (insulation
o Chest expansion: normal chest expansion
/+/ I will continue to examine you from the back, then I will check the front again /+/ Can
you cross your arms please
- Inspection: chest is symmetrical, no scars, swellings, or deformity
- Palpation; I am going to feel your chest
o Tenderness: no tenderness
o Tactile fremitus: can you say 99 for 4-5 times, whenever you feel my hands on
your back
- Percussion; I am going to tap on your chest
o Apex (2) / back (6) / sides (2)  normal percussion note:
- Auscultation; now, I am going to listen to your back
o Whenever you feel the stethoscope touching your back, can you please take a
deep breathe in and out from your mouth
o Normal vesicular breathing sounds bilaterally, no rhonchi, no wheezes
- Now, I am going to demonstrate the special tests that should be done if there is
consolidation, with auscultation:
o Can you say letter E  egophony (normal: ee / over affected area: ay)
o Can you say 99  vocal fremitus
o Whisper 1/2/3: whispering pectorology ( in audibility)

/+/ I will now go to examine you from the front again /+/
- Percussion; I am going to tap on your chest on both sides
o Apex (2) / MCL (6) / sides (2)  normal percussion note:
- Auscultation; now, I am going to listen to your lungs
o Mid clavicular line both sides
o Normal vesicular breathing sounds bilaterally, no rhonchi, no wheezes
- Cardiology exam: Auscultation

OSCE-guide-III.doc Page 114 of 255


Physical Examination

Pneumonia

CURB 65 score Pneumonia Clinical Prediction Tool


Confusion
Urea: > 7 mmol, BUN > 19
Respiratory rate: > 30 breaths / min
Blood pressure: systolic < 90 and diastolic < 60
65: or older in age
Management Mortality
01 Can treat as outpatient <5%
23 Consider hospitalization <10%
45 Consider ICU <30%

Pneumonia severity index: another clinical index (scored on age, resp rate, co-morbidities ),
used to determine whether to admit the patient to the hospital or not.

IDSA / ATS Community Acquired Pneumonia Treatment Guidelines 2007:


(IDSA: Infectious Diseases Society of America, ATS: American Thoracic Society)
Setting Circumstances Treatment Medications
Outpatient Previously well Macrolide  Azithromycin 500 mg PO once,
OR THEN 250 mg qDay for 4 day
Doxycycline (total of 5 days)
 Clarithromycin 500 mg PO bid
for 7-14 days
Comorbidities, or Beta-lactam + Macrolide  Levofloxacin 750 mg PO q24h
use of antibiotics OR for 5 days
in last 3 months Respiratory fluoroquinolones OR
Inpatient Ward  Amoxicillin 1000 mg PO tid
+ Clarithromycin 500 mg PO
bid
ICU9 3rd gen cephalosporin + Ceftriaxone 1 g IV q24h
Macrolide + Azithromycin 500 mg IV q24h
OR for 5 days (step to oral when
Beta-lactam + Macrolide tolerated)

Other medications for the management of pneumonia:


Susceptible for pseudomonas / recent use (within 3 months) of antibiotics or cortisone:
piptazo (piperacillin / tazobactam); 3.375 gm IV q6h)
MRSA: Vancomycin 1 gm IV q24h

Pneumococcus vaccination:
Elderly patients > 65 yrs old
COPD
CHF
Cirrhotic
Cancer
Immunocompromised: steroids / HIV / DM / splenectomy
Leukemia / lymphoma
9
Especially in the presence of risk factors: group home / hospital infection / immunocompromised

OSCE-guide-III.doc Page 115 of 255


Physical Examination

Case: HIV positive man C/O: SOB for 1 week / cough / fatigue ? Pneumonia (? PCP)

DD of unilateral lobar reticular pattern on CXR: PCP (pneumocystis pneumonia) / Kaposi


sarcoma in the lung / Lymphoma / Atypical pneumonia (mycoplasma / chlamydia)
DD of bilateral hilar infiltrates: PCP / TB / CMV
Signs of HIV: check the entire skin for Kaposi sarcoma / mouth and pharynx for thrush or oral
hairy leukoplakia / LNs / check the abdomen for liver and spleen
Investigations:
o O2 saturation / ABG
o CBC / differential / CD4 count
o LDH ( in 95% of PCP cases)
o Blood culture
o Sputum for cytology / culture / gram stain / acid fast stain
o Bronchoscopy: bronchial washing
Treatment of PCP:
o Septra DS (trimethoprim / sulfamethoxazole: 160mg/800mg): 2 tablets q8h x 14 days then
continue prophylactic 1 tab od
o Severe illness (PO2 < 70 mmHg):
 Admit
 O2
 Septra
 Steroids: prednisone 40 mg bid x 5 d then 40 mg od x 5 d then 20 mg od x 5 d
o If patient is allergic to TMP/SMZ:
 For treatment: IV pentamidine or atavaquone
 For prophylaxis: Dapsone or atavaquone

HIV routine prophylaxis:


o Based on CD4 count:
 CD4 < 200  prophylaxis for PCP: TMP/SMZ or Dapsone
 CD4 < 100  prophylaxis for toxoplasma: TMP/SMZ or Dapsone
 CD4 < 50  prophylaxis for mycobacterium avium complex: azithromycin
(1/week) or clarithromycin (2/d)
o Regardless the CD4 count:
 PPD testing for TB, considered positive if > 5 mm induration  INH for 9 months
 Vaccinations for pneumococcus / influenza / hepatitis B

OSCE-guide-III.doc Page 116 of 255


Physical Examination

Cardiac Examination Essential HTN case


+ 65 years old gentleman, with high blood pressure for 20 years, for follow-up visit
+ 35 years old gentleman with newly diagnosed high blood pressure, please assess
+ If 10 minutes case: vitals + examination + If 5 minutes: only examination

Analysis For how long? Medications?


How frequent do you check your BP? Is it controlled?
Impact End organ damage? Proteinuria / visual changes / headaches /
confusion?
Red flags Other cardiovascular risk factors: smoking / DM / high
cholesterol / history of MI in the family

Introduction Can you please lie down


HR and BP (lying down) Get the systolic by pulse palpation, then increase 30 mmHg, then
auscultate
Sit up, relax for 2 minutes  Orientation
 Mouth for dehydration
 Eyes for pallor
 Lung bases auscultation
 Sacral edema
 Lower limb edema
HR and BP (sitting up) No difference
General  Asses the radial pulse bilaterally
 Hands: capillary refill / clubbing / temperature
 Fundoscopic examination: disc edema / retinal hge
Neck Carotid  Carotids: bruits / palpate
JVP  JVP: measure / Kussmaul sign / hepato-jugular reflux
Heart Inspection  Symmetrical / PMI
Palpation  Apex
 Heaves (Lt para-sternal / with knuckles)
 Thrills (A-P / with palmar aspect of distal phalanges of fingers)
Auscultation Do timing by simultaneously checking the carotid pulse.
 Areas A-P-T-M: normal S1, S2, no murmurs.
 Look for S3, S4 at the apex (with the bell)
 Aortic insufficiency murmur
 Lung bases
Abdominal exam Pulsation / Bruits (aortic / renal / iliac)
Lower limbs exam Inspection / Palpation (temperature / pulsations) / LL BP
- Introduction
- Vital signs; especially: BP / HR
o Ask the patient to lie down, I am going to assess your blood pressure twice, one
while lying down, the other after you sit up
 I will start by measuring HR for 10 seconds; your HR today is which is
normal
 Do the BP: get the systolic by pulse palpation, then increase 30 mmHg, then
auscultate  your BP today is
o Can you please sit up on the edge of the bed, (feet dangled)? I need you to relax
for 2 minutes, before I proceed to other measurements, during which I am going
to ask you some questions:

OSCE-guide-III.doc Page 117 of 255


Physical Examination

 Orientation: Do you know where you are? Why? Date?


 Mouth for dehydration
 Eyes for pallor
 Can I listen to the back of your lungs (lung bases)  no crackles
 Press against the sacrum for sacral edema
 Press on the lower limbs, for edema
o Measure the sitting HR and BP  no difference in readings  no orthostatic
hypotension (no changes: no 20 mmHg in systolic, no 10 mmHg in diastolic,
no 20 in HR)
- General exam:
o Comment on the general status / respiratory distress of the patient
o Assess the radial pulse bilaterally: pulse is normal, regular, equal on both sides,
with no delay
o Hands: normal capillary refill / no clubbing / warm moist skin
o I would like to do fundoscopic examination (I am going to shine a light in your
eyes to examine the back of your eyes): looking for disc edema / cotton wool
exudates / retinal hemorrhage / nipping of the veins
- I am going to put the bed in 45 position; examine your neck:
o Carotids:
 Auscultate for bruits  use the bell
 If no bruits  palpate the carotid arteries (ONE AT A TIME)
 Pulse is regular, normal volume and contour
o JVP:
 Measure the JVP
 Kussmaul test: take a deep breathe and hold it, measure the JVP:
Normally it will  negative Kussmaul test
If JVP  positive Kussmaul test (restrictive cardio-myopathy,
constrictive pericarditis)
 Press on the liver for 10 seconds  normally the JVP will for few seconds
then will  if it does not  hepato-jugular reflux
- Chest inspection:
o Chest is symmetrical, no obvious pulsations
o PMI (point of maximum intensity) is not obvious / or is obvious
- Palpation:
o Palpate the apex and locate it  apex is palpable at in the MCL, not
hyperdynamic (vol ovrld), not sustained (pres ovrld), not displaced
o Palpate the left para-sternal area by knuckles: no Rt ventricle heave
o Palpate the Aortic / Pulmonary areas by finger tips  no thrills
- Auscultation:
o In Z format A-P-T-M: normal S1, S2, no murmurs
o Flip the bell; check the apex for S3, S4
o While listening to the left para sternal area: can you please take a deep breathe in,
exhale it out and hold, and lean forward  no enhancement of aortic
insufficiency murmurs
o Listen to the lung bases  no basal lung crackles
- I am going to put the bed flat now:
o Abdominal exam: inspect for pulsations / auscultate for bruits
o Lower limb exam:
 Inspection: symmetry / SEADS / nails hypertrophy / skin changes
 Palpation: temperature / pulsations / radio-femoral delay
 I would like to measure the blood pressure in lower limb

OSCE-guide-III.doc Page 118 of 255


Physical Examination

Secondary Hypertension

A 25 years old young man with HTN;

Cardiac Renal Endocrine


 Coarctation of the aorta  Poly-cystic kidney  OCPs
 Cocaine (nasal septum)  Renal artery stenosis  Cushing disease / syndrome
 Pheochromocytoma
 Primary hyperaldosteronism
 Hyper / hypo thyroidism

Patient disrobed to underwear, draped below waist


Introduction Can you please lie down
Vitals  There is systolic and diastolic HTN
 I want to rule out orthostatic hypotension (pheochromocytoma)
 I would like to compare UL and LL (coarctation of aorta)
 No tachycardia (pheochromocytoma / hyperthyroidism)
 No bradycardia (hypothyroidism)
General  Orientation (brain edema)
 Truncal obesity / cervical fat pad / moon face (Cushing)
 Check for proximal ms weakness (Cushing)
 Check the eyes from the side for proptosis, lid lag test
(hyperthyroidism)
 Asses the radial pulse bilaterally
 Hands: capillary refill / temperature / skin dry or wet (thyroid).
Stretch your hands please  fine tremors (hyperthyroidism)
 Fundoscopic examination: disc edema / retinal hemorrhage
Neck Thyroid  No masses
 Palpate for the thyroid
 Percuss for the retro-sternal thyroid dullness
JVP  JVP
Chest  Palpate the apex
 Look for S3, S4 at the apex (with the bell)
 Listen to lung bases
Abdominal exam  Inspection: no obvious masses, no stria, no caf au lait spots
 Auscultation: bruits (aortic / renal)
 Palpation: feel for renal / supra-renal mass
Lower limbs exam  Edema
 Rule out femoral / radial delay
Brief neuro exam  Knee or ankle reflex

Notes:
Watch for labile white coat HTN
Any HTN in middle age  secondary HTN  most likely kidney disease

OSCE-guide-III.doc Page 119 of 255


Physical Examination

Hypertension
 Predisposing Factors
Family history Sedentary lifestyle Excessive salt intake /
Obesity Smoking fatty diet
Alcohol consumption Male gender African American
Stress Age >30 Dyslipidemia
 Diagnosis:
Visit ONE:
o If hypertension urgency or emergency (sBP > 210 or dBP > 120) or end organ damage
(e.g. confusion)  diagnose HTN
o Else (provided 2 more readings during same visit)
 Search for target organ damage: history (cardio-vascular risk factors) / examination
 Investigations:
CBC / Na+, K+ / fasting blood sugar / lipids (total cholesterol, HDL, LDL, TG)
Kidney function tests / Urinalysis / Renal Doppler
ECG / Echocardiogram
For secondary HTN: TSH / Plasma aldosterone / renin levels / 24 hours urine
for metanephrines / VMA
 Life style modifications ( salt / alcohol / cholesterol / exercise)
 Follow-up visit within 4 weeks
Visit TWO; within 4 weeks
o If (target organ damage OR diabetes mellitus OR chronic kidney disease OR blood
pressure > 180/110 mmHg)  diagnose HTN
o Else (BP: 140-179 / 90-109 mmHg)  24 hours BP monitor (diagnose if mean awake
sBP >135 mmHg and/or dBP > 85 mmHg or mean 24 hours sBP >130 mmHg and/or
dBP > 80 mmHg)
 Management:
Target BP is < 140/90 mmHg, < 130/80 if DM or chronic kidney disease
Life style modifications (initial management):
o Smoking cessation and decrease alcohol consumption
o Diet: salt / cholesterol and saturated fats / follow Canada's Guide to Healthy Eating
o Weight: maintain healthy BMI (18.5-24.9)
o Moderate intensity dynamic exercise: 40-60 minutes, 4-6 times/week
Pharmacological:
o First line: Diuretics; e.g. hydrochlorothiazide 12.5 25 mg PO od Except:
 DM: ACEIs; Ramipril 2.5 5 mg PO od
 Gout: Amlodipine (5 mg PO od) OR Candesartan (4 8 mg PO od)
 Elderly (especially if IHD):
ACEIs
-blockers: metoprolol 25 mg bid
Especially if CHF / EXCEPT: asthma / bradycardia
 Pregnant:
Hydralazine: 10 mg PO qid for few days then 25 mg PO qid
OR -methyl dopa: 250 mg PO bid
 If > 3 cardiovascular RF: statins / ASA
o If partial response to standard dose monotherapy, add another first-line drug
 Do NOT give -blockers and Ca ch blockers  may cause heart block
 Do NOT give ACE and ARBs  both K+,
 Available combinations: Altace plus (ramipril + diuretic) / Diovan H
o Notes on ACEIs:
 Contraindications of ACEIs: Angio edema / Bilateral renal artery stenosis
 ACEIs are nephroprotective except in acute renal injury  nephrotoxic
 If patient on ACEIs developed cough  switch to ARBs
HTN emergency: Hydralazine: 20 40 mg IV or IM, repeated as necessary, decrease the dose in
case of renal impairment

OSCE-guide-III.doc Page 120 of 255


Physical Examination

SOB shortness of breathe

Patient who had a car accident 24-48 hours ago developed SOB.
Complications: 1st day: atelectasis / fat embolism. 3rd day: DVT / PE

Common causes: Cardiac (CHF) / Lungs (asthma / COPD /


pneumonia / PE)
Brief cardio-pulmonary Life threatening conditions: MI / PE / pneumothorax / aortic
hx dissection
Vitals I would also like to rule out orthostatic hypotension (? hypovolemia)
General  General assessment of the patient
 Orientation (SOB  hypoxia  confusion)
 Face: eyes / no signs of respiratory distress / mouth (no signs of
dehydration / central cyanosis)
 Hand: temperature / capillary refill / skin / no peripheral cyanosis
/ no nicotine staining / comment on both pulses
 Sacral edema
Neck Trachea  Trachea centrality
JVP  JVP: not engorged
Chest Inspection  Symmetry
 Accessory muscles breathing / intercostal retraction
 Obvious pulsations / PMI
 If car accident (bruises on chest wall)
Palpation  Tenderness
 Tactile fremitus
 Apex
 Para-sternal heaves
Percussion  Chest back for any dullness
Auscultation  Heart (ask the patient to turn slightly to left)
 Lungs: 8 spots scan for air entry
 Lung bases
Lower limbs exam  Inspection: SEADS
 Palpation: temperature / capillary refill / peripheral edema /
pulses / press for pain in calf muscle area
 Special tests:
- Measure leg circumference 10 cm below tibial tuberosity;
difference should be < 2.5 cm between the two calves
- Homans' sign10: there is pain in the calf or popliteal region
with examiner's abrupt dorsiflexion of the patient's foot at the
ankle while the knee is flexed to 90 degrees

Indications for intubation: ABG showing poor PO2 (60s) / elevated PCO2 (80s) / acidosis / GCS
score < 8

10
A positive Homans' sign does not positively diagnose DVT (poor positive predictive value), and also negative Homans' sign does
not rule out the DVT diagnosis (poor negative predictive value), and there is theoretical possibility of dislodging the DVT.

OSCE-guide-III.doc Page 121 of 255


Physical Examination

DVT

Clots clinical probabilities / risk factors (Virchow's Triad):


Vascular injury (endothelial damage); e.g. recent surgery
Venous stasis; immobilization (post-MI, CHF, stroke, post-operative, obesity, long
travel or flight, chronic venous insufficiency) inhibits clearance and dilution of
coagulation factors
Hyper-coagulability; aging, surgery, trauma, malignancy, antiphospholipid antibody
syndrome, hormone related (pregnancy, OCP, HRT, SERMs)

DD: muscle strain or tear, lymphangitis or lymph obstruction, venous valvular insufficiency,
ruptured popliteal cysts, cellulitis, and arterial occlusive disease

Wells Clinical Score for Deep Venous Thrombosis

Clinical Parameter Score


Active cancer (treatment ongoing, or within 6 mo or palliative) +1
Paralysis or recent plaster immobilization of the lower extremities +1
Recently bedridden for >3 d or major surgery < 4 wk +1
Localized tenderness along the distribution of the deep venous system +1
Swollen unilateral superficial veins (non-varicose) +1
Entire leg swelling +1
Calf swelling >3 cm compared with the asymptomatic leg +1
Pitting edema (greater in the symptomatic leg) +1
Previous DVT documented +1
Alternative diagnosis (as likely or greater than that of DVT) -2
A Wells score can be interpreted in a binary (likely vs. unlikely) or ternary (low, moderate, or
high probability) fashion.
Ternary interpretation Binary interpretation
3 High probability score 2 DVT likely
1 or 2 Moderate probability score < 2 DVT unlikely
0 Low probability
In the unlikely group (low-to-moderate risk of DVT)  D-dimer assay (ELISA):
Negative  rule-out DVT
Positive  duplex ultrasonography
o Negative  rule-out DVT
o Positive  treat for DVT
In the likely group (moderate-to-high risk of DVT)  duplex ultrasonography
Negative  repeat clinical evaluation and ultrasonography in 1 week
Positive  treat for DVT
When discordance exists between the pretest probability and the duplex ultrasonographic study
result, further evaluation is required.
Other non-invasive tests include MRI.
Venography is the gold standard, but is expensive, invasive and higher risk

OSCE-guide-III.doc Page 122 of 255


Physical Examination

Initial treatment of DVT:


Unfractionated heparin (UFH): requires bolus (7500-10,000 ill), followed by
continuous IV infusion (1000-1500 IU/h). Advantages: rapidly reversible by
protamine. Disadvantages: must monitor aPTT
Low molecular weight heparin (LMWH): administered SC. Predictable dose response
& fixed schedule
Long-term treatment:
Warfarin: standard treatment;
o Should be initiated with heparin overlap dual therapy for at least 5 days.
Discontinue heparin after INR >2.0 for two consecutive days.
o Warfarin should be dosed to maintain INR at 2-3. Monitor INR twice weekly for
1-2 weeks, then weekly until INR stable, then every 2-4 weeks
o Duration of anticoagulant treatment (with warfarin unless otherwise noted):
First episode DVT with transient risk factor: 3 months
First episode DVT with ongoing risk factor (e.g. cancer, antiphospholipid
antibody) or > 1 risk factor: consider indefinite therapy
Recurrent DVT (2 or more episodes): indefinite therapy
IVC filters: useful in those with contraindications to anticoagulant therapy, recurrent
thromboembolism despite adequate anticoagulation, recurrent embolism with
pulmonary HTN, or those who require emergent surgery without time to initiate
anticoagulation
Special considerations:
o Pregnancy: treat with LMWH during pregnancy, then warfarin for 4-6 weeks
post-partum
o Surgery: avoid elective surgery in first month after venous or arterial
thromboembolic event
Preoperatively: IV heparin may be used up to 6 hrs pre-operatively;
warfarin should be discontinued for at least 4 days pre-op. Surgery safe
when INR <1.5
Postoperatively: IV heparin or LMWH can be started 12 hours after
major surgery

Wells Prediction Rule for Diagnosing Pulmonary Embolism

For Predicting Pretest Probability of PE (N.B. the guideline notes that the Wells rule performs
better in younger patients without comorbidities or a history of venous thromboembolism)
Clinical Characteristic Score
Previous pulmonary embolism or deep vein thrombosis + 1.5
Heart rate >100 beats per minute + 1.5
Recent surgery or immobilization (within the last 30 d) + 1.5
Clinical signs of deep vein thrombosis +3
Alternative diagnosis less likely than pulmonary embolism + 3
Hemoptysis +1
Cancer (treated within the last 6 mo) +1

Score Clinical Probability of PE Simplified Wells


0-1 Low score > 4 PE likely
2-6 Intermediate score 4 PE unlikely
7 High

OSCE-guide-III.doc Page 123 of 255


Physical Examination

Evaluation of suspected PE: Notes:


Low clinical probability of PE  D-dimer: 1. Use D-dimers only if low clinical
o Negative: ruled out probability, otherwise, go straight to spiral
o Positive  CT scan with contrast: CT or V/Q
Negative: ruled out 2. If using V/Q scan (CT contrast allergy or
Positive: ruled in renal failure):
Intermediate OR high probability: Negative V/Q scan rules out the diagnosis
o CT scan: Inconclusive V/Q scan requires leg US
Negative: ruled out duplex to look for DVT (q2d)
Positive: ruled in High probability V/Q scan only rules in the
diagnosis if have high clinical suspicion

CXR of PE: may be normal / wedge-shaped infiltrate / unilateral effusion / raised hemi-
diaphragm

Treatment of PE:
Admit for observation (patients with DVT only are often sent home on LMWH)
Oxygen: provide supplemental O2 if hypoxemic or short of breath
Pain relief: analgesics if chest pain narcotics or NSAIDs
Acute anticoagulation: therapeutic-dose SC LMWH or IV heparin start ASAP
o Anticoagulation stops clot propagation, prevents new clots and allows
endogenous fibrinolytic system to dissolve existing thromboemboli over months
o Get baseline CBC, INR, aPTT renal function liver function
o For SC LMWH: dalteparin 200 U/kg once daily or enoxaparin 1 mg/kg bid no
lab monitoring avoid or reduce dose in renal dysfunction
o For IV heparin: bolus of 75 U/kg (usually 5,000 U) followed by infusion starting
at 20 U/kg/hr aim for aPTT 2-3 times control
Long term anticoagulation:
o Warfarin start the same day as LMWH/heparin start at 5 mg PO od overlap
warfarin with LMWH/heparin for at least 5 days and until the INR is in target
range of 2-3
o LMWH instead of warfarin for pregnancy; active cancer, high bleeding risk
o Duration of long-term anticoagulation treatment:
If reversible cause for PE (surgery, injury, pregnancy, etc.): 3-6 months
If PE unprovoked OR ongoing major risk factor (active cancer):
indefinite
IV thrombolytic therapy:
o If patient has massive PE (hypotension or clinical right heart failure)
o Hastens resolution of PE but may not improve survival or long-term outcome
Interventional thrombolytic therapy (massive PE is preferentially treated with
catheter directed thrombolysis by an interventional radiologist, works better than IV
thrombolytic therapy and fewer contraindications)
IVC filter: only if recent proximal DVT + absolute contraindication to
anticoagulation

OSCE-guide-III.doc Page 124 of 255


Physical Examination

Peripheral Arterial Disease Examination

Introduction
Vitals Based on the vitals, the patient is stable, I would like to proceed
General
Inspection  Drape the patient / expose the lower limbs (triangular)
 I would like to take a look at your feet, can you please remove the socks; do
you want me to help you?!
 SEADS
 No signs of arterial insufficiency: no hair loss / no shiny tight skin / no
hypertrophic nails
Palpation  Temperature
 Capillary refill (< 3seconds)
 Pulses: dorsalis pedis / posterior tibial / popliteal / (to examiner) I would like
to check the femoral arteries
Abdomen  Drape the patient / I would like to examine your abdomen / can you please
uncover your abdomen
 Listen for bruits (aortic / renal / iliac)
Neurology  Check for light touch, here is a piece of cotton, this is how it feels; can you
please close your eyes! Tell me when you feel it touching you! Check both
lower limbs from distal to proximal
 If light touch is ok, do not proceed with more tests
Burger test I would like to raise your legs, for 1-2 minutes, if you feel any pain / numbness /
tingling please let me know, check the color of the foot. Then dangle the feet and
check the color  no pallor on elevation, no rubor on dependence  Burger test
is negative.
Special tests I would like to arrange for ankle / brachial index

Investigations:
CBC
Fasting blood sugar / lipid profile
ECG
Angiography (side effects: nephrotoxic / allergy / aneurysm risk)
Doppler U/S study of the arterial tree both lower limbs

Treatment:
Life style modifications (refer to HTN)
Foot care
Graded exercise
Surgery (if severe disability)

OSCE-guide-III.doc Page 125 of 255


Physical Examination

Diabetic Foot

Diabetic patient with long hx of diabetes, has an ulcer for few days

Introduction
Vitals Based on the vitals, the patient is stable, I would like to proceed
General
Inspection  Drape the patient / expose the lower limbs (triangular)
 I would like to take a look at your feet, can you please remove the socks; do
you want me to help you?!
 Describe the ulcer: location (in the sole at base of 1st metatarsal), shape
(round, irregular), size ( cm), margins not elevated, no active
bleeding or oozing
 No other ulcers in the same foot / check the other foot / check in
between toes  no evidence of infection in between toes / in nails
 No pigmentation around the medial and lateral malleoli
 SEADS quadriceps wasting / swollen joints
 No signs of arterial insufficiency: no hair loss / no shiny tight skin / no
hypertrophic nails
Palpation  Temperature
 Capillary refill (< 3seconds)
 Pulses: dorsalis pedis / posterior tibial / popliteal / (to examiner) I would
like to check the femoral arteries
Abdomen  Drape the patient / I would like to examine your abdomen / can you please
uncover your abdomen
 Listen for bruits (aortic / renal / iliac)
Neurology  Check for light touch, here is a piece of cotton, this is how it feels; can you
please close your eyes! Tell me when you feel it touching you! Check both
lower limbs from distal to proximal
 If light touch is ok, do not proceed with more tests
 Light touch sensation is absent distal to the level of cm above ankle
 Proprioception: I will move your toe, close your eyes please, tell me is
it up or down. Then move to the next joint. Start with head of
metatarsal, medial malleolus, tibial tuberosity,
 Vibration: tuning fork, here is the sensation you will feel, tell me when
it stops  intact / decreased / absent
 Monofilament test: to distinguish between the light touch and pressure
sensation / 10 points on the foot (9 on the sole, and one on the dorsum
above the big toe meta-tarso-phalangeal joint)
 Ankle reflex; if you have time: knee reflex and Babinski

Burger test
Special tests I would like to arrange for ankle / brachial index

Balance (if  Gait ataxia


DM focused  Romberg test: positive (with open eyes: pt can balance himself by vision;
exam case) while with closed eyes: pt loses balance

OSCE-guide-III.doc Page 126 of 255


Physical Examination

Neurological Examination

o Introduction
o Vital signs
o General inspection of the patient: pt is sitting comfortably

- Orientation: what is your name sir? Where are you? Time? Place?
- Cranial nerves
- Upper and lower extremities:
o Inspection
o Palpation / bulk
o Tone:
 Just relax please, let me do everything for you. I am going to check the
tone in your Rt arm
 Tone is normal, no hypo or hyper tonia
o Motor power (5  0)
 5 full power
 4 less than full power (like Lt hand in Rt handed person)
 3 can do the movement against gravity
 2 can do the movement with the gravity eliminated
 1 muscle twitches, not able to initiate movements
 0 no power no movements
o Sensory:
 Light touch:
Pin prick or piece of cotton
First check on forearm or sternum
Can you close your eyes please
Distal to proximal
Bilateral  sensation is equal bilaterally
 Posterior column (B12 deficiency / alcohol / syphilis):
Vibration sense: tuning fork / test on sternum / tell me when it
stops / start distal / if intact move on / if not intact go proximal
on the next joint
Proprioception: eyes closed / start with the big toe or thumb / is
it moving or not? / is it up or down?
o Reflexes:
 0 absent
 1 weak (hyporeflexia)
 2 normal
 3 hyper reflexia
 N.B. Babinski reflex: I am going to tickle the bottom of your foot:
Planter flexion: normal response
Big toe dorsiflexion and toes fanning: UMNL (e.g. stroke)

OSCE-guide-III.doc Page 127 of 255


Physical Examination

- Gait ATAXIA
o Can you take few steps for me please?
o Protect the patient, surround him with your arms, and walk with him
- Romberg test
o Can you put your legs together!
o Can you close your eyes please!
o Watch (protectively) for few seconds!
 Ataxia due to peripheral neuropathy (B12 deficiency / DM / syphilis):
with eyes closed
 Cerebellar ataxia: no with closed eyes (always on)
- Cerebellar signs (stroke / alcohol / tumours / para-neoplastic / ):
o Nystagmus:
 Can you follow my finger please (move it side to side)
Physiological: transiently then corrected
Central: horizontal or vertical
Peripheral: horizontal only. Conditions: benign positional vertigo
/ acute labyrinthitis / drugs
o Finger to finger:
 Patient hand must be extended
 Move the examiner hand
 Check both upper limbs
o Finger to nose test: lesion in the cerebellum on the same side.
 Intentional tremors
 Loss of coordination
o Heel to shin: lesion in the cerebellum on the same side

- Cortical sensations: two points discrimination


- Mini-mental status exam

OSCE-guide-III.doc Page 128 of 255


Physical Examination

Cranial Nerves Examination

- Vital signs
- Comment on the patient general condition

CN I:
- Do you have problems with smells?
Can you please close your eyes?
- What is that? Coffee / ammonia
- What is that? Ammonia / coffee

CN II:
The optic nerve:
- Visual acuity: Do you wear glasses? reading / color (Snellen chart at 1 foot distance 35
cm)
- Visual fields: eye by eye / by confrontation (when you see my fingers wiggling)
- Pupillary reflex: I am going to shine light in your eyes, please look straight to the wall,
each eye: direct and consensual (afferent: CN II, efferent: CN III)
- I would like to do fundoscopy examinations, looking for: disc edema, retinal hemorrhage,
neovascularisation, nipping of the veins

CN III, IV, VI:


- By inspection; both eyes are symmetrical, no deviation, no nystagmus, no head tilting, no
ptosis (CN III: opens, CN VII: closes)
- I would like you to follow this pen, without moving your head please, and if you see
things double or blurred at any time, please let me know  move the pen in large H-
shaped manner, then conversion
- Normal extra-ocular muscles movements, no nystagmus or double vision

CN V:
- Motor:
o By inspection: no atrophy of the temporal or masseter area
o Can you please clench, feel the temporalis and masseter
o Can you open your mouth against my hand?
- Sensory:
o This is a piece of cotton, and this is how it feels, I am going to touch your face,
and whenever you feel it, please tell me. Can you close your eyes please?
o Touch the face in symmetrical areas; cover the ophthalmic, maxillary, and
mandibular areas. Does it feel the same?
o Facial sensation of the trigeminal nerve is intact and equal on both sides
- Reflexes:
o Corneal reflex (afferent: CN V, efferent: CN VII)

OSCE-guide-III.doc Page 129 of 255


Physical Examination

CN VII: (mainly motor)


- Motor:
o Face is symmetrical, no deviation of the angle of the mouth, normal naso-labial
folds, no drooling
o Can you please copy me:
 Raise your eye brows, can you frown (wrinkle your forehead) please?
 Can you close your eyes, and do not let me open them?
 Can you blow your cheeks? Whistle?
 Can you show me your teeth?
o If there is a mouth angle deviation, it is deviated to opposite side of lesion
o UMNL vs. LMNL:
 UMNL: intact upper face muscles (the nucleus receives double sided
innervation)
 LMNL (Bells palsy)  ptosis; all face muscles are affected
- Sensory:
o To complete the facial nerve examination, I would like to check the sensations in
the outer 2/3 of the tongue
o I would like to check the ext auditory canal and the tympanic membrane to rule
out herpes zoster of the facial nerve
- Reflexes:
o Jaw reflex
o Corneal reflex (afferent: CN V, efferent: CN VII)

CN VIII
- Check by whispering (ABC CBA), while rubbing fingers in front of the other ear OR
by rubbing your fingers
- Because the hearing is normal, I am going to skip Weber and Rinne tests
o Rinne: place the tuning fork in front of ear, then on the mastoid process
o Weber: place the tuning fork on the forehead

CN IX, X:
- Patient voice is normal, no hoarseness
- Can you swallow a sip of water please? Normal swallowing
- Can you open your mouth please? Soft palate is symmetrical, uvula is central
o Uvula deviates to the opposite side of the lesion
- To check the reflexes: I need to do the gag reflex

CN XI:
- Can you please shrug your shoulders?
- Turn your head to the right, and to the left. I am going to resist you. I feel for the opposite
side sterno-mastoid

CN XII:
- Can you please open your mouth? Can you stick your tongue out?
o Tongue is central, no deviation. No fasciculations or atrophy of tongue.
o If there is a lesion, the tongue deviates towards the lesion side
- Can you please move it to the right and to the left? Can you stick it against your cheeks?
 Normal movements of the tongue

OSCE-guide-III.doc Page 130 of 255


Physical Examination

Tremors

? Parkinson disease

Introduction Shaky right hand


Vital signs I would like to rule out any orthostatic hypotension
General comment Pt is sitting comfortably , with Rt hand tremors

Inspection Tremors Right hand tremors, not obvious on the left hand
 Count from 10 to 1 please  tremors with mental activity 
consistent with Parkinson disease, and rules out anxiety
 Stretch your hand plz / no fine tremors  r/o hyperthyroidism
 No flapping tremors  rule out liver failure
 Finger to nose / no intentional tremors  r/o cerebellar dis
Patient tremors consistent with Parkinson disease, resting tremors,
beads rolling, and limited to Rt hand (). No head nodding
Face (No) limited facial expression, decreased eye blinking, drooling
Palpation Check the wrist and elbows:
Rigidity  (No) cog wheeling Positive with parkinsonism
 (No) lead pipe rigidity
 (No) clasp knife spasticity Positive with stroke
Standing / walking  Would you please stand up! Do you need help  patient finds
Postural instability difficulty in standing up
 Can you walk few steps for me please: comment with + or -
- Stooped posture
- Shuffling (festinating) gait
- Decreased arm swinging
- Patient turns in blocks
Special Tests  Rapid alternating movements (hand supination & pronation /
oppose thumb to fingers)  dysdiadochokinesia
 Can you please repeat British constitution  monotonous
 Can you write a sentence for me  micrographia
 Can you draw a spiral parallel to this (draw spiral on paper)
I would like to do the mini-mental status exam

Treatment of Parkinsonism:
 Pharmacologic
Mainstay of treatment: Sinemet (levodopa / carbidopa). Levodopa is a dopamine
precursor, carbidopa decreases peripheral conversion to dopamine
o Levodopa related fluctuation: delayed onset of response (affected by mealtime),
end-of-dose deterioration (i.e. wearing-off), random oscillations of on-off
symptoms
o Major complication of levodopa therapy is dyskinesias
Treatment of early PD: DA agonists, amantadine, MAOI
Adjuncts: DA agonists, MAOI, anticholinergics (especially if prominent tremors),
COMT inhibitors
 Surgical: thalamotomy, pallidotomy, deep brain stimulation (thalamic, pallidal, subthalamic),
embryonic dopaminergic stem cell transplantation

OSCE-guide-III.doc Page 131 of 255


Physical Examination

Thyroid Exam

Introduction
Vital signs BP, HR
General  Can you stretch your hands:
- Fine tremors
- Palms for sweating
- Nail changes
- Hair loss (hypothyroidism)
 Examine the eyes:
- Exophthalmos stand by the patient (stand behind the right shoulder
and look from above)
- Lid lag (can you follow my finger without moving your head from
above downwards)
 Proximal muscle weakness:
- Can you shrug your shoulders (bilaterally against my hand) please
 Knee reflex: brisk11 reflex
 Peritibial myxedema: indicates hyper-thyroidism
Thyroid Exam Patient is sitting on a chair
Inspection Can you swallow12 please?  no apparent thyroid enlargement
Palpation  Thyroid gland:
- From behind the patient, bi-manually
- Then while swallowing a sip of water  thyroid movement is normal,
I do not feel any masses, nodules, and no tenderness
 Lymph nodes:
- Sub-mandibular and cervical
Percussion  DIRECT percussion on upper part of sternum
 Checking for retro-sternal extension (no retro-sternal dullness)
Auscultation  BOTH lobes
 For thyroid bruits

11
Reflexes grades: 0 absent 1 hypo 2 normal 3 hyper (brisk) 4 hyper with clonus (ankle)
12
Whenever you ask the patient to swallow, give a sip of water, it is difficult to swallow on an empty mouth

OSCE-guide-III.doc Page 132 of 255


Physical Examination

Dermatomes

OSCE-guide-III.doc Page 133 of 255


Physical Examination

Neck Examination

Patient complaining of pain in the neck

Vitals  Can I get the vital signs please


General  Comment on patient general condition: patient is sitting comfortably
Inspection  I would like to take a look at your neck; can you lower your gown please?
 From the back: SEADS (Swelling / Erythema / Atrophy / Deformity / Scars)
 From the side: normal cervical and thoracic curvatures
 Can you cough please, any pain
- No neck pain with Valsalva manoeuvre
Palpation  I would like to feel you back please
- Temperature is normal
- Tenderness: pressing on the spinal processes and para-vertebral muscles,
trapezius and sternomastoid
 Thyroid: can you swallow for me please?
 Check the LNs: no enlarged LNs
ROM  Can you touch your chest by your chin?
 Can you look to ceiling?
 Can you turn your head to the right? And to the left?
 Can you tilt your head? To the right and to the left?
- Normal flexion, extension, rotation and lateral flexion
Powers  Test again against resistance
- Neck pain is not associated with muscle contractures
Special tests  Spurling test: stretching the nerve  reproduces the pain
 Upper limb examination and neurological screening

Part of my exam is to check your upper extremities, can you roll up your sleeves please!
Inspection Upper extremities are symmetrical, normal bulk, no atrophy / SEADS
Palpation I am going to feel your shoulder; deltoid, biceps, triceps, forearm, thenar,
hypothenar  are symmetrical / no deformity / no atrophy
Motor Power Deltoid C5 Biceps C5/6 Triceps C7/8
Sensory C4: deltoid C5: biceps lateral aspect Test light
Neurological

C6: thumb C7: middle finger C8: touch


little finger
T1: elbow medial aspect
screen

Reflexes  Biceps and brachio-radialis C5/6


 Triceps C7/8
Pulse Radial pulse

Post encounter: what is the level of the lesion?


C6 nerve root lesion (C5-C6): weak biceps, weak biceps reflex
C7 nerve root lesion (C6-C7): weak triceps, weak triceps reflex
Diagnosis: Osteoarthritis of the cervical spine at level
X-ray findings: osteophytes of the cervical vertebrae / narrowing of disc space / subchondral
sclerosis (increased bone formation around the joint), subchondral cyst formation
Management:
Soft neck collar
NSAIDs / acetaminophen
Physiotherapy
If worsening: neurosurgery consult / CT myelography / nerve conduction studies

OSCE-guide-III.doc Page 134 of 255


Physical Examination

Carpal Tunnel Syndrome


Hand pain for 6 months
History
Analysis CC  Os Cf D / PQRST /
 Radiation: how about your elbow? Your fingers?
 Triggers: is it related to time? To work? What do you do?
AS  Weakness, numbness, tingling
 Swelling, redness, warm
 How about the other hand?
Impact  How did this affect your life? Your work?
Red flags  Constitutional symptoms
Differential  Hx of neck pain / injury / trauma
diagnosis  Hx of diabetes / symptoms
 Hx of thyroid disease / symptoms of hypothyroidism
 Hx of autoimmune disease / Rh arthritis / symptoms
 Headache / visual disturbances / shoes are getting tight
 Pregnancy / LMP
PMH / FH

Physical examination
Vital signs
Inspection  SEADS (thenar / hypothenar ms)
 No nail changes, no nodules / no deformity
Palpation  Temperature: is normal
 Tenderness: palpate distal radial bone, styloid process, joint line, styloid
process, distal ulnar bone, base of the thumb, carpal bones, metacarpal
bones, digits
ROM  Flexion / Extension /+/ move your hand to the right, to the left
 Can you make a fist / fan your fingers  there is no obvious damage to
the nerves / muscles / and tendons of the hand
 Thumb movements:
- Touch base of your little finger (thumb opposition)
- Move it all the way to opposite direction
- Point to the ceiling (with hand supine, flat)
- Touch the tips of your fingers
Power  Like ROM but against resistance
 Thumb 90, DIP flexed: do not let me straighten it
 Biceps ROM / against resistance / biceps reflex (C6)
Sensory  Check with cotton tip,
 For the ring finger: check both sides: ulnar / radial
Special tests  Phalen's test,
 Tinel's sign / tap on the carpal tunnel
Investigations: EMG / nerve conduction studies
Treatment:
Modify nature of work
NSAIDs
Wrist splint
Local corticosteroids injection
Surgical decompression

OSCE-guide-III.doc Page 135 of 255


Physical Examination

Hand Laceration / Wrist Laceration


Physical examination
Introduction Gloves / vital signs
Inspection  If big wrist laceration (BIG band aid): DO NOT remove the band aid,
should be removed under anesthesia
 If small laceration, remove the band aid  describe the laceration: linear /
size / position / elevated margins / active bleeding / oozing
 Other ulcers / other hand
 SEADS (Swelling / Erythema / Atrophy / Deformity / Scars)
 No nodules / no deformity
Palpation  Temperature: is normal, is hand pink and warm?
 Tenderness: palpate distal radial bone, styloid process, joint line, styloid
process, distal ulnar bone, base of the thumb, carpal bones, metacarpal
bones, digits
 Capillary refill
 Feel the pulse  radial ulnar
 Allens test
ROM  Do NOT ask patient to apply force against resistance as this may rupture
a partially severed tendon. Test active ROM only
 Median nerve: thumb abduction (thumb to ceiling)
 Ulnar nerve: finger spread (fanning)
 Radial nerve: wrist extension
 Tendon examination of the hand (flexor tendons):
- DIPs; flexor digitorum profundus: hold the MCP / PIP joints in
extension, and ask the pt to flex DIPs.
- PIPs; flexor digitorum superficialis: hold all fingers except one you
are testing in extension and ask pt to flex the remaining finger
- MCP joints: intrinsic (lumbricals)
Power  Do NOT ask patient to apply force against resistance as this may rupture
a partially severed tendon. Test active ROM only
Sensory  Check with cotton tip or pin-prick, for light touch
 Check two point discrimination on either side of each digit
 For the distribution of the three nerves: ulnar / radial / median
Neuro-motor examination of the hand
Median Ulnar Radial
Sensory Radial aspect of index finger Ulnar aspect of little finger pad Dorsal web space of thumb
pad
Motor Flex DIP if index finger (FDP) Flex DIP of little finger (FDP), Extend wrist and thumb,
extrinsic extensor carpi radialis (extensor pollicis longus)
Motor Thumb to ceiling with palm up Abduct index finger (first
intrinsic (abductor pollicis brevis) dorsal interosseous)

Structures lacerated
Diminished ulnar territory sensation Ulnar nerve
Allen test shows (no) refill from the ulnar circulation Ulnar artery
FDS weakness in little finger and ring finger Flexor retinaculum, ulnar two divisions of FDS

Management: clean and explore wound under local anesthesia and sterile conditions. Consult plastic
surgery for micro-vascular repair. If at night, may suture the skin and arrange for pt to be seen by plastic
surgeon next day.

OSCE-guide-III.doc Page 136 of 255


Physical Examination

Back Pain

Acute  Herniated disc Neurological exam


 Muscle spasm
Acute on top
of chronic
Chronic Pain at end of day shoots to legs, increases when leaning
forward:
 Disc herniation
 Degenerative disease (OA)
Pain related to position, increases when extension (or
leaning backwards), walking down the hill (arching the
back)  spinal canal stenosis
Pain wakes patient in morning, improves at end of day, Mechanical exam
stiff  Ankylosing spondylitis  limitation in all
directions

Red flags for back pain:


BACK PAIN
 B bladder or bowel dysfunction
 A anaesthesia (saddle)
 C constitutional symptoms  malignancy
 K chronic disease

 P parathesia
 A age > 50 years old
 I IV drug user
 N neuro-motor deficits

OSCE-guide-III.doc Page 137 of 255


Physical Examination

Acute Back Pain

Acute back pain: pain for 3 days


Acute on top of chronic: pain for 3 hours old patient

Introduction How do you feel?


Do you prefer to remain standing, sit, or lie down?
That is ok, whenever you want to lie down or rest, feel free to do so
Analysis Os Cf D  What were you doing? What do you do for work?
 Did you hear any crack sound?
 Were you able to stand up and continue? Were you able to
move? Did anyone help you?
 Is this the first time?
 How about at rest? During night?
 Recently did you have any discomfort? Milder back pain?
How did it affect you?
PQRST  Does it shoot to your thighs? Toes?
 Which is bothering you more; your back or your legs?
 How about lying down? Stretch your back? Coughing?
Moving? Leaning forward or backward?
 Any medication? Did it help?
Impact  How does it affect you?
 Mets to lungs / liver / brain
Red flags  Constitutional symptoms
 Overweight
In addition to your pain, did you notice any other symptoms:
 Weakness, numbness, tingling
 Difficulty with balance, falls
 Any difficulty passing urine? How about bowel Cauda equine
movement? Did you find that you soiled yourself?
 Any numbness in the buttocks area?
 Do you have morning erection? Any sexual dysfunction?
DD
PMH Cancer prostate / bladder
FH Cancer prostate
SH

OSCE-guide-III.doc Page 138 of 255


Physical Examination

Chronic Back Pain


Pain for 3 months

Introduction Do you prefer to remain standing, sit, or lie down?


That is ok, whenever you want to lie down or rest, feel free to do so
Analysis Os Cf D  What were you doing? What do you do for work?
PQRST  Quality: stiffness
 Timing: is it worse in morning? Improves with time? Or is it worse
at the end of the day?
 Does it shoot to your thighs? Toes?
 Which is bothering you more; your back or your legs?
 How about lying down? Stretch your back? Coughing? Moving?
Leaning forward or backward?
 Any medication? Did it help?
AS  Pain other joints (knees / hips / hands) / distribution? Osteoarthritis
Sero-  Pain other joints (knees / hips / hands) / distribution? Ankylosing
negative  Eye pain, redness / mouth ulcers spondylitis
symptoms  Skin changes / nail changes / hx of psoriasis
 Repeated attacks of abd pain / diarrhea
 Urethral discharge
Impact  How does it affect you?
 Mets to lungs / liver / brain
Red flags  Constitutional symptoms
 Overweight
In addition to your pain, did you notice any other symptoms:
 Weakness, numbness, tingling
 Difficulty with balance, falls
 Any difficulty passing urine? How about bowel movement? Did you Cauda equine
find that you soiled yourself?
 Any numbness in the buttocks area?
 Do you have morning erection? Any sexual dysfunction?
DD
PMH  Trauma
 Injury to back
FH  Osteoarthritis
 Ankylosing spondylitis
 Other rheumatic disease
SH

Osteoarthritis: older patient / worse at evening

Ankylosing spondylitis:
Morning stiffness improves by time
LSS x-ray: sacroiliitis OR fusion of SI joints
ESR:
HLA-B27 tissue antigen: positive
Associated symptoms: inflammatory arthritis / Uveitis / psoriasis / IBD / pericarditis / aortic regurgitation
Management:
No cure
Regular therapeutic exercises to prevent deformity (swimming / back extension exercises)
NSAIDs: Indomethacin (50 mg PO bid) or Naproxen (250 mg PO bid)
In severe cases: total joint replacement

OSCE-guide-III.doc Page 139 of 255


Physical Examination

Back Joint Examination

Physical examination
Introduction Can you stand up please?
Vital signs
Inspection  Gait / balance / stance
 Ask the patient to stand up from sitting position
 Posture: normal cervical, thoracic, lumbo-sacral curvatures
 Adams forward bend test (if scoliosis: the scapula will be higher)
- No scoliosis or kyphosis
 SEADS
Palpation  Temperature
 Tenderness: spinal processes, para-vertebral muscles, sacro-iliac joints
(medial to dimples of Venus)
ROM  Can you touch your toes with your fingers? Without bending knees
 Can you arch your back? Without bending knees (stand supported by the
bed foot: will not fall, less possibility of knee bending)
 Slide your arms on both sides (Rt and Lt)? (stand against wall, normally
the tips of finger travel > 10 cm)
 Cross your arms? Turn to the Rt and Lt (pt sitting on bed)
 Modified Schober's test: (midline, between the dimples of Venus) + 5
cm below + 10 cm above  bend forward  N> 6 cm diff.
Special tests  Occiput-to-wall distance (tragus & nose same level): normally zero
 Straight leg raise (irritation of the roots of sciatic n: L4/L5/S1/S2):
elevate the lower extremity straight, when it is painful  where it does
hurt?  straight leg test positive
 Decrease the angle, try to dorsiflex foot  Lasgue sign
 Cross straight leg raise test: elevate the other LL  trigger pain
 Fabers test (figure 4 test): to check sacro-iliac joint pathology
 Femoral nerve stretch (done for patients c/o pain radiating to the anterior
aspect of the thigh): patient prone, knee flexed,
Motor  Hip flexion (L1/L2/L3) / extension (S1/S2)
 Knee flexion (L5/S1/S2) / extension (L2/L3/L4)
 Ankle dorsiflexion (L4/L5) / plantar flexion (S1/S2)
Neurological screen

 Can you walk on your heals? Normal L4/L5 muscles


 Can you walk on your toes? Normal S1/S2 muscles
Sensory S1: little toe L5: first web L4: medial malleolus
L3: knee med L2: thigh ant L1: groin
T10: umbilicus
Reflexes Knee (L2/L3/L4 mainly L4) / Ankle (S1/S2 mainly S1) / I would like to do
the Babinski reflex (positive in UMNL)
Pulse Dorsalis pedis

Other clinical examinations: DRE; to rule-out cauda equina (sphincter weaknesses, reduced anal
tone)
N.B. dimples of Venus correspond to PSIS

OSCE-guide-III.doc Page 140 of 255


Physical Examination

Ankle Twist

Young man comes with ankle twist; history and physical examination are normal, no fractures,
and no lacerations. In the next 10 minutes counsel him about the treatment

History  Mechanism of trauma / injury


 Noise heard at the time of trauma
 Where you able to walk after the injury?
Inspection Symmetry / SEADS
Palpation  Temperature
 Tenderness: med malleolus / lat malleolus / ant joint line (between malleoli)
/ lat ligaments (slightly ant and inferior to the lat malleolus) / med ligaments
(inferior and slightly ant to the med malleolus) / tendon Achilles / navicular
head (medial bony prominence) / fifth meta-tarsal head (lat) / around the
meta-tarsal heads / origin of the plantar fascia (inf med aspect of calcaneus )
 Crepitus
ROM Patient is sitting on the side of the bed
 Dorsi-flexion (point toes up) and plantar flexion (point toes down)
 Inversion (point the bottoms of your feet towards each other) and eversion
(the opposite direction)
Power ROM against resistance / after stabilizing the ankle with your other hand
Special tests  Talar drawer sign: 1 cm is significant, indicates ant talo-fibular lig rupture
 Talar tilt: lat calcaneo-fibular lig rupture (inversion), med lig rupture
(eversion)
 Squeeze test (squeezing calf  ankle pain); if negative  NO ankle
fracture

Investigations: x-ray
 Ottawa ankle rules; for ankle series:
o Pain in the malleolar zone and any one of the following:
 An inability to bear weight both immediately and in the emergency
department for four steps
 Bone tenderness along the tip of the medial or lateral malleolus
 Ottawa foot rules; for foot series
o If there is any pain in the mid-foot zone and any one of the following:
 An inability to bear weight both immediately and in the emergency
department for four steps
 Bone tenderness at the base of the fifth metatarsal
 Bone tenderness at the navicular bone

Management:
 Complete tear should be evaluated by orthopedics  stat orthopedics consult
 RICE: rest (and crutches) / ice for 20 min QID x 3 days / compression (by tensor bandage) /
elevation
 Pain medication: NSAIDs; e.g. Ibuprofen 400 mg, PO, q6h.
 Show him how to wrap it, remove the wrap, and ask him to wrap it again (to make sure he
knows how to). Remember: from distal to proximal and 1/3 width overlap.
 Show him how to use the crutches.

OSCE-guide-III.doc Page 141 of 255


Physical Examination

Shoulder Joint
History - Trauma to shoulder / neck? X-ray done? What is your occupation?
- Neurological deficits? How does it affect your life?
Vital signs
General Patient condition (restlessness, discomfort, willingness to move)
Inspection - Both shoulders symmetrical / clavicle level / scapula level / deltoid
- SEADS (Swelling / Erythema / Atrophy / Deformity / Scars)
Palpation - Temperature: compare
- Tenderness: sternal notch / sterno-clavicular joint / clavicle / acromio-
clavicular joint / deltoid / long head of the biceps / insertion of the rotator
cuff muscles / spine of the scapula / medial border of scapula / spinal
processes of the cervical spine
- Crepitus
ROM - Active ROM: can you copy me please:
- Abduction  and comment on painful arc test
- Adduction  and comment on drop arm test
- Forward flexion (180) /+/ Backward extension (60)
- External rotation /+/ Internal rotation
- Another faster way to check:
- Hands behind your neck (abduction / ext rotation)
- Hands behind back (adduction / int rotation) between shoulder blades;
touch the tip of the contra-lateral scapula.
- Passive ROM: If patient is unable to complete the whole range of
movements actively, complete the ROM passively and comment (in
inflammation: passive ROM is > active ROM)
Power - Like the ROM, but against resistance
Special tests 1 Painful arc (between 60 and 120) All these tests are done to
2 Drop arm test  complete tear of supra- test for subacromial
spinatous tendon impingement of supra-
3 Neers test spinatous
4 Hawkins test
5 Jobes test (empty can test)
6 Lift-off test: try to push my hand away from Sub-scapularis
your back
Yergasons test; palm face up test: shake
7 For bicepital tendinitis
hands, try to let your palm face upwards, I
will resist you, and press on your shoulder
8 Speeds test: supine, semi-flexed, do not let
me push your arm down
9 Stability testing: For joint stability
+ Push ant / post
+ Pull down  sulcus sign
10 Apprehension test (ant and post): for
dislocation
To complete my exam, I would like to do:
- Check the pulses of the upper limb (radial / ulnar / brachial)
- Brief neurological examination of the upper limb
- One joint above and one joint below examination (cervical spine / elbow)
- The other shoulder examination

OSCE-guide-III.doc Page 142 of 255


Physical Examination

Rotator cuff muscles:


Supra-spinatous Abduction
Infra-spinatous External rotation
Teres minor
Sub-scapularis Internal rotation Lift-off test

Impingement syndrome:
- The most common symptoms in impingement syndrome are pain, weakness and a loss of
movement at the affected shoulder
Treatment:
- Mild: RICE / NSAIDs / PT. Rest (cessation of painful activity), ice packs and NSAIDs
may be used for pain relief. Physiotherapy (PT) focused at maintaining range of
movement and avoiding shoulder stiffness.
- Moderate: therapeutic injections of corticosteroid and local anesthetic may be used for
persistent impingement syndrome
- Severe: surgery

Investigations:
- U/S

Possibilities:
- Normal shoulder exam
- Frozen shoulder stiff, with limited active and passive ROM (ttt:
physiotherapy, NSAIDs, steroids)
- Bicepital tendinitis +ve palm face up test / speed test
- Repeated ant dislocation positive apprehension test
- Rotator cuff tear
o Complete tear drop arm test (ttt: surgery)
o Partial tear pain with initiation of movement / +ve empty can test (ttt:
physiotherapy, NSAIDs, steroids, surgery)
- Rotator cuff tendinitis similar to partial tear / +ve impingement test
- Sub-deltoid bursitis

Elbow

ROM: flexion / extension /+/ pronation / supination

Tennis elbow (lateral epicondylitis)


 With the elbow fully extended, there are points of tenderness over the lateral epicondyle
(origin of the extensor carpi radialis brevis muscle).
 Cozen's test: pain with passive wrist flexion and resistive wrist extension.[
 X-rays are used to confirm and distinguish possibilities of existing causes of pain that are not
related to Tennis Elbow, such as fracture or arthritis.

Golfer's elbow (medial epicondylitis)


 The common tendinous sheath is inserted into the medial epicondyle of the humerus
 Treatment:
o NSAIDs: ibuprofen, naproxen or aspirin /+/ Heat or ice
o A counter-force brace or "elbow strap" to reduce strain at the elbow epicondyle, to
limit pain provocation and to protect against further damage.

OSCE-guide-III.doc Page 143 of 255


Physical Examination

Hip Joint

+ Middle-age male with septic arthritis


+ Elderly female with osteoarthritis

Vital signs
General -
Patient condition (restlessness, discomfort, willingness to move)
May I ask for full exposure please?
-
Inspection -
Hip joint is deeply seated joint, I am looking for the surroundings
-
SEADS (Swelling / Erythema / Atrophy / Deformity / Scars)
-
Scoliosis / kyphosis / pelvic tilt (level of both iliac crests)
Gait: no wide stance, shuffling, drop foot, or antalgic gait
-
Balance: Trendelenberg sign; standing on one leg (while the patient is putting his
-
arms on the examiner shoulders), the pelvis drops
Palpation - Temperature: compare
- Tenderness: ASIS, iliac crest, PSIS, sacro-iliac joint, greater trochanter of the
femur.
I would like to check symphysis pubis and inguinal ligament.
- Crepitus: over femoral head (lat to femoral art, below inguinal lig)
ROM Active ROM, each one followed immediately by passively stressing (increasing) the
ROM while patient is lying
- Forward flexion (120)
- Internal rotation (30) /+/ External rotation (45)
- Create space, stabilize the contra-lateral hip with your left hand: Abduction (45)
/+/ Adduction (30)
- Backward extension (while lying prone): stabilize the lower back by your left hand,
can you lift your thigh (20)
Power - Resisted isometric testing (patient lying supine)
Special tests - Figure 4 test (Patrick or Fabers test)  the leg of the examined side flexed and
externally rotated with the ankle resting on the patella of the contra-lateral leg. The
examiner applies counter-pressure at the opposite hemi-pelvis, and applies gentle
downward force on the knee. Post hip pain indicates sacro-iliac joint pathology,
while ant lat hip pain may suggest hip joint pathology
- Thomas test  put your hand under pt LSS, and try to max flex the contra-lateral
knee
- True leg length  from ASIS (anterior superior iliac spine) to medial malleolus
on both sides
- Apparent leg test  from umbilicus to medial malleolus
To complete my exam, I would like to do:
- Check the pulses of the lower limb (dorsalis pedis / posterior tibial / popliteal)
- Brief neurological examination of the lower limb
- One joint above and one joint below examination (LSS / knee)
- The other hip examination

Septic arthritis:
Physical exam: fever / very painful joint / +ve trendlenberg test / restricted movements on all directions
DD: Septic arthritis / Osteoarthritis / Osteomyelitis
One diagnosis: septic arthritis / One diagnostic test: arthrocentesis (joint aspirate)
Management:
IV antibiotics, empiric therapy, (based on age and risk factors; oxacillin [2 g IV q4h for 4 weeks], or
vancomycin [if suspecting MRSA; 20 mg/kg IV q8h, for 8 wks], combined with ceftriaxone for gram
ve, if suspecting Gonococcal: ceftriaxone; IV for 2 wks then oral for 2 wks), adjust pending C&S
For small joints: needle aspiration, serial if necessary until sterile
For major joints such as knee, hip, or shoulder: urgent decompression and surgical drainage

OSCE-guide-III.doc Page 144 of 255


Physical Examination

Knee Joint
Vital signs, General
Inspection - Gait and stance: normal; no antalgic (painful) gait
- Bilateral joint exposure (quadriceps)
- SEADS (Swelling / Erythema / Atrophy / Deformity / Scars)
- No genu varum (bow legs) and no genu valgum (knock-knee) deformities
Palpation - Temperature: compare
- Extended knee: tenderness over patella /+/ Lateral movement of patella /+/
quadriceps muscle / quadriceps tendon / patellar ligament / Tibial tuberosity
/ popliteal fossa and popliteal artery
- Flexed knee: tibial plateau / bilateral joint lines /+/ Collateral ligaments /+/
Femoral condyles /+/ patellar crepitus
- Popliteal fossa /+/ Cuff muscles [slightly flexed knee]
Knee effusion:
- Fluid wave or bulge sign (or milking test): for small amount of effusion;
from below and med to upward and lat. Then immediately sweep hand
down the lateral aspect pushing the fluid back
- Fluid ballottement test: for moderate amount of effusion
- Patellar tap: for large amount of effusion
ROM Patient lying down:
- Flexion (130) and extension (180)
- Internal and external rotations: while knee is flexed 90, point your toes in &
out please
- Patellar movement: medial and lateral
- Patellar compression test: tight your thigh please  rough or painful
movement: patello-femoral syndrome or osteoarthritis
Power - Flexion and extension, while the knee is flexed 90
Special tests - Anterior drawer test  for anterior cruciate ligament tear
- Posterior drawer test  for posterior cruciate ligament tear
- Lachman test: hip / knee semi flexed (30)  ACL tear
- Check for the medial / lateral collateral ligaments (stability of knee); while
flexed at 30  no laxity nor pain
- McMurrays test (for medial and lateral meniscus tears)  feel for
crepitus / patient feels pain
- For medial: maximally flexed knees, externally rotated foot  extend while
applying varus force (from inside outwards)
- For lateral: maximally flexed knee, internally rotated foot  extend while
applying valgus force (from outside inwards)
To complete my exam, I would like to do
- Painful clicking  cruciate, meniscus ACL MCL
- Knee lock  torn meniscus - Knee giving way - Can not descend
- Instability  cruciate  - Inability to continue activity stairs

DD OsgoodSchlatter disease Chondromalacia patellae


Tenderness  Pain on tibial tuberosity ( by kneeling)  Pain on lateral movement of patella
Investigations X-ray (AP / LAT / skyline)
Treatment  Benign self-limited condition  Non-impact activities
 Continue activity as tolerated  NSAIDs
 NSAIDs  Physiotherapy
 Physiotherapy  Surgery for refractory cases

OSCE-guide-III.doc Page 145 of 255


Physical Examination

Obstetrics and Gynecology

OSCE-guide-III.doc Page 146 of 255


Physical Examination

History taking OB-GYN

Introduction
CC
Analysis of CC Os Cf D
COCA Blood

HPI Associated
symptoms
DD
M Menstrual
G Gynecological
O Obstetric
S Sexual
PMH
FH
SH

OB/GYN cases

History taking:
- Vaginal discharge
- Vaginal bleeding
- Amenorrhea
- Infertility

Counselling:
- OCPs
- HRT
- C-section (wants to have c-section or wants to have vag delivery after c-section)
- Abortion
- 22 years old pregnant  anti-natal counselling
- 39 years old found she is pregnant, counsel her
- 30 yrs old pregnant (36 wks), HTN/+++ ptn in urine  counsel for pre-eclampsia
- PAP smear; 16 years old wants to arrange for a PAP smear
- PAP smear: 38 year old had abnormal PAP smear

OSCE-guide-III.doc Page 147 of 255


Physical Examination

MGOS history questions:

Menstrual:
- When was your LMP? First day? Was your LMP similar to the previous ones?
- Are they regular or not? How often do you have periods?
- How long does it last? How many days?
- How about the amount? Is it large / small? How many pads/day? Any blood clots?
- Are your periods painful? [not painful  anovulatory (PCOS/infertility)]
- Any spotting / bleeding between periods?
- When was your first period? Was it regular? For how long it was not regular? Normal to be
irregular for up to 18-24 months.

Gynecological:
- Do you have history or were diagnosed with any gynecological disease (e.g. polyps)?
- Do you have history of pelvic surgery or instrumentation (e.g. D&C)?
- Do you use contraception? What method? Since when? When was the last time?
Screening:
- Have you ever had Pap smear before? When was the last time? Any reason (if long time)? What
was the result?
- (>40 yrs) have you had mammogram done before? When? (Is it painful doctor? Could be; we
need to apply pressure on the breast to get better image)
- (>65 yrs) have you had your bone mineral density (BMD) done? Any reason?

Obstetrical  GTPAL:
- Have you ever been pregnant before? Any abortions (termination)? Or miscarriages (spontaneous
abortion)?
- Number of babies you delivered? Any twins? Any children with congenital abnormalities?
- For each delivery: was it full term or pre-term? Vaginal or CS? Any complications like high blood
pressure / high blood sugar?
- Family history of: repeated abortions / CS / congenital anomalies / twins

Sexual history:
- With whom do you live?
- If (alone / with family): are you in any relationship? Are you sexually active? Have you ever been
sexually active?
- If with partner: how do you describe the relationship? Is it stable? Are you sexually active? Do
you practice safe sex, and by that I mean using condoms every time? For how long you have
been together? (> 6 months  stable). And before that, were you sexually active?
- When did you start sexual activity?
- How many partners have you had for the last 12 months? For the last month?
- What is your sexual preference? Men/ women/ both? What type of sexual activity?
- Have you screened or diagnosed before with STIs? HIV? Vaginal discharge?
- How about your partner? Any fever? Discharge? Burning sensation?
- Do you feel safe in this relation?

What if the male partner does not like condoms? Is it ok to consider it safe sex? Yes, provided
that:
- Scan the partner for STIs first
- Strict monogamy relation (no extra-marital affairs)
- Use alternative reliable contraception (e.g. OCPs)

OSCE-guide-III.doc Page 148 of 255


Physical Examination

History of pregnant lady third trimester

Are you doing regular ante-natal follow-up visits?

NO Yes

Social issue

Deal with the social issue

Last visit history / pre-eclampsia  When was your last f/u visit?
 What was your BP? Was there any headache?
 Was there leg swelling? Weight gain?
Make sure the mother is stable  Any abdominal pain? Cramps?
 Vaginal bleeding? Discharge?
 Any gush of water?
Make sure the baby is stable  Is your baby kicking like before? > 6 in 2 hrs
U/S  Have you done your U/S? How many times? When
was the last time?
 Number of babies?
 Location of the placenta?
 Amount of fluids?

N.B. to make sure the mother and baby are stable: ABCDE
 Activity of the baby
 Bleeding
 Contractions / pain
 Dripping / Discharge
 EDD (expected date of delivery)

OSCE-guide-III.doc Page 149 of 255


Physical Examination

Vaginal Discharge
Teenager / 5 minutes case
CC How can I help you?!
Analysis of CC Os Cf D
COCA Blood / color / fishy odour?
-
- Related to periods
- Related to sexual intercourse (bact vaginosis: discharge post-coitus)
M - LMP / regular / how often / similar to previous ones?
HPI AS - Any pain? With intercourse? Same system
DD - Itching? Redness? ? Candida
- Any blisters / warts / ulcers13?
- Inguinal swellings?
- Urine changes? Dysuria, frequency? Nearby systems
- Bowel movements changes? GIT symptoms
- Abdominal pain  OCD / PQRST /
- ? PID  Adnexal tenderness / fever
- Dissemination to liver (pain Rt upper abd)
- Constitutional symptoms DD
- Sore throat? Mouth ulcers? Red eyes?
- Joint swelling/pain? Skin rash? Reiters
G - IUD
- PAP smear!
- History of STI / PID?
O
S Complete sexual history for both partners
PMH - Any medications? Recent use of antibiotics
- Allergies
- DM
FH / SH - How do you support yourself?
- HEAD SS / SAD
Conclusion: STI because of risky sexual behaviour
 Physical examination including pelvic, speculum exam / PAP smear / swabs for C&S including those
for Chlamydia & Gonorrhea / saline slide microscopy / KOH / Whiff test
 DD: Gonorrhea, Chlamydia, Candidiasis (whitish), Bacterial vaginosis (thin gray, clue cells),
Trichomonas (frothy yellowish / greenish discharge, motile organism).
 Treatment:
o Gonorrhea: Ceftriaxone 250 mg IM single dose
o Chlamydia: Azithromycin 1g orally single dose
o Candidiasis: Miconazole 200 mg vag supp, 1 vag supp od qhs x 3 d
o Bacterial vaginosis: Metronidazole 500 mg PO bid x 7 d
If pregnant: Amoxicillin 500 mg PO tid x 7 d
o Trichomonas: Metronidazole 500 mg PO bid x 7 d
 Follow up with in 4 weeks
 Her partner(s) to be notified and to come for treatment, ask about sexual health (fever, discharge)
 Advice regarding safe sex (condoms, multiple partners, STIs)
 Chlamydia and Gonorrhea are reportable diseases
 HIV testing and other STIs screening if high risk sexual behaviour
 Advise regarding PAP smear regularly, vaccination against HPV

13
Blisters: HSV (Herpes Simplex Virus) / warts: HPV (Human Papilloma Virus) / ulcers: syphilis

OSCE-guide-III.doc Page 150 of 255


Physical Examination

Vaginal Bleeding Non-Pregnant / Not-Known Pregnant


Analysis of CC Timing: OS Cf D When did it start to be continuous?
COCA / fresh blood vs. clots  if large amounts  impact  anemia /
dehydration symptoms (pallor, SOB, dizziness, fainting, heart racing)
-
- Related to sexual intercourse (if yes: cervical*)
M - For the last few weeks, how do you distinguish between your regular
periods and the bleeding? Related to periods
- LMP / regular / how often / how much / similar to previous ones?
- First menstrual period? Regularity?
HPI AS - Any chance you are pregnant? How do you know for sure? Same
DD - Any nausea / vomiting / breast engorgement? Frequency? system
- Any pain? With intercourse?
- Itching? Redness?
- Any blisters / warts / ulcers?
- Grape like tissue (hydatiform mole)
- Urine changes? Urinary symptoms? Nearby
- Bowel movements changes? GIT symptoms systems
- Abdominal pain: OCD/PQRST (? PID / ectopic)
- Pelvic fullness / heaviness?
- Constitutional symptoms DD
- Bleeding disorders/ tendencies? Any bleeding elsewhere
(nose, gums, with stool, easy bruising)?
- Blood thinners? Aspirin?
- Thyroid problems? Symptoms?
G - Polyps, fibroids, endometriosis, gyn cancer?
- Hx pelvic surgeries? Instrumentations?
- Contraception history; OCPs / IUD / HRT / mammogram
- PAP smear! Was it normal?
O - Any previous pregnancies? Abortions? How many? Route
of delivery?
S - Number of partners / safe sex / when did you start activity?
- Hx of STIs
PMH - Breast cancer, mammogram (if > 40 years), HTN
FH - Gynecological cancer / Breast cancer
SH - How do you support yourself?
- SAD / HEAD SSS (if teenager)
Vaginal bleeding cases:
- Middle age / risky behaviour / old abnormal Pap smear  cervical cancer.
- A 48 years patient with vag bleeding and all symptoms will be negative  dysfunctional uterine
bleeding (DUB); intermittent / lose track of periods / no pain with periods.
- A 52 years pt / constipation / OCP/HRT / no pregnancies14  endometrial cancer / constitutional
- A 62 years pt with intermittent bleeding / small amount with secretions  atrophic vaginitis;
(menopausal symptoms / dyspareunia / itchy vulva)  Rule out cancer (endometrial biopsy) 
estrogen vaginal cream
- DD: Fibroid / Cervical polyp / hyper/hypo-thyroidism / Trauma / Coagulopathy / PCOS
Investigations: pregnancy test -HCG / progesterone challenge test / hysteroscope / PAP / U/S /
endometrial biopsy / TSH

14
Cervical cancer for prostitutes (risky behaviour) and endometrial cancer for nuns (no pregnancies)

OSCE-guide-III.doc Page 151 of 255


Physical Examination

Vaginal Bleeding Pregnant / Ante-Partum Hemorrhage


 Patient usually sleeping in a left lateral position with oxygen, very anxious, crying
 Reassure her & ask her to bear with you for a while to get to the bottom of her problem
 Ask about her feelings & empathize (impact on her and her partner)
Analysis of CC Timing: OS Cf D first time or happened before
COCA / fresh blood vs. clots  if large amounts  impact  anemia /
dehydration symptoms (pallor, SOB, dizziness, fainting, heart racing)

M - LMP / regular / how often / how much / similar to previous ones?
HPI AS - How many weeks? date based on LMP Same
DD - How did you know that you are pregnant? Was it planned? system
- Did you have regular antenatal care? F/U visits? U/S?
- What is the result of U/S?
- Any pain / discomfort? (OCD / PQRST)? Contractions
- Water gush / grape like tissue (hydatiform mole)
- Fetal movement
- Urine changes? Urinary symptoms? Nearby
- Bowel movements changes? GIT symptoms systems
- Abdominal pain: OCD/PQRST (? PID / ectopic)
- Pelvic fullness / heaviness?
- Constitutional symptoms DD
- Bleeding disorders/ tendencies? Any bleeding elsewhere?
- Headache, hand or feet swelling
- Trauma, sexual intercourse
G - Polyps, fibroids, endometriosis, gyn cancer?
- Hx pelvic surgeries? Instrumentations?
- PAP smear! Was it normal?
O - GTPAL
- Any previous pregnancies? Abortions? How many? How
many weeks? Route of delivery?
S - Hx of STIs
PMH - Breast cancer, mammogram
- HTN, DM, kidney disease, blood gp & Rh
- Medication / allergies / hospitalization / surgeries / blood transfusion.
FH - Gynecological cancer / abortions
SH - How do you support yourself?
- SAD / HEAD SSS (if teenager)

Threatened abortion Separation of part of the placenta


Will do physical and obstetric examination. Vaginal exam only after U/S excludes placenta previa
- 50/50% chance to keep or loose the baby - Admit to the hospital
- Investigations: continuous vitals monitoring / CBC / INR / PTT / - Arrange for continuous fetal heart monitoring, U/S
fibrinogen / Rh status / blood grouping and cross matching / US / and biophysical profile
fetal monitoring - Management: O2, IV fluid, LLP, if fetus is still in
- Management: distress, arrange for C/S
- If she is improving / the fetus is not distressed: she will go home,
Placenta previa Placenta abruption
resume normal activity, come back if more blood or more pain
- Otherwise: bed rest / steroids / fetal monitoring / Rhogam / platelets - Painless - Painful (contractions)
Incomplete abortion: 3 findings on vag exam would confirm the diagnosis: (1) Cervix dilated, (2)
Ruptured membranes, (3) Product of conception passed

OSCE-guide-III.doc Page 152 of 255


Physical Examination

Abnormal Uterine Bleeding (AUB)

 Dysfunctional Uterine Bleeding (DUB):


Abnormal bleeding not attributable to organic (anatomic / systemic) disease. DUB is a
diagnosis of exclusion. Anovulatory AUB often used synonymously with DUB.
Causes:
o progesterone: luteal phase defect (estrogen-dependent DUB)
o PCOS
o Endocrinal ( TSH / prolactin)
o Stress, weight loss, exercise
o Liver and kidney disease
 Investigations for AUB:
Beta-hCG
o CBC, serum ferritin
o Coagulation profile (esp. adolescent): rule out von Willebrand's disease
TSH, free T4
Prolactin if amenorrhea
o FSH, LH
o Day 21 (luteal phase) progesterone to confirm ovulation
o Serum androgens (especially free testosterone)
Pelvic U/S: detect polyps, fibroids; measure endometrial thickness (postmenopausal)
Sonohysterogram (SHG): very sensitive for intrauterine pathology (polyps, submucous
fibroids}
Hysterosalpingography (HSG)
o Pap test
o Endometrial biopsy: women > 40 years are at higher risk of endometrial cancer
 Must do endometrial biopsy in all women presenting with postmenopausal bleeding
to exclude endometrial cancer
o D&C: not for treatment; diagnosis only (usually with hysteroscopy)

OSCE-guide-III.doc Page 153 of 255


Physical Examination

 Treatment of AUB:
Treat underlying disorders / if anatomic lesions and systemic disease have been ruled out,
consider dysfunctional uterine bleeding (DUB)
Medical:
o Mild DUB
 NSAIDs
 Anti-fibrinolytic (e.g. Cyklokapron) at time of menses
 Combined OCP
 Progestins (Provera) on first 10-14 days of each month if oligomenorrheic
 Mirena IUD
 Danazol (pseudo-menopause)
o Acute, severe DUB
 Replace fluid losses, consider admission
 Medical treatment:
(a) estrogen (Premarin) 25 mg IV q4h x 24h with Gravol 50 mg IV/PO q4h or
(b) Ovral15 1 tab PO q4h X 24h with Gravol 50 mg IV /PO q4h
Taper Ovral: 1 tab tid X 2d  bid X 2d  OD
 After (a) or (b), maintain patient on monophasic OCP for next several months or
consider alternative medical treatment
o Clomiphene citrate: consider in patients who are anovulatory and who wish to get
pregnant
Surgical:
o Endometrial ablation; consider pre-treatment with danazol or GnRH agonists
 If finished childbearing
 Repeat procedure may be required if symptom recurrence
o Hysterectomy: definitive treatment

Post-coital bleeding / middle age:


STIs
Cervical (Cervicitis / Cervical polyp / Cervical cancer / Ectropion; if OCPs)
Bleeding tendencies
Trauma
Management:
Vaginal and cervical swabs; C&S and gram stain
Polyp: Sonohysterogram
Biopsy
If ectropion: stop OCPs, cautery (silver nitrate)

Cancer uterus:
Risk factors:
Early menarche
Nulliparity
Weight gain
HRT / estrogen therapy
Unopposed estrogen
Management:
Endometrial biopsy; if positive
o Total abdominal hysterectomy and bilateral salpingo-oophorectomy
o Adjuvant chemotherapy

15
Ovral is progestin (levonorgestrel) and estrogen (ethinyl estradiol) combination OCP

OSCE-guide-III.doc Page 154 of 255


Physical Examination

Amenorrhea
CC  Did not have periods for 6 months?!
 Did you seek medical attention? Any recent changes?
Analysis of CC  During these 6 months; any irregular bleeding? Spotting?
M  When was your first period? What age? Was it regular? For how long it was
regular / not regular? How often? How much? LMP?
 When it was regular; was it painful? (painless  anovulatory)
 Did you use any contraception? When did you stop? Why?
HPI AS  Any chance you are pregnant? How do you know for sure?
DD  Any nausea / vomiting? Breast engorgement? Frequency?
 For how long have you been trying to get pregnant?
 Any previous pregnancies? Abortions?
 Constitutional symptoms?
 Are you under stress? Hypothalamus
 Excessive exercise?
 Any concerns about your weight? (anorexia)
 Any headache? Vomiting in morning? Visual changes? Pituitary
Difficulty seeing to sides? Milk secretions from breast?
 History of thyroid disease? Heat/ cold intolerance?
Bowel movements? Moist/ dry skin?
 Do you have excessive hair growth? Acne? Did you Ovarian
notice any weight changes? Hx of DM / thirsty /
frequency? Fm Hx of PCOS?
 Hx of chemotherapy? Radiotherapy? Hot flushes?
Vaginal dryness? Soreness?
 Any change in your voice? Muscle bulk?
G  Any repeated surgical procedures? D&C? Uterine
 Pelvic surgeries? Instrumentations?
 PAP smear!
O  Any previous pregnancies? Abortions?
S  Hx of STIs
PMH - Any medical conditions? Psychiatric illness?
- Any medications? Recent use of antibiotics
FH - Family hx of PCOS / infertility?
SH - How do you support yourself?
- SAD
 Investigations: -HCG / progesterone challenge test / hormonal assay (estrogen / progesterone / FSH /
LH / prolactin / thyroid-TSH / serum testosterone; total and free) / US / CBC
 Ovarian causes of amenorrhea: PCOS /+/ Premature ovarian failure /+/ Androgen-producing tumours
 PCOS (poly-cystic ovarian syndrome):
History: previous pregnancy / contraception hx
To diagnose PCOS: must have 2 of 3 criteria: (1) oligomenorrhea / irregular menses for 6
months, (2) hyper-androgenism (hirsutism or blood level), (3) PCOS by US
Investigations: -HCG / US / High LH:FSH ratio > 2:1 / Fasting blood sugar
Treatment:
o Lifestyle modification ( BMI, exercise) to peripheral estrone formation
o Metformin 500 mg PO tid
o Clomiphene citrate; if she wants to become pregnant
o Tranexamic add (Cyklokapron); for menorrhagia only
o OCPs; if she does not want to become pregnant
 Dysmenorrhea:
DD: PID, fibroid, endometriosis
Investigations: U/S to exclude other conditions
Treatment: NSAIDs (ibuprofen 400 mg tid), OCPs.

OSCE-guide-III.doc Page 155 of 255


Physical Examination

Infertility

Transitional statement before going in details with the history:


In order for a couple to achieve pregnancy, both partners should be capable of having children
and relatively healthy. For that reason, I am going to ask some questions about your health and
your partner health; some of these questions are personal, but it is important to ask. And I would
like to assure you that all the conversation is strictly confidential and I will not release any
information without your permission!
Introduction
CC
Analysis of CC For how long have you been trying to conceive?
HPI DD - Anovulation (irregular cycles / painless / no pre-menstrual syndrome)
- PCOS: Do you have excessive hair growth? Acne? Did you notice any
weight changes? Hx of DM / thirsty / frequency? Fm Hx of PCOS?
- Tubal occlusion: surgeries / STIs and PID / IUD
- Endometriosis: dysuria / dyspareunia / dyschezia / back pain
- History of chemotherapy and radiotherapy
Gynecology - Do you have hx or were diagnosed with any gyn disease (e.g. polyps)?
Hx - Did you use contraception? What? Since when? When was the last time?
- Have you ever had Pap smear before? When was the last time? Any
reason (if long time)? What was the result?
Obstetric Hx - Have you ever been pregnant before? Any abortions (termination)? Or
miscarriages (spontaneous abortion)?
- Family history of: repeated abortions / CS / congenital anomalies / twins
Coital history - For how long are you in this relation? For how long have you been
trying to achieve pregnancy?
- How frequent? Regularity?
- Are you aware of sexual cycle and ovulation (both you and your partner)
Partner - Was he sexually active before? Did he father kids from another partner
history before?
- Did he receive any chemo or radio therapy?
- Does he complain of any testicular problem; varicose veins,
inflammation?
- Does he complain of any penile discharge?
- Did he go through any investigations; e.g. semen analysis?
PMH - Allergies
- DM
FH Infertility
SH

Investigations:
Semen analysis
Ovulation documentation (mid-luteal phase progesterone; d 21-22 / US)
Tubal patency (HSG / laparoscopy)

When to start investigations:


If the woman is < 35 years  after 12 months of trying to conceive
If the woman is 35-40 yrs  after 6 months of trying to conceive
If the woman is > 40 years  start investigations after 1 months of trying to conceive

OSCE-guide-III.doc Page 156 of 255


Physical Examination

Counselling pre-eclampsia
36 weeks pregnant lady comes for f/u visit, BP 160/110, +++ protein in urine, Manage.
Introduction Like the B12 results case
I will discuss results with you
Ethical challenge: travel permission
History Last visit history / pre-eclampsia
Make sure the mother and baby are stable
U/S
Obstetric history / Gynecological history
PMH / Social history
Counselling Explain what is pre-eclampsia
Serious concerns with pre-eclampsia
Management Hospitalize
If insisting to leave  sign a LAMA
Introduction
- Good afternoon Ms I am Dr I understand that your blood pressure was measured and urine
test was done, I have the results with me and I will discuss it with you. However, because this is
my first time to see you, I need to ask you some questions, to get a better understanding of your
health condition, is that ok with you?
- Is this you first time to have these checks during your pregnancy?
- Are you under regular follow-up?
o Yes  proceed to history
o No  any reason? My husband had a car accident! I am sorry to hear that; was he hurt?
Was anyone else hurt? When was that? It must be difficult, how did this affect your life?
Ethical challenge: travel permission
o Actually I am here to get a note.
- What type of notes?
o Travel note, I really need to travel.
- It looks like it is an important trip for you; usually pregnant ladies do not travel during this time of
pregnancy!
o It is a business trip that would save our financials.
- I see it is important for you, however, before we proceed, let me check your health condition first,
and I will start by asking you some questions:

History
Last visit history / pre-eclampsia
- When was your last f/u visit?
- What was your BP? Was there any headache?
- Was there leg swelling? Weight gain? Did they do urine test?
- How about before being pregnant? Any hx of high blood pressure?
Make sure the mother and baby are stable: ABCDE
- Activity of the baby, is your baby kicking like before?
- Bleeding
- Contractions / pain
- Dripping / Discharge
- EDD (expected date of delivery)
U/S
- Have you done your U/S? How many times? When was the last time?
- Number of babies?
- Location of the placenta?
- Amount of fluids?
Obstetric history: any pregnancy before / any similar conditions? Gynecological history
PMH: high blood pressure Social history: SAD / support / home environment

OSCE-guide-III.doc Page 157 of 255


Physical Examination

Counselling
Explain what is pre-eclampsia
- Your blood pressure is 160/110, which is high, and the urine test shows protein in large amount
(+++) which is not normal, the most likely diagnosis is a medical condition called pre-eclampsia
OR pregnancy-induced hypertension.
- I would like to ask more questions to see how it affected you!
o My dad had HTN, and lived with it, I am ok.
- These are different conditions; your dad had HTN, but you have pregnancy-induced HTN,
which is a serious condition, with very serious and may be fatal consequences.
o Have you had hx of headache?  OCD / PQRST (not detailed)
o Nausea / vomiting
o Change in your vision? Flashing lights? Flying objects?
o Any abdominal pain in your upper right part of your abdomen?
o Any bruises? Yellowish discoloration / itching / dark urine / pale stools?
o Any chest pain / heart racing / SOB?
o Any weakness / numbness?
o Any swelling in your body / face/eyes? Did you feel your shoes tight?
o Did you gain weight?
o Any changes in the urine? Frothy? Burning sensation?
- Based on all this, the most likely explanation for your increased is pre-eclampsia; and this is a
very serious condition, we need to admit you to the hospital to monitor you. Then, the obstetrician
will assess you and may consider delivering the baby now.
o But doctor, I need to travel, just 2 days and I will come back.
- I understand your concern about traveling, but we have a serious situation here.
- We do not know exactly why patients have pre-eclampsia. We believe it is imbalance of
hormones, or it might be related to placenta, however the only treatment is delivering the baby.
Serious concerns with pre-eclampsia
- What happens is that there is a narrowing of blood vessels, this leads to the amount of blood
reaching the baby, subsequently the amount of oxygen and nutrients. On the long term this will
lead to some injury and even damage to the baby AND the mother.
o This includes your heart and blood vessels, that is why you have BP,
o This includes your kidney, that is why you have +++ protein in urine,
o This includes your liver, that is why you may have abdominal pain,
o This includes your brain, that is why you have headache, visual changes,
o This includes your baby, that is why he is not kicking like before
This is not because of your pregnancy; all of these are due to this condition.
- The concerns we have is that we can not predict the outcome, without the proper medical care,
patients having pre-eclampsia will end up going to the next stage which is eclampsia; do you
any idea what is e0clampsia?
- A condition in which, the patient will start to seize, lose conscious, will not be able to breath and
turn blue. The only resolution for this is delivering the baby.
- Imagine that I give you the note, and they allow you to take the trip, 2 hours later while you are in
the plane, you start to fall down and seize. What will happen? Nobody will be able to help you.
- By this you endanger your life and your babys life.
Management
- What we need now is to admit you to the hospital and arrange for obstetrical assessment.
- If insisting to leave  sign a LAMA (leaving against medical advice)
- Suggest solution for her business travel, like giving a doctor note that she needs to be hospitalized.
Treatment Plan
- Assess severity including good history and physical exam focusing on heart, lungs, reflexes, fetus,
urine analysis and BW (important CBC, liver function tests, Uric Acid)
- If all above are stable, consider daily check, urine dips and fetal kick counts as outpatient. If any
of above unstable may need to hospitalize as inpatient for close monitoring
- Measure L/S ratio of the baby, give corticosteroids for lung maturation
- MgSO4 and delivery
- Blood pressure controlled often with labetalol, Ca channel blockers

OSCE-guide-III.doc Page 158 of 255


Physical Examination

Caesarean Section Counselling wants to have CS

Young 18-20 years old pregnant lady would like to have CS, counsel her.
Introduction
Any reason you want to have CS? Social issue
History Last visit history / pre-eclampsia
Make sure the mother and baby are stable
U/S
Obstetric history / Gynecological history
PMH / Social history
Counselling Address patient concerns
Why not caesarean section?
Management Refer to obstetrician
Spend some time to think / stabilize

Introduction
Any reason you want to have CS?
- I understand that you are here to discuss the possibility of CS; we will discuss this in details, but
before that I would like to ask you is there any reason you would like to have CS?
o I do not want to have this severe pain!
- How do you know it is painful?
o I had previous abortion OR
o I attended my sister delivery and it was very painful experience
- When was that? Did you attend?
o Congratulations! How is your sister doing? How is the baby?
o I understand that you saw her in pain, but people differ! And within few minutes I will be
explaining different options to control labour pains!
History
- Let me ask you some questions to assess the condition first!
o How do you feel? How is your mood?
o How about this pregnancy, was it planned? How do feel about your pregnancy? How is
the feeling of your partner?
o Are you under regular follow-up? NO! Any reason?
There may be social issue here.
 Empathy: it looks like you are doing through difficult times! How are you coping?
 Offer social support: being pregnant lady without support, you have priority and
there are a lot support and resources in the community. I will make sure to connect
you with social worker who will help you with proper support (housing / financially /
for both of you and the baby)

Last visit history / pre-eclampsia


- When was your last f/u visit?
- What was your BP? Was there any headache?
- Was there leg swelling? Weight gain? Did they do urine test?
- How about before being pregnant? Any hx of high blood pressure?
- Did you have your blood sugar checked? How about before pregnancy?

Make sure the mother and baby are stable: ABCDE


- Activity of the baby, is your baby kicking like before?
- Bleeding
- Contractions / pain
- Dripping / Discharge
- EDD (expected date of delivery)

OSCE-guide-III.doc Page 159 of 255


Physical Examination

U/S
- Have you done your U/S? How many times? When was the last time?
- Number of babies?
- Position of the placenta?
- Amount of fluids?

Obstetric history: GTPAL


- Any pregnancy before? Any abortions or miscarriages?
- What were the circumstances? How many weeks?
- How did you feel about it? How did you cope with that?
Gynecological history: Fibroid, Genital herpes

PMH: Medications / allergy / blood transfusion


Social history: SAD / support / home environment

Counselling

Address patient concerns


- I know that you are here to talk about CS. But first let me explain some facts about delivery. The
natural route for delivery is the vaginal delivery, and if there is no real indication for CS, we
would prefer to go for vaginal delivery.
- However, I appreciate you concerns, if your concern is the pain there is a lot of options to control
it.
o We can start by learning some relaxation techniques
o And then on the delivery day, when you go there, there are a lot of support groups; one of
them is called doolas; they attend with you and they provide a lot of emotional support
o Finally, when you start your labour, we have good measures; and I mean what we call
epidural anesthesia; this is very effective and safe. Where the anesthesiologist puts a
needle into your back and injects a freezing substance that helps you to go through the
delivery without pain (this is like the dentist freezes your mouth before doing painful
procedures). It might however cause headache, bleeding, and less likely infection.

Why not caesarean section


- I would like to ask you; what is your understanding of CS?
- It is commonly used obstetrical intervention, used when there is a problem or contraindication for
vaginal delivery and if there is an emergency situation that necessitates immediate delivery; and in
these cases it is life saving; for both the mother and the baby!
- However, it is a major surgery, has the risks of bleeding, higher risk of infection, DVT, you stay
longer in the hospital, and it will leave scar in your abdomen.

Management
- After all, I am not the person who makes the decision; this should be decided by the obstetrician.
- I am going to refer you to the obstetrician; who will perform further and detailed assessment then
discuss the results with you.
- Meanwhile, I would recommend you spend some time to think about what I told you, try to
stabilize yourself emotionally.

- I will give you some brochures and web sites so that you can read more about that.
- I will connect you with the social worker.
- And if at any time you have any questions or concerns, you can come to see me.

OSCE-guide-III.doc Page 160 of 255


Physical Examination

Caesarean Section Counselling does not want to have CS


Middle aged pregnant lady (36 weeks) is here to have her file. Three years ago, she had urgent CS
for cord prolapse. Now she would like to deliver at home with the midwife.
Introduction
Any reason you want to have your file? Concern
Deal with the patient concern
What was the type of your CS?
History Last visit history / pre-eclampsia
Make sure the mother and baby are stable
U/S
Obstetric history / Gynecological history
PMH / Social history
Counselling What is CS? The two types of CS
Risks of vaginal delivery post CS
Management Prepare a copy of the file
Speak with your midwife
Introduction
Any reason you want to have your file?
- I understand that you are here to have a copy of your file for the urgent CS you had 3 years ago.
This is your right, and I will ask someone to prepare a copy for you.
- But first I would like to ask you some question, is it ok with you!
- Is there any reason you would like to have your file?
o Yes, I would like to have delivery at home this time. A lot of my friends did it at home
with the midwife and they say it is much easier and relax.
- Are you seeing obstetrician? Are you doing any regular follow-up visits? Any reason for that?
o No, I am going to follow up with the midwife.
o I did not like last time when they did CS at the hospital!
- What happened last time?
o They told me the baby had cord prolapse!
- Were there any consequences?
o How is the baby doing? How old is he? What can he do? Walk? Talk?
o How about you? Any complications? Infections? Scars?
Deal with the patient concern:
- What is your understanding of cord prolapse?
- It is a condition where the umbilical cord goes into the birth canal before the head, and then gets
stuck and squeezed by the head.
- This is a very serious condition. The cord delivers blood and nutrients to the baby. If blocked for
long time, the baby will suffer from brain damage.
- That is why they had to do urgent CS, which was a life saving procedure for the baby, and it had
to be done immediately.
- Do you know what the type of your CS was? Classical!
History
- I understand your point of view, but first let me ask you some questions about your health and
your pregnancy!
Last visit history / pre-eclampsia
- When was your last f/u visit?
- What was your BP? Was there any headache?
- Was there leg swelling? Weight gain? Did they do urine test?
- How about before being pregnant? Any hx of high blood pressure?

Make sure the mother and baby are stable: ABCDE


- Activity of the baby, is your baby kicking like before?

OSCE-guide-III.doc Page 161 of 255


Physical Examination

- Bleeding
- Contractions / pain
- Dripping / Discharge
- EDD (expected date of delivery)
U/S
- Have you done your U/S? How many times? When was the last time?
- Number of babies?
- Location of the placenta?
- Amount of fluids?
Obstetric history: GTPAL
- Other than the pregnancy that you had CS 3 years ago; any pregnancy before? Any abortions or
miscarriages?
- What were the circumstances? How many weeks?
- How did you feel about it? How did you cope with that?
Gynecological history
PMH: Medications / allergy / blood transfusion
Social history: SAD / support / home environment

Counselling
What is CS? The two types of CS
- I would like to ask you; what is your understanding of CS?
- It is commonly used obstetrical intervention, used when there is a problem or contraindication for
vaginal delivery and if there is an emergency situation that necessitates immediate delivery; and in
these cases it is life saving; for both the mother and the baby!
- There are two types of CS:
o The transverse (done at the lower segment of uterus); it is the most common type; its
advantages include: smaller scar and better healing.
o The classical or vertical type; it is done less common; as we cut through the muscle fibers
of the uterus it produces weaker scar; but it is indicated and actually needed in urgent
case, like yours. As it allows quick access and fast delivery, because in some cases (like
cord prolapse) we can not afford even few minutes more.
Risks of vaginal delivery post CS
- Due to the scar formed after the CS procedure; it is always recommended to deliver by CS, to
avoid the tearing pressure of the uterine contractions during vaginal delivery.
- If you decide to go for vaginal delivery, my concern is that the scar might undergo severe tearing
pressure and might rupture, which will lead to massive bleeding. This is an obstetrical emergency
that necessitates immediate intervention. Because you may end up losing your life and/or losing
your baby.
- I do not want to scare you, but the risks of having uterine rupture after classical CS is 12%, of
which 10% of cases end up losing their lives.
- For that reason: once classical CS, it is always CS.
In case of counselling transverse CS:
- Risks of having uterine rupture after transverse CS is 1%.
- Even though, if you want to try vaginal delivery, we can not take the risk to try this at home, we
can try this in the hospital, so that just in case any emergency might happen, we can intervene in
the proper time.

Management
- I will ask someone to prepare a copy of your file
- Speak with your midwife:
- I am sure that your midwife is highly trained and qualified, and we share the same guidelines. I
would recommend that you take your file and speak with your midwife, and I am sure she will
explain the situation to you.
- I will give you some brochures and web sites so that you can read more about that.
- And if at any time you have any questions or concerns, you can come to see me.

OSCE-guide-III.doc Page 162 of 255


Physical Examination

OCPs / Contraception Counselling


XX is a 16 years old girl. She would like to get information about OCPs. For the next 10 minutes
talk to her

Introduction
Concerns Do you have any concerns?
Why do you need / think about contraception at this point?
Have you used contraception before? What is your previous
experience? Why did you stop it?
History Exclude Any chance you are pregnant? How do you know for sure?
pregnancy Any nausea / vomiting? Breast engorgement? Frequency?
M Painful periods? / Irregular? / Heavy bleeding?
G Last PAP / any abnormal PAP
Previous D&C
O GTPAL / IUD is not recommended in nullipara
S DETAILED
If risky behaviour: OCPs will not protect against STIs
Risky behaviour or previous STI/PID: NO IUD

Available Definition: birth control is an umbrella term for several techniques and methods used to prevent
methods fertilization
Hormonal (OCPs / implants / injections)
IUD (contragestion: prevents the implant)
Barrier methods (condoms / diaphragms) spermicidal
Behavioural (fertility awareness/timing) / coitus interruptus
Post-coital contraception
Sterilization (male / female)
OCPs IUD
Mechanism of OCPs are hormones: estrogen and Mechanically prevents the implantation of
action progesterone the fertilized ovum
OCPs prevent ovulation, increase thickness Available forms: Copper / hormone-
of cervical secretion coated
21 tab + 7 sugar pills  28 days
Contraindications HTN / CAD / Cerebro-vascular disease / Structural uterine anomalies
DVT History of ectopic pregnancy
Breast or gynecological cancer (past
personal and family hx) Undiagnosed vaginal Bleeding
Undiagnosed vaginal Bleeding History of PID(s)
Active liver disease Risky behaviour
Smoker > 35 yrs
Migraine
Benefits Regulate periods Longevity
Independence to coitus or compliance
ABCD:
Improves anemia ( bleeding)
benign breast lesions
ovarian cysts and cancer
risk of uterine cancer
dysmenorrhea

OSCE-guide-III.doc Page 163 of 255


Physical Examination

Side effects Does not prevent STIs Heavy periods


risk of DVT / CAD If hormonal coated: prog side effects:
TRANSIENT: headache / wt gain / mastalgia
Breast tenderness
Weight gain
Headache
Nausea / vomiting
Failure rate 0.1% 5% 0.5% 2%
OCPs IUD
Plan Will do physical and pelvic examination, and document blood pressure, pap smear
Brochures and information
Follow up within 1 month
How we will start:
First day of next menstrual cycle
Put package in obvious place that you see
frequently
Take the pill in the same approximate time
every day
Use another method of contraception for the
first TWO months
If you miss a pill: see below

NOTES

 N.B.: if woman with risky behaviour:


- Counsel for safe sex
- OCPs do not protect against STIs

 Failure rates are very high with behavioural methods:


- Because it depends largely on the knowledge, experience of user and the usage technique;
perfect-use versus typical-use failure rates.

 What happens if I missed 1 or 2 pills (OCPs)?


- 1 missed pill  take 2 pills the next day
- 2 missed pills  take 2 pills a day, for the next 2 days
- If missed more than 2 consecutive pills: use a backup method of contraception
simultaneous to finishing up packet of pills (2 pills a day) until next menses

 Woman comes to request sterilization (tubal ligation)


1- Discuss various other alternatives, make sure she knows everything about all of them
(OCPs, IUDs, diaphragm, condoms, vasectomy)
2- Counsel the patient on the permanent nature of the procedure, the operative risks, and
the chance of failure (1 in 200)
Counsel about the risks of regret for the decision (young age, recent emotional trauma,
family coercion)
3- Bring the patient back after 1 month for the bilateral tubal ligation procedure,
preferably immediately after menstruation to decrease possibility of pregnancy

OSCE-guide-III.doc Page 164 of 255


Physical Examination

Advantages Contraindications Side effects Available Failure rate


OCPs - Regulate periods - Pregnancy - Does not prevent STIs Combined pills (E+P): 0.1% 5%
- Un-dx vag bleeding - risk of DVT / CAD low dose estrogen (20,
ABCD: - Cerebro-vascular dis / CAD 35, 50)
- Improves anemia ( - Active liver disease TRANSIENT:
bleeding) - Hormone-dep cancer - Breast tenderness
- benign breast lesions - Smoker > 35 yrs - Weight gain
- ovarian cysts and cancer - Hx of DVT / PE - Headache
- risk of uterine cancer - Migraine - Nausea / vomiting
- dysmenorrhea
IUD - Longevity - Pregnancy - Heavy periods - Copper 0.6% 2%
- Independence to coitus or - Un-dx vag bleeding - If hormonal coated: prog - Hormone coated
compliance - Structural uterine anomalies side effects: headache /
- Hx of PID(s) wt gain / mastalgia
- Risky behaviour
- Hx of ectopic pregnancy

IM - Dysmenorrhea - NO absolute contraindications - Irregular bleeding Every 3 months < 1%


Injection - Effective - Amenorrhea after 1 year
- Independence - Delayed post-use
Depo- fertility
provera
SC implants - Longevity - Un-dx vag bleeding - Irregular bleeding Every 5 years < 1%
- Effective - Acute liver disease - Headaches
Nor-plant - Independence to coitus or - Thrombophlebitis - Mood changes
compliance - Thromboembolic disease

Condoms 14%
Diaphragm 20%
Behavioural - High failure rates up to 25%
- To decrease the failure rate, can combine 2 methods

OSCE-guide-III.doc Page 165 of 255


OB-GYN

HRT counselling

Introduction / overview
History General  How do you describe your general health?
 Support
M DETAILED
G D&C / OCPs / PAP / mammogram / BMD
O GTPAL
S Dyspareunia
Menopausal  Irregular menstrual cycles
symptoms  Vasomotor symptoms: sweating / hot flashes (hot flushes) /
palpitations
 Uro-genital symptoms: vaginal dryness / soreness /
superficial dyspareunia / urinary frequency and urgency
 Neurologic symptoms: mood changes / sleep disturbance /
depression / anxiety
Risk factors for Breast Cancer Uterine Cancer Ovarian Cancer
CANCER  Early menarche  Obese
 Late menopause  Diabetic
 OCPs  Nullipara
 No breast feeding  PCOS
 Age
 Past medical history of cancer or biopsy
 Family history
Risk factors for  Screen the first major risk factors for osteoporosis (see
Osteoporosis osteoporosis counselling case)
Counselling What is your understanding about HRT?
Common forms / products  Local preparations: creams / pessaries / rings
of therapy  Systemic formulations: oral drugs / trans-dermal patches and
gels / implants

- Estrogen alone
- Combined estrogen and progestogen
- Selective estrogen receptor modulator (SERM)
Indications for HT  Menopausal symptoms For SHORT term only, 1 2 years
 Osteoporosis
Contraindications to HT
Pre-treatment evaluation Hx / PE / baseline investigations
Adverse effects and risks
Any concerns? Media spoke that HRT increases incidence of stroke, heart attacks
and breast cancer, this was done by the (Women's Health
Initiative), on the other hand, smoking, obesity, cholesterol
increases the risk of these dis much more than HRT. In your case,
you do not have the risk factors for cancer, and it will be
beneficial for your hot flashes, vaginal dryness, and will protect
you against osteoporosis In medicine we always weight risk /
benefits

OSCE-guide-III.doc Page 166 of 255


OB-GYN

Introduction / overview:
- The reproductive years of a womans life are regulated by production of the hormones
estrogen and progesterone by the ovaries. Estrogen regulates a woman's monthly
menstrual cycle and secondary sexual characteristics (e.g. breast development and
function). In addition, it prepares the body for fertilization and reproduction.
Progesterone concentrations rise in a cyclical fashion to prepare the uterus for possible
pregnancy and to prepare the breasts for lactation.
- Toward the end of her reproductive years when a woman reaches menopause, circulating
levels of estrogen and progesterone decrease because of reduced synthesis in the ovary,
which may lead to several symptoms, the severity of which can vary widely.
- Hormone therapy (HT) involves the administration of synthetic estrogen and
progestogen. HT is designed to replace a woman's depleting hormone levels and thus
alleviate her symptoms of menopause. However, HT has been linked to various risks, and
debate regarding its risk-benefit ratio continues

Contra-indications of HRT:
No absolute contraindications of hormone therapy have been established. However, HT is
relatively contraindicated in certain clinical situations (similar to OCPs):
- Breast and/or endometrial cancer
- Undiagnosed vaginal bleeding
- Acute liver disease
- Thromboembolic disorders / DVT
- Endometriosis / Fibroids
- Diabetes, HTN, Heart disease

Required baseline investigations


- CBC
- Urinalysis
- Blood sugar levels
- Fasting lipid profile
- Electrocardiography
- Pap test
- Ultrasonography to measure endometrial thickness and ovarian volume
- Mammography

Possible adverse effects are as follows (similar to OCPs):


- Nausea / bloating
- Fluid retention
- Weight gain (equivocal finding)
- Mood swings (associated with use of relatively androgenic progestogens)
- Breakthrough bleeding
- Breast tenderness
- HT may slightly increase the risk for breast cancer
- There is association between HT and uterine hyperplasia and cancer
- There is increased risk of thromboembolism with HT

OSCE-guide-III.doc Page 167 of 255


OB-GYN

Needle Stick Counselling HIV

You are about to see Mrs 33 years old female nurse, upset because she had just had needle
stick after she gave an IV injection to a patient. Counsel her.

Introduction
Concerns HIV infection / fatal disease / will impact her family
History - Can you tell me what happened?
- Complete immunization record, including tetanus and hepatitis B
- Previous occupational exposure to body fluids
- Intravenous drug abuse
- Sexual history
Inform the - What is HIV? Major pathogens of concern!
patient about First of all; let me tell you the transmission rates: (no accurate studies)
HIV - Risk of blood transmission is: 0.3% for percutaneous exposure
- Risk of female to male transmission is: 0.03%
- Are you pregnant? Risk of intrauterine tx is: 3% with treatment and 30%
without treatment
- Advancement of HIV treatment
Address pregnancy concerns:
- Patient should receive ttt (not teratogenic)
- HIV positive mothers should not breastfeed their babies
Plan - I will speak with the patient, explain the whole situation and ask him to
consent for HIV status
- If he agrees; we will know possibility of tx to you. If he is HIV negative,
NO post-exposure prophylaxis is needed
- If he refuses or if he is HIV +ve; we will have to assess what is called
exposure code and match it with HIV status code; to simplify this,
guidelines state we should assume you were exposed and give
prophylaxis treatment: 4 weeks of 2 anti-virals (the basic regimen)
- Hepatitis B vaccination 3 doses + immunoglobulins (immediately)
Workup - Blood tests for the patient if possible and for the exposed
- I am going to speak with the patient now, and I will come back to you
with his decision.
- Any other questions or concerns?

What is HIV?
- Human immunodeficiency virus (HIV) is a blood-borne, sexually transmissible virus.
The virus is typically transmitted via sexual intercourse, shared intravenous drug
paraphernalia, and mother-to-child transmission (MTCT), which can occur during the
birth process or during breastfeeding.
- The major pathogens of concern in occupational body fluid exposure are HIV, hepatitis
A, hepatitis B, hepatitis C, and hepatitis D. These pathogens are viruses that require
percutaneous or mucosal introduction for infectivity. The major target organs are the
immune system (HIV) and the liver (hepatitis).

OSCE-guide-III.doc Page 168 of 255


OB-GYN

Advancement of HIV treatment:


- It used to be fatal
- Currently, it is not cured, but controllable, may be in the future they can develop a cure to
it
- A lot of anti-virals were developed since 1990s till now, with efficacy and side
effects, we started by giving many pills q4h, now it is once or twice a day
- We follow the guidelines with monitoring of what is called viral load and cell counts
of the patient immunity cells. To decide when to start treatment

Workup:
- Source patient (if available)
o HIV
o Hepatitis B antigen
o Hepatitis C antibody
- Victim/health care worker
o HIV; testing now, at 1 month, and at 3-6 months
o Hepatitis B surface antibody / titre (if vaccinated)
o Hepatitis C antibody; testing now and after 4 and 8 weeks
- Prior to initiating retrovirals:
o Pregnancy test (stat) if she is not pregnant
o CBC count with differential and platelets
o Serum creatinine/BUN levels
o Urinalysis with microscopic analysis
o AST/ALT levels / Alkaline phosphatase level
o Total bilirubin level

The ethical questions that might arise:


- We can solve all this by calling the lab and adding HIV status check for the list of blood
works of the patient, we had just send his blood to the lab!
o You are right, this will save us the whole prophylaxis plan, however, we need to
speak to the patient first; we can not do this HIV status test with the patient
knowledge and consent.
o What I am going to do to help you is that after we finish, I will go to speak
with the patient, explain the whole situation to him, and ask his permission that
we do this HIV statue test.
- Even if the patient refused, we can ask Dr , his surgeon, may be he knows!
o May be Dr knows the patient status or may not, but if the patient refuses to let
us know, it is patient confidentiality, we can not ask Dr about this information
unless the patient consents we can ask the surgeon.

OSCE-guide-III.doc Page 169 of 255


OB-GYN

Counselling PAP smear

CC I understand you are here because you have some inquiries/worries about your
last PAP test, is this right? How can I help you today?
HPI M
G  Previous Pap test? How many? How frequent? Any abnormal Pap test?
Any previous colposcopy?
 Contraceptive history
O GTPAL
S  RISK factors for cervical dysplasia:
- Early age of sexual activity
- Risky behaviour: unprotected sex / multiple partners
- Smoking
AS Same system - Any pain? With intercourse?
- Discharge? Itching? Redness?
- Any blisters / warts / ulcers?
- Inguinal swellings?
Nearby systems - Urine changes? Dysuria, frequency?
- Bowel movements changes? GIT symptoms
- Abdominal pain  OCD / PQRST /
General - Constitutional symptoms
PMH Any allergy / medication / disease
FH Gynecological tumours
SH If teenager: HEAD SSS
COUNSELLING:
- What do you know about (LGSIL)? What would you like to know?
- Have you had any experience with in the past?
- Have you [read / talked to someone / searched the internet] about this issue?
Worried about PAP results
- PAP smear or test is done to screen for any changes that might happen in the cervix,
before it turns to serious disease (to early detect pre-malignant lesions).
- At the cervix there is transitional zone between two types of cells, it undergoes rapid
growth, if there is irritation due to HPV, it might turn malignant. It takes years from the
moment it begins to grow abnormally to the moment it becomes malignant, that is why
we do frequent PAP tests, to detect it before it turns into malignant tumour.
- The results come back from PAP test either ASCUS (Atypical squamous cells of
Undetermined Significance) / LG-SIL (low grade squamous intra-epithelial lesion) / or
HG-SIL (high grade squamous intra-epithelial lesion)

OSCE-guide-III.doc Page 170 of 255


OB-GYN

- For ASCUS:
o Woman 30 yrs  HPV DNA testing
 If negative  repeat cytology after 1 year
 If positive  colposcopy
o Woman < 30 yrs  repeat cytology in 6 months
 If negative  repeat after 6 months  still negative  routine screening
 If ASCUS  colposcopy
- For LG-SIL:
o Colposcopy
o Or repeat cytology after 6 months
 If negative  repeat after 6 months  still negative  routine screening
 If ASCUS  colposcopy
- For HG-SIL:
o We send you for colposcopy

- For colposcopy, we will refer you to the gynaecologist who will perform special
procedure, during which, the gynaecologist will take a biopsy, and send it for further
investigations;
o If the biopsy is negative, we will repeat the PAP after 6 months
o If the biopsy is positive, we will do more investigations to establish a diagnosis
and may need to do another larger biopsy called cone biopsy

Treatment options:
- Laser
- Cone biopsy
- LEEP (loop electrosurgical excision procedure)

Colposcopy
- Colposcopy is a magnification of the cervix (10-12 times), the procedure may cause some
discomfort but is not painful.
- The gynaecologist will insert a speculum (the same instrument used for Pap test), and
then she/he will use a special magnification device (the colposcopy) to visualize the
cervix.
- The gynaecologist will apply acetic acid (vinegar) that helps make the vascular patterns
more visible, application of this acetic acid may give an itchy sensation.
- Then if the gynaecologist suspects a lesion, she/he will need to take a biopsy, you will
feel a punching sensation, and you might experience a little discomfort and spotting for
few days.
- You need not to have anything inserted into your vagina for 24 hours before and 2 days
after the procedure (no vaginal intercourse, no douching), and you might need to take
some OTC medications (Advil) for few days after the procedure.

OSCE-guide-III.doc Page 171 of 255


OB-GYN

Antenatal Counselling

o Ms XX has missed her period for 2 wks; she did a home preg test which was positive. This is
her first experience. In the next 10 min, please talk to her and give her necessary advices
about her pregnancy.
o A 38 yrs old pregnant lady came to you because she is concerned about problems during
advanced-age pregnancy, counsel

Pregnancy  Was this planned pregnancy?


 What is your feeling about that? And your partner feeling? congratulations
 How do you describe your general health?
 Symptoms suggesting pregnancy: any nausea / vomiting? Breast engorgement
/ heaviness? Frequency?
M  LMP, calculate due date
G  Pap smear?
O 
S 
PMH  HTN, Diabetes, Kidney disease, Heart disease, Epilepsy, exposure to
chickenpox as a child, Rubella vaccine, HBV, HIV, Blood Group & Rh group
 Allergy
 Medications
FH  FH of complicated pregnancies, abortions, MR, genetic & congenital
abnormalities
Social Hx  Home, violence, support
 Environment: work
 Activity: exercise / any pet
 Diet
 SAD
Concerns  Address any problem you find while youre taking history

OSCE-guide-III.doc Page 172 of 255


OB-GYN

Counselling:

 To ensure healthy outcome of the pregnancy I need to see you on scheduled visits, every 4
weeks till the 28th week, then every 2 weeks till the 36th week, and then every week thereafter
and till delivery
 Today well do physical examination including pelvic exam, Pap smear if more than 6
months, blood work including CBC, Lytes, INR/PTT, Urea, Creatinine, Blood Type, VDRL,
Rubella antibody, Hepatitis, HIV, Urine dip and microscopy, ECG.
 Anatomy US at 20 weeks. Glucose challenge test at 24 weeks
 Risks of Down syndrome are: 1/400 at 30 yrs of age, 1/200 at 35 yrs of age, and 1/100 above
40 yrs of age  we try to anticipate it by US and integrated prenatal screening then confirm it
by amniocentesis
 U/S for nuchal translucency: at 12 weeks
 IPS I: 11-14 wks /+/ IPS II: 15-18 wks (Maternal serum alpha-fetoprotein, -hCG, uE3
Unconjugated estrogen)
 Amniocentesis (U/S-guided trans-abdominal extraction of amniotic fluid / for identification
of genetic anomalies): at 15-16 wks, 0.5% risk of spontaneous abortion and risk of fetal limb
injury
 You need a well balanced diet; Canada's Food Guide to Healthy Eating suggests 3-4 servings
of milk products daily (greater if multiple gestation), a daily caloric increase of -100 cal/d in
the 1st trimester, -300 cal/d in the second and third trimesters and -450 cal/d during lactation.
If you do not consume an adequate diet, you can take daily multi-vitamins (avoid excess
vitamin A)
 Important nutrients during pregnancy: folate; 0.4-5 mg per day / calcium; 1200-1500 mg per
day / iron: 1 mg/d in T1, 4 mg/d in T2 and > 6 mg/d in T3
 Pregnant ladies tend to have constipation, you can take Lactulose for this, avoid raw or
processed meat
 Haemorrhoids, back pain, heartburn and increased vaginal discharge are common
 Will gain weight; 5-10 pounds in 1st half, 1 pound /week in 2nd half, total of 25-35 pounds in
average
 Exercise is OK walking, swimming, avoid strenuous activities
 Stay away from cats litter
 No medication without asking your doctor, no x-rays
 Smoking increase the risk of abortion, LBW, premature delivery
 No safe level of alcohol during pregnancy, better to avoid it totally
 Offer brochures, connect to support groups and classes for pregnant women

OSCE-guide-III.doc Page 173 of 255


OB-GYN

Endometriosis
You are covering for your colleague Dr. Smith. You are about to meet Mrs. XX to discuss the
result of her laparoscopy & inform her that she has endometriosis. For the next 10 minutes, please
talk to her& address all her concerns.
- Introduction: I would like to discuss the result of your laparoscopy but I need to get some
information.
o Why did you have laparoscopy done and what was your doctors concern?
o You have endometriosis:
This means implantation of the interior lining of the uterus somewhere in
other places outside the uterus including the ovaries, the supporting structures
of the uterus or on the intestine (draw a picture of the uterus and ovaries for
the patient).
During periods, this outside tissue also begins to bleed. This explains the
painful periods.
This may cause infertility in some people.
Sometimes it runs in the family
o I would like to ask about some symptoms (to fit everything together)
Pain: Analyze (OCD / PQRST). Relation to the period. Dysmenorrhea,
dysuria, dyschezia, dyspareunia. Do you need pain killer?
Infertility: I understand your frustration. For how long have you been trying
to conceive? Are you currently sexually active? How frequent?
Irregular vaginal bleeding  analyze
Frequency
Blood with stool, diarrhea
- Menstrual history brief
- Gynecological history
- Obstetric history GTPAL
- Sexual history brief
- Past medical history: HTN, Diabetes, kidney disease, blood group & Rh. Allergies /
medications / hospitalization / surgeries / blood transfusion
- Family history of abortion
- Social history: smoking, alcohol, drugs / work / home environments / support
- Conclusion: endometriosis
- Plan:
o Will do physical and obstetric examination
o Give her treatment options
Medical:
 NSAIDs e.g. naproxen sodium (250 mg PO bid)
16
 Pseudo-pregnancy: OCPs trial for 6-12 months (Ovral 1 tab PO od)
 Pseudo-menopause (only short-term <6 months): Danazole (weak
androgen / Side effects: weight gain, fluid retention, acne, hirsutism,
voice change)
Surgical treatment:
 Conservative laparoscopy: laser ablation / resection of implants
 Definitive: bilateral salpingo-oophorectomy hysterectomy
o Brochure & support groups
16
Ovral is progestin (levonorgestrel) and estrogen (ethinyl estradiol) combination OCP

OSCE-guide-III.doc Page 174 of 255


OB-GYN

Woman wanting an abortion

- History

Pregnancy Pregnancy (LMP, symptoms, how did you find out, Rh status) How do
you feel? How is the feeling of your partner about the pregnancy?
Gynecological history: contraception history, surgeries, infections, PAP
Obstetrical history: hx of previous pregnancies / GTPAL
Social Partner involvement? Abuse, rape?
Support; family, friends, spoken to any one else about this?
Education, Religious beliefs?
SAD
Abortion What are your thoughts regarding abortion?
Depression Exclude depression: MI PASS ECG
PMH / FH / SH

- Counselling

Be supportive and not judgmental


Pregnancy Health while pregnant:
Counsel for nutrition, exercise, and activity
Social Social support: it is important to communicate with support groups
Abortion Does not need to rush decision
Abortion options are available in Canada; most hospitals will do less
than 14-16 weeks, no problems with some up to 20 weeks. Private clinics
also provide the service
No legal gestational age where it can no longer be performed
Most commonly done via D&C under mild anesthetic
Discussion about contraception especially if not the first time (not
necessarily at this meeting)
Alternative choices: adoption / or rethink about keeping the baby with
the help of social support
Depression Normalize the patient depressed mood and feelings in view of
circumstances
Close follow-up
May refer to psychiatrist

OSCE-guide-III.doc Page 175 of 255


OB-GYN

Osteoporosis Counselling / OR / Short Case


Age related decrease in bone mass, starts at 35 years old, accelerated post-menopause, leading to bone
fragility and an increased risk of fracture. May manifest later as kyphosis, loss of height and increased
abdominal girth
Risk factors for osteoporosis:
1- Personal history of fragility fracture (above the age of 40 years)
2- Maternal history of hip fracture
3- Malabsorption syndrome (celiac disease / inflammatory bowel syndrome)
4- Prolonged use of corticosteroids (> 7.5 mg/day for > 1-3 months)
5- Primary hyper-parathyroidism

6- Weight < 57 Kg
7- Current smoker
8- Premature ovarian failure (female on Tamoxifen for breast cancer / surgical menopause)
9- Male on androgen-deprivation therapy for prostate cancer
10- Heparin or anti-epileptic use or biologics (anti-cancer treatment)

Investigations:
- BMD:
Age group < 50 years 50 65 years > 65 years
When to do BMD If > 2 of the first (5) If > 2 of any from the list Always do BMD, screen
risk factors of risk factors even there is no C/O
- Blood work:
o Serum calcium and phosphate levels
o Alkaline phosphatase
o Creatinine
o SPEP (serum protein electrophoresis)
o PTH (para-thyroid hormone)
o Give vitamin D for 2-3 months, then assess the level, if > 0.75 nanogram  it is normal and
do not repeat it again

Treatment:
- Based on BMD, risk factors, age of patient  Fracture Risk Stratification  low, moderate, or high
LOW MODERATE HIGH
- Life style If fragility fracture (in thoraco-lumbar x-ray) OR prolonged - Life style
modifications use of corticosteroids modifications
- F/U DEXA NO YES - Medical
after 5 yrs - Life style modifications - Life style modifications treatment
- F/U DEXA after 2 yrs - Medical treatment

Life style modifications Medical treatment


- Weight-bearing exercises (walking, jogging) - Bisphosphonates
- Ca 1200 mg/d (including the dietary intake, - SERM (Raloxifene): agonistic effect on bone
avoid Ca) but antagonistic effect on breast and uterus
- Vit D 1000 IU/d if < 50 yrs, and 2000 IU/d if > - Parathyroid hormone (PTH)
50 yrs - Calcitonin (if back fragility fracture + pain)
- Smoking cessation - HRT
- alcohol and caffeine

OSCE-guide-III.doc Page 176 of 255


OB-GYN

Pediatrics

OSCE-guide-III.doc Page 177 of 255


OB-GYN

History taking Pediatrics

History taking Pediatrics In any pediatrics case: always


 Introduction: CONSIDER child ABUSE / NEGLECT
 Chief complaint  report to CAS
1- Analyze the CC
2- Impact
3- Rule out infection
4- Differential diagnosis
5- BINDE - BINDE screening
6- Past medical history - Mother attitude!
7- Family history

1- Analyze the CC - TIME: Os Cf D: When did it start? How did it start? Sudden or
gradual?
- At that time, did your baby have any fever, flu-like symptoms?
- Is it continuous or on and off? How often? Day and night?
- Character:
- PQRST
- If vomiting or diarrhea: COCA + BLOOD
- Timing: is it related to feeding / meals?
- Factors: is it related to position? Meals?
2- Impact - Is he drowsy? Floppy?
- Does he cry? Is it high pitched cry?
- Did you notice his suckling is weaker than before?
3- Red flags - Constitutional symptoms!
Rule out infection - R/O infection: Did you notice if your child has fever or skin rash?
Cough / wheezes? Ear pulling or discharge? Runny nose? Foul
smelling urine? Abdominal distension? Diarrhea?
4- Differential - Rule out child neglect - BINDE screening
diagnosis - Mother attitude!
5- BINDE - Scan for risk factors for child abuse / neglect
6- Past medical history - Hospitalizations / surgeries / blood transfusion
- Illnesses (cancer) / infections
- Medications / allergies.
- Travel
7- Family history - Family history of similar disease in the family

0-6 years 6-14 years 14-18 years


BINDE School performance HEAD SSS
Pregnancy / Birth Home
Immunization Education
Nutrition Activity
Developmental Diet
Environmental Suicide
Sexual activity
SAD (smoking / alcohol / drugs)

OSCE-guide-III.doc Page 178 of 255


OB-GYN

BINDE

Pregnancy:
- Was your pregnancy planned? If no  social issues
- Were you having regular follow-up visits? How about U/S? Was it normal?
- During your pregnancy, did you have any illness? How about any fever or skin rash? Have
you ever been in contact with sick kids? Kids with skin rash or fever? Have you ever been in
contacts with pets?
- Did you take medications? Even OTC? Did you smoke? Drink alcohol? Have you ever tried
recreational drugs? What about before pregnancy?
- Were you screened for Hepatitis B virus? HIV? other diseases? There is screening test that
we do a vaginal swab at 36 weeks called GBS, did u have it?
- What is your blood group? What is your baby blood group?

Birth:
- Was your baby full term or not?
- Was it a vaginal delivery or c-section?
o If c-section  why? Was there any complication? Abnormality?
o If vaginal  was it difficult labour? Prolonged labour? How many hours? Was
there any early gush of water? How many hours? Did you need any help to make
it easier; e.g. vacuum?
- Did your baby cry immediately or not? Do you know what his Apgar score was? Did he need
special attention? When did you leave the hospital?
- Were there any bruises or swellings on your babys body?
- Were you told that your baby had any special features?
- After delivery, did you have any fever / discharge? Did you take any medications?

Immunization:
- Are your babys shots up-to-date?
o Yes  when was the last shot?
o No  any reason for that?
 Our religion prohibits vaccination: ok, that is fine
 We think vaccines cause autism: correct this info, vaccines are safe
 We were busy  neglect concern  what is baby weight?

Nutrition:
- Weight: Weight calculation:
o What is your babys weight today?
o What was his weight at birth? At birth : x Kg
o What was his highest weight? 5 months :2x Kg
o Do you have access to growth charts? 1 year :3x Kg
 If below 3rd percentile: underweight 2 years :4x Kg
 If (at any time) he crossed (down)
two major lines: failure to thrive More than 2 years old:
[(age X 2) + 8] Kg
Even low birth weight, catch up weight later, i.e. @ 1 year
they must be around 10 Kg, not only 3 x

OSCE-guide-III.doc Page 179 of 255


OB-GYN

- Height:
o To calculate height:
 At birth X cm 50 cm
 1 year 1 X cm + 25 75 cm
 2 years 1 X cm + 12.5 87.5 cm
 3 years 1 7/8 X cm + 6.5 94 cm
 4 years 2 X cm 100 cm
o For each year: the baby gains () of the previous year increase, so the baby gains
X by the first year, X by the second year, 1/8 X by the third year.
- Diet:
o What do you feed your baby?
 Everything we eat. No restrictions!  that is fine
 Breast feed  if more than 4 months:
Any iron supplement?
Any Vit D supplement?
 Formula:
Since when?  if since birth: any reason that you chose formula
over breast feeding?
Which formula? Any recent change in formula?
o For any case of chronic diarrhea?
 Do you give him solid food; biscuits / bread / cereal?
 Which started first? The diarrhea or this new food?
 How many diapers do you change per day? (normally 5 6)

Developmental:
Now I would like to ask you some questions about the kind of activities that your child can do,
and other questions to assess his development.
Gross motor Fine motor
Sit alone / roll over 6 months Draw line 15 months
Crawling 9 months Draw cross 2 years
Standing / cruising 1 year Draw circle 3 years
Walking 15 months Draw square 4 years
Go upstairs holding 18 months Draw triangle 5 years
Go downstairs 2 feet 2 years
Tricycle 3 years
Social Speech / verbal
Social smile 6 weeks Mama / papa 9 months
Stranger anxiety 6 months 2 words beyond Ma, Pa 1 year
Separation anxiety 9 months 2-3 words phrases 2 years
Says NO 2 years Short sentences 3 years
Speaks fluently 5 years

N.B. (autism / Down syndrome / child abuse): there is no stranger or separation anxiety.

OSCE-guide-III.doc Page 180 of 255


OB-GYN

Environment:
- How do you feel being a new mom? How do you feel about your baby?
o How is your mood? You look down for me, any chance you are being depressed?
Did you have depression before?
- With whom do you live? How is the relation between you?
o How is the relation between you and the baby?
o How is the relation between your partner and the baby?
- How do you support yourself financially?
- Do you live in home (basement: mold) or apartment? Is it an old building (lead)?
- Any other children in the house?
- Do you or any body in the home smoke? Drink? Use recreational drugs?
- Is anyone of your family seeing a psychiatrist? Has mental illness?
- In ABUSE cases: tell me more about your childhood

HEAD SSS
Home:
- With whom do you live?
- How is the relation between you? Are they supportive?
- Any siblings?

Education:
- Do you go to school? Do you like going to school?
- Which grade? Which subjects do you study?
- How about your marks, what marks do you get? What about in the past?

Activity:
- What kind of hobby do you have?
- Have you travelled recently?
- In EPILEPSY case: do you operate machines / drive / go hiking?

Diet:
- How about your diet? What do you eat? Do you follow special diet?
- What is your weight? What was your weight before?

Suicide:
- How is your mood?
- Any chance that you might hurt yourself?

Sexual activity:
- Are you dating? Are you in relationship?
- Are you sexually active? When did you start? When was the last time?
- How many partners do you have? Do you practice safe sex?

Smoking / Alcohol / Drugs:


- Now, I would like to ask you some personal questions, it is important to ask it, and it is
confidential, do you Smoke? Drink Alcohol? Have you ever tried recreational drugs?
- Sometimes people at your age might start to smoke, drink, or use recreational drugs. Do you
know any of your friends doing this? How about you? Have you tried that?
- For IV drugs: When was the last time? Did you share needles?

OSCE-guide-III.doc Page 181 of 255


OB-GYN

Jaundice
A new born 5 days old, with jaundice since day 2
Introduction Differential diagnosis of newborn jaundice
CC - Physiologic (usually days 2-7) 
- Analyze the jaundice (OCD) unconjugated
- Impact / consequences - Breast milk jaundice
- Red flags / rule out infection - Breast feeding jaundice
- DD - Pathologic (anytime)
- BINDE - Hemolysis (unconjugated)
- Birth  pathological - Infection  sepsis (conjugated or
- Nutrition  physiological unconjugated)

FH
Introduction:
Good morning Mrs , I am Dr , I am the physician in charge today, I understand that you are
here because your son has jaundice (or is yellow). In the next few minutes I will be asking you
some questions to help me figure out the condition, before I proceed, I would like to know the
name of your child? This is a nice name.

1- Analyze the CC:


- When did it start? Early in the 2nd day (or before: pathological) or late (pathological or
physiological)?
- Who noticed it? You or someone else? When? Where did you notice it? How about his eyes?
How about his feet? Is it spreading? Is it or ?
- How about his urine, is it darker? And stools, is it pale?
2- Impact / Consequences:
- Is he drowsy? Floppy?
- Does he cry? Is it high pitched cry?
- Did you notice his suckling is weaker than before?
3- Red flags / Rule out infection:
- Did you notice if your child has fever or skin rash? Cough / wheezes? Discharge from his
ears? Runny nose? Foul smelling urine? Abdominal distension?
- Any night sweats / chills? Any lumps or bumps in his body? Tender points?

4- Differential diagnosis:
Physiological Pathological
How do you feed him? Breast milk? Formula? Infection should be ruled out or
Breast feeding jaundice: (or lack of confirmed by now
breastfeeding jaundice): Not enough milk Hemolysis:
 dehydration  What is your blood group? Your
Breast milk jaundice: is more of a baby blood group? Father blood gp?
biochemical problem (inhibition of  Rh incompatibility  IUGR
bilirubin conjugation leads to increased  Were you screened for infections
levels of bilirubin in the blood). during pregnancy?
Treatment: substitute with formula Biliary atresia
Hepatitis: neonatal

OSCE-guide-III.doc Page 182 of 255


OB-GYN

5- BINDE
Birth:
Nutritional history:
- How do you feed him? Breast milk? Formula?
- Breast:
o How many times do you feed him?
o Do you use 1 breast or both of them? How long each?
o After feeding him, do you feel your breast engorged?
- Formula:
o Any reason to choose formula feeding?
o Which type of formula? Do you know how to prepare it?
Environment:
- Any other children? Did any of them develop jaundice after birth before?

6- PMH?!
7- FH:
- Jaundice
- Liver disease
- Blood disease
- Disease called cystic fibrosis

Diagnostic - Hemolytic workup: CBC / blood gp (mother and baby) / peripheral blood
workup: smear / Coombs test / bilirubin (direct and indirect)
- Septic workup: CBC / differential / blood & urine cultures / TORCH screen
- TSH and G6PD screening
- Liver enzymes / bilirubin / and coagulation profile
When to - If in the first day (or early second day) of life
suspect - Bilirubin rises > 85 mol/L/day
pathological - Bilirubin level > 220 mol/L before 4 days of age
jaundice? - Conjugated (direct) bilirubin > 35 mol/L
- Persistent jaundice lasting beyond 1-2 weeks of age
Treatment - Ensure proper hydration and feeding
- If sepsis: treat the underlying infection
- Phototherapy: if total bilirubin is > 300 mol/L, and only for unconjugated
hyperbilirubinemia, it is contraindicated in direct hyperbilirubinemia
- Exchange transfusion: if total bilirubin is > 400 mol/L

The mother has a concern: will he develop mental retardation?


1. This is a reasonable concern, I am glad you came here today to figure out
2. What made you think about that?
3. In order to be able to answer this, I first need to ask you some questions, examine him
and we may need to do some blood tests. However I would like to inform you that
jaundice in newborns is a common condition, and it is usually physiologic jaundice that
does not lead to mental retardation

Complications of Rh incompatibility: (1) kernicterus (brain damage  seizures), (2) Hydrops


fetalis (generalized edema), (3) Hypoglycemia

OSCE-guide-III.doc Page 183 of 255


OB-GYN

IUGR

A newborn 3 hours old with IUGR, counsel the mother

Introduction Good morning I understand that you just gave birth, my colleagues are
taking care of your baby. And I would like to ask you some questions
regarding your child health, but first tell me;
- How do feel right now?
- Have you seen the baby?
- Did you pick a name?
News Your baby has just been diagnosed with a condition called intra-uterine
growth retardation or low birth weight For that reason; I would like
to ask some questions about your pregnancy!
BINDE
Obstetrical history - GTPAL
- Were you pregnant before? How many times? Any abortions?
Miscarriages?
Mother PMH - Any history of chemo therapy or exposure to radiation
- Any family history with congenital anomalies

+ The mother has a concern: the baby is green


 This means he passed meconium (baby stool) which means the baby had a stressful delivery

+ The mother has a concern: is it my mistake?


 This condition is related to multiple factors, some factors are related to pregnancy, and others
are related to the baby or the environment. On the other hand, there are NO safe limits for
smoking / drinking alcohol during pregnancy, so, in the future, if you decide to become pregnant,
it is important NOT to smoke / drink alcohol / use recreational drugs during your pregnancy

Possible causes:
Smoking / alcohol / cocaine during pregnancy, (cocaine during pregnancy 
microcephaly, IUGR, MR)
TORCH infection,
Extreme of age, esp. advanced age pregnancy

Risks for the next 48 hours: asphyxia / hypoglycemia

OSCE-guide-III.doc Page 184 of 255


OB-GYN

Crying Baby

Introduction
CC
Analysis of CC OCD / all the time / day and night?
Is he crying > 3 hrs/day for > 3 days/week for > 3 weeks
What initiates or increases the crying?
o Any chance the baby is hungry? What do you feed him?
o Any chance that he is too hot / too cold? Do you adjust the
temperature?
o Any chance that he is wet? How often do you change his diapers
daily? Is there any skin or diapers rash?
What improves or decreases the crying? When he cries, what do you do?
o Did you try to hug / hold / burp / sooth / play music / give him a
walk?
o Did you try to rock him? Shake him? What happened to him?
When he cries, does he pull his legs? Is he passing gases? Is his abdomen
distended? Is it related to feeding? How are you coping with this?
Impact How does this affect your life? And your partner life? Are you able to go
to work?
Is he drowsy? Floppy?
Red flags / R/O Did you notice if your child has fever or skin rash? Cough / wheezes?
infection Discharge from his ears? Runny nose? Foul smelling urine? Abdominal
distension? Diarrhea?
DD Any infection (there will be other symptoms)  review of systems
Infantile colics (crying > 3 hrs/day for > 3 days/week for > 3 weeks),
between the age of 3 weeks and 3 months, without another explanation
 reassure
Child neglect
Feeding problems: overfeeding / hungry
BINDE Scan for risk factors for child abuse
Nutritional How do you feed him? Breast milk? Formula?
What about his weight?
Environmental With whom do you live? How is the relation?
How do you support yourself financially? Do you get
enough support?
Any other kids? Any repeated visits to the ER?
FH Mental problem
Parent SAD

Investigations (not including those for suspicious child abuse): CBC / urinalysis / stool analysis

OSCE-guide-III.doc Page 185 of 255


OB-GYN

Chronic Cough Asthma


Child complains of cough for 6 weeks, post-pneumonia, father wants to renew antibiotics

Here, we have two issues:


 The productive cough & fever 6 weeks ago  pneumonia
 The intermittent / dry cough that presented after

CC Cough
HPI Analysis of the  Os Cf D /+/ COCA + B + Phlegm
CC  Certain time of the day? Night?
Cough Acute phase Chronic phase
Continuous / productive / Intermittent / dry cough / on and off /
fever / loss of appetite no fever
 Seen by a doctor? What  SOB, noisy breathing, wheezes,
diagnosis? Treatment? chest tightness, nausea / vomiting
 Anti-biotic history!  Does he cough to the extent of
 Did you renew it? From vomiting or LOC
the same doctor? Was  Pertussis vaccination?
he examined? Any x-
rays were done?
Impact How did this affect his life? Daily activity?
Red flags  Constitutional symptoms
 Triggers of Asthma: any thing that this cough?
Differential diagnosis  Chronic diarrhea  cystic fibrosis
 Any allergy
BINDE Brief
PMH Other allergic diseases: atopic dermatitis / allergic rhinitis
FH Allergic diseases: asthma / skin allergies

Triggers Infection  Recent chest infection? Flu-like symptoms? Fever / chills?


Medications  How do you use puffers? Stored properly? Not expired?
 Did you start new medication? -blockers? Aspirin? Any recent
in dose of these medications?
Outdoor  Exercise
 Cold air
 Pollens (is it seasonal?)
 Dust: construction / smug (smoke/ fog/ exhaust)
Indoor  Do you smoke? Anybody around you?
 Do you have pets? People around you?
 Fabrics related: carpets floor? Any change in linen? Pillows?
Blankets? Mattress? Curtains?
 Relation to any type of food? + Perfumes
 Do you live in a house (basement  mold)?
 Any construction renovation? Exposure to chemicals?
Stress  Any new stressful situations?

OSCE-guide-III.doc Page 186 of 255


OB-GYN

Questions:
Diagnosis: hyper-reactive airways disease
Investigations: x-ray
Treatment: steroids puffer for 4 weeks

Counselling:
The most likely explanation for that is a condition called: hyper-reactive airways disease.
It is a term used to describe asthma-like symptoms in infants (< 6 years old) that may
later be confirmed to be asthma when they become old enough to participate in asthma
tests (spirometry and bronchodilators).
This is a common problem, and is usually triggered by infection (acute bronchitis or
pneumonia), it may last up to 10 weeks after infection.
It may be self limited; however, we need to start treatment with puffer (steroids puffer
for 4 weeks).
When the child becomes older than 6 years, and if the condition is still persistent for
more than 10 weeks, we send the child for investigations (spirometry and
bronchodilators) to confirm the diagnosis of bronchial asthma.

If this condition happens in adults, we treat with puffer for 4 weeks, if no improvement;
we send to investigate for asthma (spirometry and bronchodilators then metacholine
challenge test).

Case: 9 weeks history of cough on Amoxil for 2 weeks


DD: asthma / bronchitis / cystic fibrosis / recurrent pneumonia
Investigations: CXR / CBC, differential, lytes / sweat chloride test

OSCE-guide-III.doc Page 187 of 255


OB-GYN

Anemia
6-9 months, mother complains he is pale?
1- Analyze the CC - Clarify CC: What do you mean he is pale? Is he yellow?
- Os Cf D
- Who noticed it? You or someone else? Is there any chance that
he had this pallor before and you were not aware of it?
2- Impact - Is he drowsy? Floppy?
- Does he cry? Is it high pitched cry?
- Did you notice his suckling is weaker than before?
Signs of - Is he active / playful like before? What can he do? Is he crawling?
anemia - If he is doing activity, did you notice any SOB? Fainting?
- Is he gaining weight?
3- Red flags: rule out - Constitutional symptoms!
infection - Did you notice if your child has fever or skin rash? Cough /
wheezes? Ear pulling or discharge? Runny nose? Foul smelling
urine? Abdominal distension? Diarrhea?
4- Diff diagnosis: - Rule out child neglect
 Iron def. anemia - Bleeding disorders: nose / gums / coughing / vomiting / bruises
 Thalassemia on body / blood in urine / stools / joint swelling
 Hemolytic disorders - Leukemia: Constitutional symptoms / Bone pain [if he walks,
 Bleeding disorders does he limp? if you carry him, does he complain of tender
 Chronic diseases points in his body] / cough / repeated infection
 Lead intoxication
 Leukemia
5- BINDE - Scan for risk factors for child abuse / neglect
- N: What are you feeding him? Breast milk? From the beginning?
Do you give him any iron supplements or iron fortified cereals?
- B: was he term or not?
- E: with whom do you live? How do you support yourself
financially?  offer social support
- Where do you live, if old place, have you ever seen him eating
the paint scales?
6- Past medical history - Any heart / lung / kidney / liver disease?
- Hospitalizations / surgeries / illnesses (cancer) / infections
- Medications (Sulpha drugs G6PD deficiency) / allergies
- Travel
7- Family history - Family history of similar disease in the family
- Any bleeding disorder
- Any repeated surgeries? (cholecystectomy / splenectomy)
- Ethnicity: some blood diseases are more common in certain parts
of the world, that is why I need to ask you about your
background, what about your partner?
- Are you related by blood to your partner?

Investigations: lab works; CBC / differential / lytes / serum iron studies (ferritin, TIBC) /
hemoglobin electrophoresis / KFTs / INR / PTT
Treatment: iron supplement

OSCE-guide-III.doc Page 188 of 255


OB-GYN

Vomiting

The mother of (6 weeks 3 months) old baby came to the clinic complaining of childs repeated
vomiting.

Introduction
Chief complaint

1- Analyze - When did it start? How did it start? Sudden or gradual?


the CC - At that time, did your baby have any fever, flu-like symptoms?
- Is it continuous or on and off? How often? Day and night?
- COCA + BLOOD: what do you feed him? Is he vomiting the entire amount? Is
it watery or curdy?
- Character: is the vomiting forcible? Projectile?
- Timing: is it related to feeding / meals?
- After vomiting, do you fell he is still hungry? General condition after vomiting
- Factors: is it related to position? When lying down?
- Other GIT symptoms: diarrhea
2- Impact - Is he drowsy? Floppy?
- Does he cry? Is it high pitched cry?
- Did you notice his suckling is weaker than before?
- Dehydration: do you feel his lips dry / skin dry? Does he tear? How many
diapers
- Failure to thrive: what about his weight, do you know his weight? What was
his weight at birth? Do you have access to his growth charts?
3- Red flags - Constitutional symptoms!
- R/O infection: Did you notice if your child has fever or skin rash? Cough /
wheezes? Ear pulling or discharge? Runny nose? Foul smelling urine?
Abdominal distension? Diarrhea?

4- DD Differential diagnosis of 6 weeks vomiting


- Rule out child neglect - BINDE screening
- Mother attitude!
- Pyloric stenosis - Family history of pyloric stenosis
- Appear at age of 2 4 weeks
- Projectile / non-bilious / baby still
hungry after feeds
- Any infection / meningitis - Continuous
- Brain tumour - Other symptoms / neurological:
weakness / neck stiffness / seizures
- GERD - After feeds
- Wrong formula OR not preparing - Not all feeds
it well - No in weight
- Overfeeding OR NOT burping

OSCE-guide-III.doc Page 189 of 255


OB-GYN

5- BINDE - Scan for risk factors for child abuse / neglect


Nutritional history:
- How do you feed him? Breast milk? Formula? Which type? Do you know how
to prepare it properly? Any solid food?
- Which happened first, the vomiting or switching to the new formula?
- Weight analysis!
Environment:
- How do you feel being a new mom? How do you feel about your baby?
- How is your mood? You look down for me, any chance you are being
depressed? Have you ever been depressed before?
- With whom do you live? How is the relation?
- How do you support yourself financially?
- Do you get enough support?
6- PMH
7- FH

Pyloric stenosis DD: duodenal atresia / tracheo-esophageal fistula

Management plan:
- Investigations: lab works (CBC, lytes, ABG) / US
- If dehydrated: admission
- If suspicious child neglect: contact CAS

Potential risk factors for child abuse:


- SAD parents (smoking / alcohol / drug use)
- Pregnancy not planned
- Preterm baby
- Congenital anomalies
- Baby who needed special attention after delivery
- Separation from the child
- Difficult child
- Young couple
- Parents with history of abuse
- Stress or financial difficulties in the family

Investigations for child neglect CONTACT CAS


- Full blood work / CBC / albumin level
- Fundoscopy
- Skeletal survey

OSCE-guide-III.doc Page 190 of 255


OB-GYN

Diarrhea
Diarrhea
Failure to thrive FTT NO FTT
 What about his/her appetite? - Toddlers diarrhea
 What other associated symptoms? (Respiratory / Gluten) - Infections
Cystic fibrosis Celiac disease Milk protein HIV - Lactase Deficiency
allergy (lactose intolerance)
- Good appetite - Poor appetite From cow milk
- Respiratory - Gluten Should not be
given < 1 year

A 50 years old father comes with 9 months child with 6 weeks of diarrhea (CHRONIC)
1- Analyze the - Os Cf D
CC - COCA + BLOOD + others:
- Watery / loose / bulky
- Any undigested food
- Difficult to wipe?
- Factors: Juice (Excess fruit juice)
- Identify FTT weight: What is weight today? At birth? Last visit? The
highest weight? Not gaining weight?
- Other GIT symptoms: vomiting
- APPETITE
2- Impact - Is he drowsy? Floppy?
- Does he cry? Is it high pitched cry?
- Did you notice his suckling is weaker than before?
- Dehydration: do you feel his lips / skin dry? Does he tear? How many
diapers
- Failure to thrive: what about his weight, do you know his weight? What
was his weight at birth? Do you have access to his growth charts?
- Long period  malabsorption  anemia and rickets
3- Red flags: - Constitutional symptoms!
(R/O infection) - Did you notice if your child has fever or skin rash? Cough / wheezes? Ear
pulling or discharge? Runny nose? Foul smelling urine? Abdominal
distension? Diarrhea?
4- Differential DD for ACUTE diarrhea:
diagnosis - Use of antibiotics
- Infectious:
- Camping / travelling
- Any body else at home with diarrhea?
- Does he go to day care?
DD for CHRONIC diarrhea without failure to thrive:
- Toddlers diarrhea: does he drink too much juice daily?
- Infectious parasitic / travellers diarrhea
- Lactose intolerance:
- Does he pass a lot gas?
- Does he have any redness / skin rash at his buttocks?

OSCE-guide-III.doc Page 191 of 255


OB-GYN

DD for CHRONIC diarrhea WITH failure to thrive:


- HIV (if susceptible): was he screened for HIV
- Milk protein allergy: cow milk given before 1 year
- Cystic fibrosis
- Celiac disease
Cystic fibrosis Celiac disease
- Good appetite - Poor appetite
- Repeated chest infections? Cough? - bulky foul-smelling stools
- When did he poo the first time? - What do you feed your son? How
- Did he have yellow discoloration many times?
after birth? For how long? - Do you know how to prepare the
- Does he have any bulging through formula? Was it changed
his buttocks? recently?
- Did you start to give him solid
food? What type of food? How
about bread, cereals, biscuits?
Which started first, the diarrhea or
the new food?
- After you feed him, does he have
abdominal distension? Gases?
Pulling his legs?
- Does he have flat buttocks? Thin
legs and arms? Distended
abdomen?
5- BINDE - Scan for risk factors for child abuse / neglect
- Nutritional history
- Weight analysis
6- PMH
7- FH

Questions:
- What is your differential diagnosis:
o Cystic fibrosis
o Celiac disease
- If the biological mother called, want to know about her son, do you tell her or no?
o In order to determine whether I should release any information or no, I would
like first to know who has the legal custody (guardian) of this child. It might be
the adopting father, a social worker (case manager)

Notes:
- If the child was adopted, and you are speaking with one of the new parents:
o Are you the biological mother/father?
o Is this adoption or foster home?
o When was the child adopted? At which age? From where?
o What were the circumstances?
o Do you have information about the biological parents?
o Was he screened for HIV?

OSCE-guide-III.doc Page 192 of 255


OB-GYN

Mother worried about her child weight


- Not gaining weight
- Not eating well

1- Analyze the CC Weight analysis:


- Weight today, birth, last visit, highest
- When did you start to worry about that? Why?
- Who noticed it?
- Who is the primary care giver? For how long have you been with him?
- Do you have his growth chart? If no: if you do not mind, I need to
contact his family physician to take a look at his charts
Height analysis:
- Height today, birth, last visit

2- Impact - Is he drowsy? Floppy?


- Is he playful? Active like before? Any limitations? Does he turn blue
with activity?
3- Rule out any - Constitutional symptoms!
serious condition: - Any congenital or long term disease?
chronic inf/ dis / - Review of systems: cardiac/ chest/ GIT/ urinary/ MSK/ skin/ allergy/
malignancy pale/ bleeding
4- Differential - Rule out child neglect
diagnosis - Difficulty swallowing (CP, Cleft Palate)
- Chronic loss: chronic Diarrhea (Celiac disease, CF, pancreatic
insufficiency) / chronic vomiting (pyloric stenosis in a younger child)
- Diabetes mellitus (drinks too much water, pees a lot, tired)
- Chromosomal abnormalities / inborn error of metabolism
5- BINDE - Scan for risk factors of potential child abuse / neglect
- Apgar score at birth
- Diet in details: breast feeding/ formula/ cow milk? Any reason? For
how long? Any supplements? Any solid food?
- Developmental milestones
- Environment: with whom do you live? Who takes care of the baby? Is
he/she capable of doing this?
- SAD during pregnancy and now
6- PMH -
7- FH -

Under weight:
- Failure to thrive (FTT): weight decreases first then height will be affected later
- Endocrine causes: fat and short
- Congenital: everything is small / short, thin with small head

Failure to thrive
- Weight < 3rd percentile or falls across 2 majors percentiles
- Most common cause is inadequate intake

OSCE-guide-III.doc Page 193 of 255


OB-GYN

Case: A 2 years old boy does not want to eat. The father carries a bag!
History:
o When you ask about the bag, he says it is for the boy lunch, it is full of candy and a coke.
o Details about breakfast, lunch, dinner and snacks
o Review of systems  will be negative

Plan; to improve the health and diet education of the father:


o Educate about nutritious food and supplementation
o Give brochures and Canada dietary guidelines
o Refer to dietician specialist or nurse
Recommend supplementation:
o Iron
o Multi-vitamins

Examine the baby


Blood works: CBC / lytes / serum iron studies (ferritin, TIBC)

Differential diagnosis:
o Stresses int the family
o Child abuse / neglect
o Failure to thrive

Case: A 6 years old developed severe allergy to peanut, child is now stabilized, counsel the
father.
Is it first time to eat peanuts? Any similar reaction before? Any known food allergy?
Review of systems  will be negative

Past history of asthma, allergic rhinitis, allergic skin reaction?


Family history of allergy? Asthma, allergic rhinitis, allergic skin reaction?
Other siblings with allergic reactions?

Management:
Will send the boy for allergic testing
Strict avoidance of allergens
Epi-pen

OSCE-guide-III.doc Page 194 of 255


OB-GYN

Fever

Introduction
CC FEVER
1- Analyze the CC - Os Cf D
- Any flu at that time?
- Any diurnal variation? More at morning or night?
- Any special pattern? More every 2nd or 3rd day?
- Do you measure it? How many times daily? How do you measure
it?
- Did you try to give any medications to help? Did it help?
- Is it the first time?
- Other constitutional symptoms
- Other persons at home with the same symptoms?
2- Impact - Is he drowsy? Floppy?
- Does he cry? Is it high pitched cry?
- Did you notice his suckling is weaker than before?
3- Red flags - The fever and constitutional symptoms are already analyzed
- Review of systems: DD
4- Differential - Is he tired?
diagnosis: - Did you notice any skin rash?
Review of systems  OCD / distribution / color / do you feel it elevated?
 Are his shots up-to-date?
- Buttocks / abdomen  henoch schonlein purpura /
SKIN RASH

Investigations: urinalysis Treatment: steroids


- Trunk  vesiculo-papular  chickenpox
- Face: measles / rubella
- Cheek: fifth disease
- Headache / drowsy / neck pain / rigidity / nausea / vomiting? Does
he recognize you? Talk to you?
- ENT
- Chest: cough / phlegm / SOB / wheezes
- Abdomen (pain, distension, diarrhea) / liver (yellow color, itching,
dark urine, pale stools) / urinary (urine changes, crying while
peeing, loin pain)
- Joints: pain / swelling / mouth ulcers
5- BINDE - Scan for risk factors for child abuse / neglect
6- Past medical history

7- Family history

OSCE-guide-III.doc Page 195 of 255


OB-GYN

Runny Nose / Flu / URTI

8-15 years child is coming to see you with his mom, c/o: runny nose / flu / URTI?
Introduction - To BOTH the mother and the child
- During the encounter, distribute the questions and interaction between
both the mother and the child
1- Analyze the CC - Os Cf D
- COCA
- What or
- Is this the first time? Or did it happen before?
2- Impact - Is he playful? Active like before? Any limitations?
3- Red flags - Constitutional symptoms
Review of systems:
- Rule out infection: Any recent flu-like symptoms? Do you feel tired/
fatigue? History of sinusitis / Pain in your face? Any sneezing? Red
eyes? Pain/discharge in ears? Any sore throat/ oral ulcers/ tooth pain?
Pale / bleeding
- R/O meningitis: Neck stiffness / pain? Headache? N/V?
- Cardiac / chest / GIT / urinary / MSK / allergy
- Skin rash
4- DD - Allergic rhinitis: runny nose related to seasons, recurrent, no fever
- Viral flu: respiratory symptoms / joints & muscles ache
- Viral common cold
5- BINDE - Scan for the risk factors of potential abuse
- Immunization
- School performance
6- PMH - Any congenital or long term disease?
7- FH - Other members in the family with symptoms?
- School contacts?

Physical exam Mouth


ENT
LNs
Chest exam

OSCE-guide-III.doc Page 196 of 255


OB-GYN

Rash

Clinical Presentation When does the kid go back to school?


Mumps Fever, headache, parotitis (bilateral; pushes earlobes up and out), myalgia, Infectivity: 7 days pre-parotitis to 7 days
malaise post-parotitis
Measles Appearance: erythematous maculo-papular rash; Koplik spots No rash, no fever (infectivity: 4 days pre-
Timing: 10-14 days incubation, rash 3 days after start of symptoms rash)
Distribution: starts at hairline, spreading downwards; palms and soles
typically not involved
Rubella Appearance: pink, maculo-papular rash. Infectivity: 7 days pre-rash to 5 days post-
Timing: 14-21 day incubation; rash 1-5 days after start of symptoms. rash
Distribution: starts on face spreading to neck and trunk.
Chickenpox Appearance: macules  papules  vesicles  crusting; all stages apparent Infectivity: 1-2 days pre-rash until vesicles
(varicella) at once (polymorphous rash)  very pruritic have crusted over
Timing:
 10-21 days incubation;
 1-3 days prodrome: (fever and respiratory symptoms),
 Then rash
Distribution: face, trunk, extremities, mucosa, palms and soles.
Erythema Appearance: uniform, erythematous maculo-papular rash Infectivity: prior to onset of rash
Infectiosum Timing: 4-14 days incubation, rash 10-17 days after symptoms
(fifth disease) Distribution: bilateral cheeks with circum-oral sparing, can affect trunk
Roseola Appearance: pink maculo-papular rash (faint).
Timing: 5-15 days incubation; rash 3-5 days after symptoms.
Distribution: starts at neck and trunk spreading to face and extremities
Management: rest / anti-pyretics / fluids / good nutrition

Reye Syndrome:
 Acute hepatic encephalopathy and non-inflammatory fatty infiltration of liver and kidney
 Mitochondrial injury of unknown etiology results in reduction of hepatic mitochondrial enzymes, diagnosis by liver biopsy
 Associated with aspirin ingestion by children with varicella or influenza infection.
 40% mortality

OSCE-guide-III.doc Page 197 of 255


Pediatrics

Delayed Speech

Introduction Clarification: is it not gaining words, losing words OR


not speaking at all?
Verbal assessment
Rule out any serious condition - Hearing loss
- Autism
- Anatomical: tie tongue / cleft palate
BINDE
PMH - History of meningitis / jaundice
FH - Family history of deafness or hearing loss

VERBAL ASSESSMENT
- Would you please tell me more about that!
- When did you start to have concerns? Did you seek medical attention before?
- Is the child able to speak at all? How many words is your child capable of using? When
did he start to say it? Can he use many words in one sentence?
- Was he able to use more words (talk better) and lost them?
- How can he communicate with you? What does he do if he wants something?
I would like to ask you some questions in order to reach to the cause of this condition:
HEARING:
- How do you describe his hearing? Does he have hearing difficulties?
- If you call him, would he respond and reply? What if you are behind him? What if you
are in another room?
- Did you notice that he keep increasing the volume of the TV?
- Did he get repeated ear infections? Fluids in the ears? Discharge?
- Did he take any medications? Any antibiotics (aminoglycosides)?
- Was he ever screened for hearing test, when he was born?
AUTISM:
- Does he maintain eye contact? Does he show emotions?
- Is he aggressive? Does he play with other kids?
- Does he do repeated movements like rocking, or head banging?
- Does he have a favourite toy? How does he play with it? (train / spinning wheels)
- Any family history of autism?
BINDE:
- Start with the development: to rule out MR
Developmental (mile stones):
- What can he do? When did he start to sit? Crawl? Stand? Walk? Climb stairs?
- As a child, did strangers make him nervous?
- Does he control his urine / bowel movements?
Environment:
- Screen for neglect: how many hours you spend with him? Is he a difficult child?
- Family factor: how many languages do parents and other family speak at home?
Pregnancy / Birth:
- Did you have skin rash during pregnancy? TORCH infection? SAD during preg?
- Was it complicated labour? Apgar score?
- Did he have any special features? Congenital malformations? Cleft palate?

OSCE-guide-III.doc Page 198 of 255


Pediatrics

Seizing child counselling

Refer to the seizing child phone call case in the emergency medicine section for analysis of the
event

History Analyze the event During / before / after


Fever Analyze the fever
Rule out Rule out meningitis / pneumonia
BINDE
PMH
FH Febrile seizure / epilepsy
Counselling Introduction
Febrile seizures

Introduction:
Based on what you have told me, the most likely explanation of your child seizures is a
medical condition we call benign febrile seizure
What do you know about febrile seizures? Do you want me to clarify some information
about it? In details?

Febrile seizures:
This condition usually affects kids 6 months to 6 years, it is not uncommon, and a lot of
children (around 3%) might have attacks.
We do not know exactly the reason for it, but it is related to fever and may be because the
children brain is not fully developed at that age, and can not tolerate high fevers.
Usually it is self-limited, benign, typical attack is less than 15 minutes, and will not recur in 24
hours. Most children will outgrow their condition after the age of 6 years.
Another attack(s):
From the studies we know it might happen again; for each 100 child who got 1
febrile seizure attack:
o 65 children will not have it again
o 30 children will have another attack
o 3 children will have many other attacks even without fever
o 2 children will develop seizure disorder
The best treatment for it is the prevention that is why it is important to make sure that
whenever he gets a fever, to seek medical attention and to decrease the fever ASAP
using Tylenol or cold foments. Then find the source of fever and treat.
In case it happens again:
Turn the child on his side / protect him from hitting any nearby object / do not
force objects into his mouth
Bring to ER if seizure does not stop within 15 minutes
Diazepam 5 mg PR suppository
If repeated attacks, we may consider prophylactic anti-convulsion therapy
Will do CT, EEG
I will give you some brochures and web sites in case you want more information.
Any other questions or concerns.

OSCE-guide-III.doc Page 199 of 255


Pediatrics

ADHD counselling

The father comes to you saying that his son was diagnosed with ADHD two days ago and he has
concerns about ADHD and Ritalin. Counsel for 10 minutes.
Introduction To diagnose ADHD:
Address concerns - 2 settings (school / home)
 Diagnosis (symptoms of ADHD) - > 6 months duration
 Impact - < 7 years old child
 Differential diagnosis Differential diagnosis:
BINDE - ODD /+/ Conduct disorder
PMH - Specific learning disability
FH  ADHD / MR / autism / depression - Seizures (petit-mal epilepsy)
Conclusion - Depression
Introduction:
- Who diagnosed it? Usually teachers recognize it first (pick it), but to make a diagnosis a
psychiatrist, paediatrician, or a specialized nurse assessment is needed

Concern do you give Ritalin (which is amphetamine) to children?


- Actually yes. A lot of children use Ritalin, it is the first line of treatment for ADHD, and it is
effective and has been used for long time.
- It is not exactly amphetamine, it is the same family, it is called methylphenidate and it is
approved for this indication.
- In children, it helps them to focus as increases their concentration and channels their energy,
this is crucial for children, as it allows them to do better in schools. Even though we might
not be able to cure all children with ADHD, by we try to help them with education, so that
they can have career and live independently in the future, without problems with the law.
- It is generally a safe medication in children. It is not addictive, and we can stop it at any time.
However, like any other medication, it has its side effects, that include: insomnia (that is why
we give it early), abdominal pain, and not all children improve on it.

Before talking further about ADHD and Ritalin, let me first ask you some questions to see if your
child meets the criteria of ADHD or any other developmental challenge:
Diagnosis (hyperactive / inattentive / impulsive):
- Did the teachers complain that your child is full of energy? Spinning all the time? Refuse to
stand still? Talk all the time? Answers even if he is not asked? Does he stand in-line or does
he break the queues?
- Can he focus on one subject for > 30 minutes? Can he finish his tasks (e.g. the homework)?
Does he jump from one activity to another without finishing it? Does he lose his stuff? Does
he forget his belongings?
- Is this only at school or also at home?
- Did you notice that yourself?
- How much time do you spend with him? How about the mother, is she involved?
- How about before? Did anyone mention that or no?

IMPACT:
- Impact on functioning, school performance, relationship with peers

OSCE-guide-III.doc Page 200 of 255


Pediatrics

Differential diagnosis:
ODD - Does he like not to follow the instructions?
- Does he like to challenge his teachers and other family members?
Conduct - Is he aggressive? Does he fight a lot with other children?
disorder - Does he have a pet? How does he treat his pet / or other pets?
- Did you notice that he takes others belongings without telling them?
- Does he tell the truth all the time?
- Does he like to set fires?
Learning - Does he like to go to school?
disability - Does he have specific difficulty in reading / writing / mathematics?
Petit-mal - Does he have a history of seizures?
epilepsy - LOC? Abnormal movements?
Depression - Was he stressed recently? Any loss of a beloved one?
- Is he sad? Crying? Nightmares? Losing weight?
Autism -
MR -

BINDE: detailed developmental history

Conclusion:
- I am really sorry for this loss; it must be difficult for children in his age to go through all
that. How is he/she coping with that?

He has NO symptoms OR does not fulfill the criteria:


- Based on what you have told me, and the fact that it happened > 8 years old, it is less
likely that he has ADHD.
- On the other hand, his symptoms and behavioural changes could be related to the loss of
his mother, which might lead to depression. And in his age group, depression may
manifest as behavioural changes like what he is experiencing right now.
- We can refer your child to a child psychiatrist who can help him deal with that.

He has symptoms:
- Based on what you have told me, your child symptoms meet the criteria for diagnosis
with ADHD. However, this is not uncommon condition, and there is medical treatment
for it, in which the first line is Ritalin.
- Counsel on Ritalin.

Notes:
- Whenever you hear that one of the parents has passed away  show empathy.
I am sorry to hear that, it must be difficult for children in his age to go through all that.
How is he/she coping with that? How are you coping?

OSCE-guide-III.doc Page 201 of 255


Pediatrics

Vaccination counselling
New comer to Canada, comes to you as she has some concerns about vaccinations
Introduction / welcome her / how do you feel? Speak with enthusiasm (to
Identify the language barrier encourage) with three
Identify concerns counselling sessions:
- Deal with concerns one by one - Pap smear
- Pose frequently and ask if she has any questions - Breast feeding
Candidacy for vaccination - Vaccination
Mother vaccination
What are vaccines? Otherwise, speak neutrally
How do we vaccine?
Side effects of vaccines
Introduction / welcome her / how do you feel?
- Good evening Mrs vich, my name is Dr I understand that you are a new comer to Canada,
and you came to the clinic because you have some concerns about vaccinations. We will discuss
all you concerns. First of all, welcome to Canada, for how long have you been here? How do you
feel being here?
Identify the language barrier
- Before we proceed, am I clear, or do I need to talk slower? We can arrange for an interpreter or a
family member, if you would like to.
Identify concerns
- Now, can you tell me more about your concerns?
- Do you need general information, or do you have specific concerns?
o I heard that vaccines cause autism!
o I think we do not have these diseases in Canada, why should we give the vaccines for
diseases not common here?
Thanks for coming here to discuss your concerns with me.
Vaccines cause autism!
- What gave you this feeling? Concerns?
- There is misinformation among the general population that there is a connection between vaccines
and autism. And the origin of this misinformation is a study done in England many years ago, the
study found there is a connection between autism and 1 type of vaccines; namely the MMR.
- And because we take vaccines very seriously, a much larger study was done, in large number of
countries, including very large number of children. Now we found for sure that there is no
connection between vaccines and autism. The only relation is a coincidence between the age in
which parents start to notice autism symptoms and the age we start to give MMR.
- When we tried to figure out why the first study found the connection, the explanation of that was a
bias in the selection by the author and the study was conducted to favour this outcome. Another
theory to explain the connection was the preservative used in the vaccine (Thiomersal) and it
contains mercury. However Canadian vaccines do not contain it.
- I can assure you that there is no connection between vaccines and autism. Any questions till now?

Why to vaccine against diseases not preset in Canada?


- Even though we do not have these diseases commonly in Canada, and this is because we have a
successful vaccination program. But because the world is a becoming more and more a small
village, and people travel easily from a place to another, we do not want your child to be
vulnerable to preventable diseases if there is any outbreak anywhere. And I will give you some
examples:
o In England, people stopped to vaccinate their children for MMR, they started to have
measles infections again, and some children ended up losing their lives.
o Diphtheria is also coming back because of lack of vaccinations, even in many European
and developed countries.

OSCE-guide-III.doc Page 202 of 255


Pediatrics

o Tetanus is universal; it is in the soil everywhere, if a child is injured, the injury is


vulnerable to be contaminated with tetanus, which has serious fatal consequences.
- Do you have any questions or any more concerns?

Candidacy for vaccination / history BINDE:


Before I proceed further, I would like to make sure that your child is a good candidate for vaccination,
and we usually start vaccination at 8 weeks.
- Does your child have any illness, sickness, fever, flu-like symptoms? If high fever, we wait. Did
he get recurrent infections before? (if not very young)
- Was he diagnosed with any neurological diseases?
- Was he scanned for HIV? Were you scanned for HIV? Is there any risk that you might have HIV?
- Does he have any allergy?
Based on what you told me, your child is a candidate for vaccination.
- Is it ok till now?

Mother vaccination:
- As a child, were you vaccinated? How do you feel about that?
- If it is ok with you, we can set up a follow-up meeting to discuss in details your vaccination status
and find what vaccination(s) you might need to take.

What are vaccines?


- Vaccines are one of the most significant preventive interventions in medicine. It helped to save the
lives of large number of children all over the world. As children are vulnerable to be infected with
many infections, even thought it might affect adults, but usually with no serious consequences as
compared to children!
- We have large number of diseases and infections, caused by large number of bugs; some of them
cause serious illnesses. We try to protect against many of it. We take the different bugs (bacteria
and viruses), and we either kill it or we make it so weak so it will not cause any harm, then we
inject it into our bodies by a needle. Usually the body reacts by forming anti-bodies which are
protective chemicals against these bugs. Later in life, if the child will get exposed to the real bugs,
these anti-bodies will protect him.
- Was I clear? Any questions?
How do we vaccine?
- Because we have large number of diseases, and each one needs more than one shot, we try to
decrease the number of shots to be given.
- For example, there is a shot called pentacel that stimulates the body to produce protective
antibodies against 5 diseases, it protects against diphtheria, tetanus, polio, pertussis and
hemophilus influenza B. It is to be given at 2, 4, 6, 12 and 18 months and another booster dose at
around 5 years.
- You do not need to worry about memorizing them; I will give you a schedule of the required
immunizations to follow with his family doctor. And these vaccines are covered by OHIP.
- Any questions or concerns do you have?
Side effects of vaccines
- Vaccines are generally safe, millions and millions of children were and are vaccinated and it saved
the lives of large number of them.
- However, like any other medical intervention, vaccines have some side effects.
- It may cause minor issues, like pain, redness and swelling, at the site of injection, it may cause
fever, which we deal with Tylenol for pediatrics.
- There are some few rare side effects that include prolonged crying, seizure, and some children
become floppy, again, this is rare, but if this happens, you need to seek medical attention
immediately.
- Lastly, very rarely, children experience a serious allergic reaction, and that is why we keep the
children under medical supervision for 20 minutes after vaccination.
I hope that I answered you concerns, and now you have a better idea about vaccinations. Any other ideas or
concerns you need to talk about?
Thanks for coming and see you next visit for the first vaccination shot.

OSCE-guide-III.doc Page 203 of 255


Pediatrics

Child with DM counselling


Father of 6 years old boy comes to you as he has some concerns about his diabetic son

History - When was the child diagnosed? How?


- What type of insulin?
- Is DM controlled? Regular measure of blood glucose? Hb A1c!
- Symptoms of DM
- Symptoms of DKA
- Symptoms of complications: nephropathy / retinopathy / neuropathy
(usually develops more than 5 years after diagnosis)
Counselling - Do you have special concerns?
- What do you know about diabetes? Did you read? Hear? Anything?
- Type 1 DM
- Insulin
Complications - DKA /+/ Hypoglycemia /+/ Prevention of complications
- Macro and micro vascular complications
Management - Meal plan, education, exercise, psychological support
- Insulin injections with BS monitoring
- Children are more prone to brain damage with hypoglycaemia, therefore
high target range in younger and tight control in older kids

Diabetic ketoacidosis (DKA):


- Diabetic ketoacidosis (DKA) is an acute, major, life-threatening complication of diabetes.
DKA mainly occurs in patients with type 1 diabetes, but it is not uncommon in some
patients with type 2 diabetes.
- DKA is a state of absolute or relative insulin deficiency aggravated by ensuing
hyperglycemia, dehydration, and acidosis The most common causes are underlying
infection, disruption of insulin treatment, and new onset of diabetes.
- DKA is defined clinically as an acute state of severe uncontrolled diabetes associated
with ketoacidosis that requires emergency treatment with insulin and intravenous fluids.
- Symptoms of hyperglycemia associated with diabetic ketoacidosis may include thirst,
polyuria, polydipsia, and nocturia.
- Signs of acidosis may include shallow rapid breathing or air hunger (Kussmaul or
sighing respiration), abdominal tenderness, and disturbance of consciousness.
- Signs of dehydration include a weak and rapid pulse, dry tongue and skin, hypotension,
and increased capillary refill time.
Hypoglycemia:
- Hypoglycemia is a syndrome characterized by a reduction in either plasma glucose
concentration or its tissue utilization to a level that may induce symptoms or signs such
as altered mental status and/or sympathetic nervous system stimulation.
- Symptoms of hypoglycemia may be categorized as neurogenic (adrenergic) or
neuroglycopenic. Sympathoadrenal activation symptoms include sweating, shakiness,
tachycardia, anxiety, and a sensation of hunger. Neuroglycopenic symptoms include
weakness, tiredness, or dizziness; inappropriate behavior (sometimes mistaken for
inebriation), difficulty with concentration; confusion; blurred vision; and, in extreme
cases, coma and death

OSCE-guide-III.doc Page 204 of 255


Pediatrics

Bed wetting counselling / Nocturnal Enuresis

Introduction
1- Analyze the CC - OS CF D:
- When did it start? How did it start? Sudden or gradual?
- Frequency
- Primary or secondary (dry period(s) of time)?
- Is it continuous or on and off? How often? Day and night? Every
day? Every night?
- Factors: stress / drinking too much fluids before bedtime
2- Impact - How does Mom feel about it?
- How does the child feel about it (impact of this on child)?
3- Rule out infection - Constitutional symptoms!
- Did you notice if your child has fever or skin rash? Odd smell or
colour of urine? Pulls his penis? Cries while peeing?
4- DD - Rule out child - BINDE screening
neglect / abuse - Parent attitude!
- Medical conditions - DM (drinking too much water / going more
often to pee / feeling tired / losing weight)
- Diabetes insipidus (history of meningitis /
brain infection / head trauma)
- UTI (detailed in No 3)
- Neurological: trauma or surgery to back /
bowel dysfunction / leg weakness or
numbness
- Seizure disorder
- Stressors - New sibling
- Home / school change
- School performance
5- BINDE - Very briefly because the child is more than 6 years old
- Scan for risk factors for child abuse / neglect
- How is his school performance?
- Who is the primary care giver, who else does live with them at home,
is he the only child, any sisters or brothers?
6- PMH - Kidney disease
7- FH - Kidney disease
- Bed wetting
- DM
- Seizure disorder

OSCE-guide-III.doc Page 205 of 255


Pediatrics

Counselling - The condition is common;


- By the age of 4 years, 25% of children have this problem
- By the age of 7 years, 5-10% of children have this problem
- More common in boys than girls
- May be it is due to regression of his development because of the
current stresses in his life
- Condition is usually self limiting & you need to give the child some
time & he will adapt very well to the changes
Management - Try behavioural modification for 3 months:
- Avoid drinking late before sleep,
- Try to wake him up to go to the washroom,
- Document dry nights and use reward system.
- Alarm therapy
- Pharmacological measures:
- Desmopressin (DDAVP): 1 hour before bedtime. Be careful to
water intake, the only serious side effect is seizures due to water
intoxication
- Oxybutynin: at bed time, it bladder contraction therefore it leads
to bladder capacity

OSCE-guide-III.doc Page 206 of 255


Pediatrics

Breast feeding counselling


A lot of enthusiasm
 Congratulation for the news, how do you feel being a (prospective) mother? It is good to hear
that you plan to breast feed. Do you have any concerns?
 I need to ask some questions
If still pregnant:
How the pregnancy is going so far?
Pregnancy questions of BINDE
Ante-natal care follow-up (from OB-GYN)
If already delivered:
Birth questions of BINDE

 PMH: medications (lithium, antibiotics, anti-convulsions) & allergies


 Social history: smoking, alcohol, drugs / work and home environments

1- Rule out contra-indications for breast feeding:


- The mother has active herpes simplex on breast
- The mother has untreated, active T.B.
- The mother has been infected with HIV / is taking antiretroviral medications
- The mother is using or is dependent upon an illicit drug
- The mother is taking prescribed cancer chemotherapy agents, such as antimetabolites
that interfere with DNA replication and cell division OR is undergoing radiotherapy;
however, such nuclear medicine therapies require only a temporary interruption in
breastfeeding
- An infant diagnosed with galactosemia, a rare genetic metabolic disorder
- N.B.: Hepatitis B is NOT a contraindication to breastfeeding

2- Counselling:
 Advantages of breast feeding
- Highly nutritional, providing all elements baby needs (especially colostrum), breast milk
contains: more vitamin C, easily absorbable iron, less protein load on the baby
- Contains antibodies to help your child fight infections
- Ready, worm, clean, economic, sterile
- Less allergic
- Secures bonding between mom and baby, emotional satisfaction for the mother and
creates sense of security for the baby
- Help mom reduces weight, a method of contraception

 Problems with breast feeding that might happen to the mother:


- Sore nipple (clean & apply soothing lotion)
- Engorgement (pump)
- Mastitis (treat & do not stop feeding)

OSCE-guide-III.doc Page 207 of 255


Pediatrics

3- Advice:
- Mother should get enough nutrition, fluids, vitamins and rest.
- Give supplementations of:
o Vitamin D from day 1
o Iron from 4 6 months
o Start solid food from 4 6 months, I will give you a table with the recommended
time and types to start solid food
- Mother can use OCP but it will reduce amount of milk OR use an IUD
- Avoid using any medication without asking your Doctor
- Avoid smoking & alcohol
- Care of the breast: frequent cleaning with water and proper hygiene, warning signs:
engorgement, tenderness, redness, hotness
- I will give brochures & information about BF classes
- I will give you the immunization schedule so that you remember to bring him for follow-
up and for vaccination
- Do you have any questions or concerns?

4- Frequently asked questions about breast feeding:


 How often you should feed your child?
- On demand at the beginning
- Then the child will adapt to a schedule of every 3 4 hours

 How long should the baby stay on each breast? (10 minutes)

 How do you know that your baby is feeding well?


- Gaining weight
- Sleeping well
- Wetting 5 6 diapers daily
 What do you do if you do not have enough milk? (You can supplement with formula)

 How can you breast feed & work at the same time?
- Use pump & keep the milk in a bottle for 3 6 hours outside and 24 hours in a fridge,
you can keep it in the freezer

OSCE-guide-III.doc Page 208 of 255


Pediatrics

Psychiatry

OSCE-guide-III.doc Page 209 of 255


Pediatrics

Mental status exam the psychiatry interview


ABS MAP TCIJ ALWAYS clarify! What do you mean by that?
Appearance Dressed Dishevelled: dressed and groomed
Groomed poorly
Given age matches his chronological age
Dressing matches the weather
Behaviour Psychomotor retardation vs. Psychomotor agitation
Eye contact *
Cooperative *
Hostile *
Abnormal movements / jerks / tics / lip smacking (anti-psychotics)
Speech Volume (low / normal / loud)
(process of Tone (parkinsonism: monotonous)
talking) Articulation
Fluent
Pressured speech
Mood How is your mood? How do you feel? Write it in patient own words
Affect Quality (by pt observation) Elevated
Euthymic
Depressed
Anxious
Congruency Appropriateness
Others Stable / range / labile / flat
Perception Normal perception
Illusions
Hallucinations
Thought Processing (how does the patient connect Goal directed
ideas); e.g. how did you come here today? Thought block
Flight of ideas
Loose association
Tangentiality
Circumstantiality
Contents Obsessions
Delusions
Suicidal / homicidal ideation
Cognition Mini mental exam
Insight
Judgement

Brief comment: (1) The patient is well dressed, well groomed; and his appearance matches his
chronological age. (2) He has (good / poor) eye contact, cooperative (not), with psychomotor
(retardation / agitation) (3) His speech is of normal volume, tone, fluent, not slurred, and not
pressured. (4) His mood is (5) His thoughts are organized (or disorganized). (6) There are no
delusions or hallucinations. (7) There is no suicidal ideation or homicidal thoughts. (8) Judgement
(good / poor), insight (intact / lost).

OSCE-guide-III.doc Page 210 of 255


Pediatrics

Perception
Hallucinations:
- Visual:
o Usually organic (tumour / epilepsy / cocaine and amphetamine)
o Brain tumour /+/ alcohol intoxication / DT /+/ cocaine / hallucinogens
o Do you see objects / things that others do not see?
o Can you describe what do you see?
o Do they give you any messages?
o Are these messages asking you to harm yourself or anyone else?
- Auditory:
o Usually schizophrenia
o Do you hear voices / things that other people do not hear? When you are alone,
do you hear voices coming from your head?
o How many voices
o Are they familiar or not?
o Are they talking to you or about you? What are they telling you?
o Did they ever ask you to harm yourself or somebody else? What is preventing
you from doing this?
o How do you feel about these voices?
- Tactile:
o Cocaine chronic use (most probably) OR delirium tremens
o Do you feel ants / insects crawl on your body / skin?
- Smell: usually epilepsy

Though
Processing:
o How did you come here today?

Content:
+ Obsessions:
- Repeated intrusive thoughts that the patient knows it is wrong, and he can not resist, if he
resists  anxiety  take actions to try to anxiety (compulsions)
- Mostly regarding: cleanliness, contamination / order / checking /
o Do you have any repeated thoughts or images that you find difficult to resist?
About what? What do you do?

+ Suicidal / homicidal ideation:


o Do you have any thoughts or ideas of harming yourself?
o Or harming other people?
o Any access to weapons?
- If the patient is suicidal / homicidal / can not take care of himself  admit, if he/she
refuses  form 1 (for involuntarily admission for 3 days for psychiatric assessment
by another physician). Form 1 has to be filled within 1 week from seeing the patient.
- I want to file form 1 for the patient and call the hospital security to bring the patient back.
I have concerns about safety of the patient and other people.

OSCE-guide-III.doc Page 211 of 255


Pediatrics

+ Delusions:
- False fixed believes, that do not match with the patient cultural and religious background
- You can not convince the patient it is wrong, even with proof
- The ideas
o Believable (could be) non bizarre
o Unbelievable (could never be) bizarre
o Do you believe that other people would like to harm you? OR conspire against
you?
o Do you think that others would like to control you?
o Read your mind? Thought broadcasting
o Put thoughts into your head? Thought insertion
o Steal thoughts from your head? Thought withdrawal
o If you are watching the TV or reading the newspaper, do you believe that they
are talking about you? Delusion of reference
o Do you believe that you are a special person? With a special talents? Or special
power? Do you believe that you have a special mission to do in life? Do you
think you deserve to be treated specially? Grandiosity
o Do you feel other people are falling in love with you? Eromantic
o Do you believe any part of your body is rotten?

Cognition:
- Are you becoming forgetful? Are you losing your staff?
- Assess abstract vs. concrete thinking!

Insight:
- Do you think that you are doing well? Or do you need help?

Judgement:
- If there is a fire in the building, what are you going to do?
- If you find a stamped and addressed envelop on the ground, near the mail box, what
would you do?

General screening:
- Depression:
o What is your mood? How do you feel?
o Did you lose interest in things that were interesting to you before (e.g. certain
hobby, playing something)?
- Anxiety:
o Are you the kind of person who worries too much?
o Do you have excessive fears or worries?
- Psychosis:
o Do you hear voices or see things that others do not?
o Do you think that someone else would like to hurt you?

OSCE-guide-III.doc Page 212 of 255


Pediatrics

DSM-IV-TR
Diagnostic and Statistical Manual of Mental Disorders 4th Ed/2000 Text Revision
Multi-axial system (5 axes)
The DSM-IV organizes each psychiatric diagnosis into five dimensions (axes) relating to
different aspects of disorder or disability:
- Axis I: Clinical disorders, including major mental / psychiatric disorders, and learning
disorders, Substance Use Disorders
- Axis II: Personality disorders and intellectual disabilities (although developmental
disorders, such as Autism, were coded on Axis II in the previous edition, these disorders
are now included on Axis I)
- Axis III: Acute medical conditions and physical disorders
- Axis IV: Recent stressors, i.e. psychosocial and environmental factors contributing to the
disorder
- Axis V: Global Assessment of Functioning or Children's Global Assessment Scale for
children and teens under the age of 18 (a questionnaire)
Example of a full proper psychiatric diagnosis:
- Bipolar I / Anti-social personality / DM+HTN / Divorce / global assessment was not
done because the patient was not cooperative
Diagnosis of diseases based on DSM-IV-TR is based on CRITERIA and TIME.
- Depression:
o MI PASS ECG or MIS GE CAPS
o You need to find at least 5 of the 9 for > 2 weeks, including at least one of the
Mode or Interest.
o If not fulfilling these criteria: non-specified mood disorder
o In teenagers: we do not need M or I, we can replace it with agitation OR drop in
school performance + other 4 criteria.
- Schizophrenia:
o 4 positive symptoms: hallucinations, delusions, disorganized speech,
disorganized behaviour.
o 1 other category; negative symptoms: mood, catatonia
o At least 1 month of active symptoms (2 of 5) + 6 months of deterioration in
functioning.
o 1 active symptom (not 2) is accepted in the following cases:
 If the hallucinations are > 2 voices (commanding or commenting)
 The delusions are bizarre
- Anxiety:
1- Panic attack vs. panic disorder:
a. In panic disorder, there is at least ONE panic attack with at least ONE month
of worries and fears of having it again
b. Panic attack might be one or more attacks
c. If patient is avoiding going outside  with agoraphobia
2- Phobias specific to certain objects
3- OCD
4- GAD: excessive unrealistic fears for more than 6 months PLUS other manifestations
5- PTSD (acute or chronic): Have you ever encountered a situation in which your
personal or mental safety and wellbeing were endangered? When? Do you have
flashbacks or nightmares?

OSCE-guide-III.doc Page 213 of 255


Pediatrics

History taking Psychiatry

MOAPS: mood / organic / anxiety / psychosis / serious conditions (self care, suicide, homicide, support) / HEADSSS
Major psychiatric illness Suicide Minor psychiatric illness
Personality disorder / Drinking
S / addiction / Eating / sleeping
Mood Anxiety Psychosis A disorders / Somatic disorders /
D Cognitive (delirium /
dementia) ...
 Low: MI PASS ECG 1- Panic attack vs. panic Criteria (1 month of 2-5 active P Criteria
 High: DIG FAST + disorder symptoms + 6 month of E
MI PASS ECG 2- Specific phobias function deterioration) R
- 1st time or did you have it 3- OCD - 1st time or did you have it S
before? 4- GAD before? O
- What about the opposite? 5- PTSD N
Dx: one of the mood disorders S
Past psychiatric history Past psychiatric history
Organic: MOAPS
1- SAD if IV drug use: check for liver (hepatitis) / constitutional symptoms (HIV)
2- PMH, including constitutional symptoms
3- Rule out medical conditions as DD, e.g. medications and specific diseases
Thyroid disease Mitral valve prolapse Brain tumour / HIV
Anxiety / psychosis Mood / psychosis Mood / anxiety
Serious conditions (red flags):
- Self care (are you eating / sleeping well?)
- Suicidal / homicidal ideation
Social history:
- How do you support yourself financially?
- With whom do you live? Family support?
For teenagers, add: HEADSSS
Family history of psychiatric illness: suicide / depression / SAD / seen by psychiatrist

OSCE-guide-III.doc Page 214 of 255


Psychiatry

Mood disorders:

OSCE-guide-III.doc Page 215 of 255


Psychiatry

Psychosis
Pt comes to the clinic complaining of strange feelings in his right hand

Clarify the CC 1- Can you tell me what is going on!


OSCD 2- What do you mean? Is it pain, numbness, tingling?
3- OS: When did it start? What were you doing?
4- C: Is it all the time, or on and off? Any specific setting?
5- In you opinion, why do you have this?
6- In addition to , do you have any other strange feelings?
Criteria :  Any hallucinations: What? For how long?
 Hallucinations  Any (tactile or visual) hallucinations  will be mostly organic
 Delusions  cocaine (substance abuse) until proven otherwise
1st time or did you have it
before?
Past psychiatric history Have you seen a psychiatrist before?
Organic Cover the following:
 Head injury / trauma
 Brain infection (fever, vomiting) / Brain tumour (N/V)
 HIV
 Thyrotoxicosis
 Medications: thyroxin, anti-parkinsonism (L-dopa)
 SAD (cocaine, marijuana, amphetamines)
Mood / Anxiety
Serious conditions Constitutional symptoms / self care / suicide
Social history
Family history

Differential diagnosis:
- Schizophrenia (a mental disorder that impairs the way you perceive reality. It could be
very disabling)
- Brief psychotic disorder
- Post-partum psychosis
- Drug-induced
- Brain tumour
- HIV
- Delirium / dementia
- Mood disorder

Investigations:
- CBC / toxicology screen
- HIV / syphilis test
- Septic workup
- CT / MRI brain

Pay attention to patient cues


- Poor hygiene
- Looking at wall or ceiling
- Paranoid
- Talking to some body.

OSCE-guide-III.doc Page 216 of 255


Psychiatry

Management:
Will examine and do some tests
o Because you have stopped your medications, it looks like your schizophrenia is
relapsing, that is why we need to admit you and refer you to psychiatrist to
reassess your condition, how do you feel about that?
Will start medication which is helpful in reducing the symptoms (Risperidone). Explain
about side effects: weight gain / blood glucose level / cholesterol / drowsiness
Arrange follow up visit
Information e.g.: support groups / brochures

Notes:
Whenever you suspect substance abuse: after you ask have you ever tried recreational
drugs? ask what about crack cocaine? Do you sniff? Do you inject? Did you share
needles
o If shared needles  scan for hepatitis (liver symptoms), HIV (repeated
infections / repeated diarrhea)
If the patient came because his parents or roommate have concerns, you can ask the
patient: what kind of concerns does have?

Difficult situations:
If the patient with hallucinations tells you that he sees a radiation and gives you a photo and asks:
do you see it doctor?  For me it does not look like radiation, but I can understand that you see
this as radiation
At any time the patient starts to agitate and worries about special hallucinations!
o You are safe here, no body will harm/hurt you
If the patient is away:
o Do not chase him/her around the room, stand by your chair
o I would like to assure you that you are safe here, no one will harm you
I do not like Egyptian people, by the way, are you Egyptian doctor?
o Why are you concerned about that?
o Whether I am Egyptian or not will make no difference in this situation
I do not like gays, by the way, did you see a gay patient today doctor?
o Why are you concerned about that?
o As a physician, I deal with all patients, regardless their race, religion, sex, sexual
orientation or anything else!
Do you think I am crazy doctor?
o There is no medical term called crazy. However sometimes some people have
difficulties in the way they handle their thoughts and the way they interact with and
perceive reality, we call that schizophrenia. It is a mental illness like any other illness that
can affect the body, that we can treat with medications

Case: A young man can not move his neck, DD acute dystonia:
Trauma
Meningitis
Subarachnoid hemorrhage
Cervical disc
Muscle spasm
Anti-psychotic medication (e.g. haloperidol), treatment: lorazepam

Case: Patient is in the balcony, wants to fly, mother is calling you?! What do you tell her?
Talk to him to attract his attention
The doctor should get the phone number and address and ask the nurse to call 911
Ask her if you can speak with the patient  psychosis patient

OSCE-guide-III.doc Page 217 of 255


Psychiatry

Schizotypal personality disorder

- Delusions
- Magical believes
- Limited number of friends that share the same believes

Ethical challenges:
- Will you hook me to the cleaning machine that cleans the blood? I am glad you came
here today, I think you need help, but not with the machine.
- Will you admit me doctor? We need further psychiatrist assessment then we may need to
admit you.

OSCE-guide-III.doc Page 218 of 255


Psychiatry

Panic attack

Patient comes to the clinic complaining of dizziness

Clarify the CC When you say dizziness, do you feel:


- light headed
- Spinning
Analysis of CC Os Cf D
Analyze the attack: How did it end? How many attacks? Are they similar?
What were you doing? When was your last one?
HPI Criteria  cardiac: heart racing17 / chest pain / tightness / excessive sweating
during  respiratory: SOB / wheezing
attack  neurology: dizziness / numbness / tingling / weakness / shaky /
buzzing sounds / headache / vision changes / difficulty balance
(AS)  GIT: nausea / vomiting / difficulty swallowing
 Depersonalization: you feel that you are outside of your body
 Derealisation: feel things around you are strange / not real
 Excessive fears of: losing control / going crazy / dying
Between  Do you have fears of having other attacks?
attacks  How does it affect you? Do you avoid going out? (Relation to
agoraphobia?)
Anxiety  Are you the kind of person who worries a lot? Excessive fear
 Are you under any stress in your life? How can you cope with this?
 Any special fears? High altitudes? Closed places? Talking in public?
Pets?
 Have you ever encountered a situation in which your personal or mental
safety and wellbeing were endangered? When? Do you have flashbacks
or nightmares?
MOAPS  Hypoglycemia
 Thyroid disease
 Pheochromocytoma
 SAD (cocaine / amphetamine / alcohol withdrawal) / Caffeine
 Arrhythmias / MVP
 Anemia (fatigue / light headedness / heavy menses / PMH anemia)
Past psychiatric
history
Serious conditions  Constitutional symptoms
 Self care
 Suicide
PMH AMPLE
Family history Heart diseases / thyroid / abdominal tumours
Social history

17
Any heart racing, ask the patient can you tap it for me, then comment to the examiner: it
looks regular / irregular for me

OSCE-guide-III.doc Page 219 of 255


Psychiatry

COUNSELLING

- With what I heard from you today, the most likely diagnosis to your symptoms is a
medical condition that we call panic attack. We still need to do physical examination,
some investigations like blood works, urine analysis, electrical tracing of your heart
(ECG), to exclude other medical conditions and to confirm our diagnosis.
o Now Mr what do you know about panic attacks?
o Do you want me to explain this in details over the next few minutes?
- Inform the patient:
o Explain the pathophysiology: panic attack or panic disorder is a kind of severe
anxiety, it happens suddenly, in attacks. Usually it is related to stress.
o It is due sympathetic over-activity, imagine you are crossing the road, and a
speedy car is approaching you, normally, our body reacts to this by enhancing the
sympathetic nervous system, which leads to some changes: increase in the heart
rate, rise in blood pressure, and you feel alert. This is normal and useful reaction.
o The same reaction might happen suddenly without any external trigger, and this
would be stressful, and this is what we call a panic attack.
o Consequences: this might happen again / may cause significant limitations
- Preventive measure:
o Life style modification (caffeine and alcohol / better sleep hygiene)
o Relaxation techniques (e.g. breathing techniques / meditation)
- Treatment:
o Like many other conditions, it could be treated.
o Treatment varieties include:
 Talk therapy
 Medications: 2 types
Anti-anxiety: Lorazepam 0.5 mg qhs x 2 weeks (it is important
to use it on schedule, not irregularly)
SSRIs: Paroxetine 10 mg od x 4 weeks similar to what we
usually use with depression. Like any other medication, they
have their side effects; GIT disturbances, headache, some sexual
dysfunction. And this improves by time.
Follow-up 2-3 weeks
- Offer more information: brochures / web sites
- Whenever you suspect social problems  involve the social workers

Generalized Anxiety Disorder (GAD)


 Excessive anxiety and worry (apprehensive expectation), occurring more days than not for at
least 6 months, about a number of events or activities (e.g. work / school)
 The person finds it difficult to control the worry
 Treatment:
o Lifestyle: caffeine and alcohol avoidance, sleep hygiene
o Psychological: psychotherapy, relaxation, mindfulness, and CBT
o Pharmacological:
 Benzodiazepines (short term, low dose, regular schedule, long half-life,
no PRN)
o Buspirone (tid dosing)
o Others: SSRIs/SNRI (paroxetine), TCAs (nortriptyline), beta-blockers
o Avoid Bupropion due to stimulating effects

OSCE-guide-III.doc Page 220 of 255


Psychiatry

Tiredness OR weight loss

Introduction
CC Tiredness
Clarify the CC - Is it weakness? Can not do?
- Lack of energy? Tiredness?
- Limitation of activity? How many
blocks are you able to walk?
- Not being refreshed after sleep? Do
you have any special concerns?
Os Cf D Timing:
- Morning or all day: ?depression
- End of the day: organic
Ask about sleep - How many hours? And before?
- Find difficulty falling asleep?
- Do you wake up during night?
- When you wake up, do you feel
refreshed? Do you need naps?

sleep /or/ the same sleep / insomnia


? organic ? depression
- Constitutional symptoms Criteria : MI PASS ECG
- Tender points in your body
All systems review (head to toe): 1st time or did you have it before?
- Cardiovascular What about the opposite?
- Lung Past psychiatric history
- GIT / liver Organic //
- Urinary Anxiety / psychosis
- MSK / skin / rheumatology / auto- Serious conditions
immune Social history
- Anemia / bleeding Family history
LMP / mens. hx / pregnancy
Cancer colon for males
- Endocrine (thyroid / DM) Counselling on depression
MI mood / interest
PMH of cancer
Social history SAD
Family history

Diabetes Mellitus:
- Hx of DM - Blood sugar measured - Symptoms:
Fluctuations (acute) Emergencies Complications (chronic) vascular
MICRO MACRO
- Eat more - Blurred vision DKA - Nephropathy - CAD
- Drink more - Tired Hypoglycemia - Neuropathy - CVS screen
- Pee more - Weight loss - Retinopathy - PAD /
impotence
N.B. -blockers are contraindicated in DM: it causes hyperglycemia / and it masks hypoglycemia

OSCE-guide-III.doc Page 221 of 255


Psychiatry

Sleep / fatigue notes

1- Sleep hygiene questionnaire:


- How many hours? How about before?
- How does this affect your life? Do you work night shifts? empathy
Before - When do you go to bed?
- Find difficulty falling asleep? How long does it take you?
- Do you sleep in dark room? Lit room?
- Do you eat before sleeping? Heavy meals? Late meals?
- Do you drink before sleep? Alcohols? Coffee?
- Do you exercise before sleep?
- Do you read in bed? Watch TV?
During - Do you wake up during night? Any reason? Can you sleep again?
- Do you sleep alone? Or do you have sleep partner?
o Does he notice you are snoring? Do jerky movements?
o Does he snore? Does he do jerky movements?
- Do you have dreams? Nightmares?
After - Do you wake up early?
- When you wake up, do you feel non-refreshed? Do you need naps?
- Do you work on shifts?

2- Depression:
 Psychomotor question: do you think things take more time to do now? Compared
to before?
 Pancreatic cancer  depression
 Whenever you find alcoholic patient  check for complications:
i. Cancer pancreas
ii. Liver damage ( liver enzymes) / hepatitis / cirrhosis / carcinoma
iii. GIT: upper GIT bleeding / peptic ulcer perforation
iv. Depression (alcohol / depression / suicide) is common combination
 Treatment for depression (or most of the psychiatric diseases):
i. Life style modification
ii. Talk therapy
iii. Medications
Usually in combination

3- Domestic abuse presentations 4- Fatigue


- Headache - Depression / PTSD
- Abdominal pain - Domestic abuse
- Insomnia / sleeping pills - Hypothyroidism
- Vaginal bleeding - Fibromyalgia
- Fatigue - Anemia:
- Old person: think cancer & occult blood
- Young female: think menorrhagia
- Diabetes Mellitus, polyuria
- Anorexia nervosa

OSCE-guide-III.doc Page 222 of 255


Psychiatry

Insomnia

A lady complaining of insomnia


Common presentation to: domestic abuse / depression / anxiety

Introduction
CC Insomnia / Tiredness
Clarify the CC - Difficulty falling sleep
- Waking up
Analysis CC: Os Cf D - More at certain time of the week?
- Did you try anything to help? Did it work?
Ask about sleep Sleep hygiene questionnaire
Anxiety - Do you have too many worries?
- What comes in your mind before falling asleep?
- Any changes / stresses in your life?
- Do you wake up with nightmares?
Depression - Screen with MI; if positive  screen MI PASS ECG
PMH
Social - With whom do you live? Support?
 Screen for domestic violence or spouse abuse
- Children?
- Financial support?

Notes
 Did you ever think to hurt yourself? NO, my kids need me,
o What about if they are not around? Maybe!
o This means: implicit yes to suicidal ideation

 Lady looking for renewal of benzodiazepines:


o Renew it and tell her that she needs to use it properly
o Tell her that she is in grief, she needs SSRIs, start SSRIs, taper benzodiazepines

OSCE-guide-III.doc Page 223 of 255


Psychiatry

Domestic Violence Spouse Abuse

Screen for domestic violence or spouse abuse:

- ASSURE confidentiality: I would like to assure you that our conversation is completely
confidential, whatever you will tell me here, I will not release any information, unless
otherwise required by the law!
- With whom do you live? How do you describe this relationship? Supportive?
o How long have you been in this relation?
o Do you feel safe at home? In this relationship?
- Do you or your partner go through stressful times?
o Do you sometimes have conflicts? Arguments?
- Is there any chance that you partner drinks or uses drugs? How often? When he drinks,
does he become angry? Lose control? When was the last time?

Verbal / emotional:
- Does he start to shout at you? Swear at you?
- Does he call you names? How does this affect your self-esteem?
Physical:
- Did he ever get angry to the extent that he became physical?
- Did he try to put you down? Does he try to control you? How did this affect you?
- Did he try to push you? Hit you? How many times?
- Any visits to the ER? When was the last time?
Financial:
- Who is controlling the spending at home?
- Do you have access to financials? Do you take permission?
- Did he ever to try to take you money against your wishes?
Sexual:
- Did he ever force you to do sexual activity against your will? How do you feel?

- Duration of abuse? Severity? Hospitalized!


- How does this affect you? Are you pregnant now? Do you work now? How do you
support yourself financially?

Children involvement:
- Did he ever mistreat / abuse you in front of the children?
- Did he ever mistreat / abuse the children?

Fatality:
- Do you have access to weapons at home?
- Did you ever have thoughts to put an end to this all by ending your life or his life?
- Did you ever talk to anyone about this?

OUTCOME:
- The patient decides to end the relationship and leave  you must provide support and
shelter
- The patient decides to continue: either with OR without police involvement

OSCE-guide-III.doc Page 224 of 255


Psychiatry

Wrap-up:
- Based on what you have told me, what you are experiencing (or have gone through) is
called domestic violence or spouse abuse, and it is common. It is an illegal crime, and it
is against the law.
- It is not your mistake, and you should not feel guilty about that. It is unacceptable, and
nobody deserves to be treated in this way.
- We know from studies that the situation will not improve, on the contrary, it will
deteriorate, and you do not need to accept this. The studies show that the longer you stay
in this relationship, the higher the chance of abuse.
- Consequence to the children (if any): psychological trauma
- It is important that you consider reporting the situation to the police for your safety. It is
difficult decision to leave or stay.
- The husband needs help, check willingness to get counsel.

- If you like to end the relationship and leave:


o You can call the police, they will come and arrest him, then they will investigate
the case, and may put charges against him, and you do not need to worry that he
might hurt you, the police can give a restraining order
o I will connect you with the social services and support groups, you do not need to
go through all of this by yourself, they will be able to help with housing,
financial support for both of you and the kids
- However, if you want to continue the relationship, you can still involve the police. We
know from the studies that if you involve the police, situation will improve, they will
come, speak with him, do some investigations, and then, by law, your husband will be
obliged to attend special training courses:
o Anger control
o Stress management and relaxation techniques
o Drinking problem rehabilitation
o Marital counselling
- My concern is that if you go back without taking any measures, things might deteriorate
and get out of control and one of you might end up losing her/his life, at least:
o You need to prepare an escape plan:
 Where to go when you dont feel safe!
 A bag with essential belongings (e.g. IDs, bank documents, )
 Easily accessible (you can pick it in second and leave)
o We will schedule a follow-up visit within few days
- Is it ok with you if I document this?
- Support:
o Police is not the only option, in the city, there are community recourses: shelter,
hotline, legal aid, they are free, and confidential.
o Give social support group numbers.

Clues for domestic abuse:


- She is avoiding eye contact
- Not answering directly for questions about the relationship with partner
- I hate drinking
- He is a great father, likes his daughters, and works very hard.
- I wish I can be a better mom / I am a failure / do you think too I am clumsy.

OSCE-guide-III.doc Page 225 of 255


Psychiatry

Child Abuse18

The child came to the ER with femur fracture, the skeletal survey showed multiple healing
fractures, counsel

Introduction I assure you that he is ok, and after we finish I would accompany
you to see him, is it ok with you. Before this I would like to ask you
some questions to know more about his condition / fracture
Analyze the event - Can you describe what happened? What he was doing?
- Who witnessed it? Anybody else?
- When did this happen? When did you come to the ER?
Is it the first time? - Other injuries before or visits to ER?
- Analyze each event
- Did you take him to the same hospital?
BINDE  Are you the biological mother?
 Is your current partner the biological father?
- B: screen for the risk factors for child abuse:
 Was this pregnancy planned? Regular f/u visits?
 Was he a term baby? Did he need special attention?
 Has he had congenital anomalies?
 Do you think he is a difficult baby? Fussy baby?
 SAD for both partners!
- I: Are his shots up-to-date? If no, any reason?
- N: What is his weight? Do you know about his growth charts?
Regular f/u visits?
- D: Is he hyperactive baby? Challenges you most of time?
- E:
 How do you support yourself financially? Any support from
the biological father?
 Anybody at home seeing a psychiatrist? Illness?
 Tell me more about your childhood
Other children - Do you have other children?
- Repeated visits to ER?
PMH of the child Chronic illness / bone or metabolic diseases

Screen for domestic violence or spouse abuse

18
Good TWO screening questions: immunization (not up-to-date) / weight (FTT or under
nutrition)

OSCE-guide-III.doc Page 226 of 255


Psychiatry

Wrap-up:

- How do you feel your child has so many fractures?


- I know that you are concerned about your son, sometimes it is challenging to look after a
child, especially if you do not have enough support.
- In the same time, children at that age have flexible bones, and it is difficult to explain the
nature of his fracture(s) only by jumping from a couch.
- In these situations we usually involve the children aid society (CAS), this is a kind of
social services devoted to the safety and well being of children
o Please do not do this? Why?
o They will take my son! Why r u saying so? Any experience with them?
o Not necessarily that they take your child, this is not their first priority
- Their first concern is the safety of your child, what will happen is that they will:
o Ask you some questions about what happened!
o Come to visit you at home
o Talk with your partner
Then they will take their next step based on the results of these meetings
- I am sure you are sharing my concerns about (the child name) safety!

If there is spouse abuse / domestic violence:


- Also, based on what you have told me, what you are experiencing (or have gone through)
is called domestic violence or spouse abuse, and it is common. It is an illegal crime, and
it is against the law.
- It is not your mistake, and you should not feel guilty about that. It is unacceptable, and
nobody deserves to be treated in this way.
- We know from studies that the situation will not improve, on the contrary, it will
deteriorate, and you do not need to accept this. The studies show that the longer you stay
in this relationship, the higher the chance of abuse.
- It is important that you consider reporting the situation to the police for your safety.

OSCE-guide-III.doc Page 227 of 255


Psychiatry

Domestic abuser

You are bout to see a 55/60 years old gentleman, whose wife is recovering in the ER, she has
bruises, and he asked to speak with you. In the next 10 minutes counsel him

Introduction
Analysis
SH / Safety
Counsel  Domestic violence

 Anger control
 Stress management and relaxation techniques
 Drinking problem rehabilitation
 Marital counselling

 Offer social support if there is a need

Introduction:
 If the patient asked to see you: I understand that you are here because you are accompanying
your wife, she has bruises and my colleagues are taking care of her right now. How can I help
you today?
 If the patient is inquiring about her status: I can assure you that she is stable and in safe hands
now.
 If the patient asks to see her: After we will finish, I will ask her, if that is ok with her, I can
take you there.

Analysis:
 Do you have any idea how did she end up having all these bruises?
 Was there any argument / disagreement / shouting? Did you lose control? Did it end up that
you physically hurt her?
 Is this the first time or happened before? Any repeated visits to the ER before?

Social history:
 How long have you been together? What is the nature of your relationship? Stable? Was there
and significant conflicts before?
 Was there any recent change or stressor in your life? How do you support yourselves
financially? Do you have enough resources?
 Do you have anybody else at home? Any family support? Do you have children? How is the
relation with them?
 SAD

Safety:
 Criminal record / access to weapons at home
 If you go home now and face the same situation, how would you react?
 Any chance that you might hurt yourself or any other one?

OSCE-guide-III.doc Page 228 of 255


Psychiatry

Counselling:

 I can see that you are going through stressful period of time. It must be difficult for you and
your wife. Sometimes this stress might present by changes in behaviour and/or personality.
 If you do not have enough support at home, things might get out of control.
 What happened is what we call domestic violence; it is a kind of physical abuse. It is not
acceptable, and it is considered illegal crime. However, this is your wife decision. If she
chooses to report you, that is her right, and nobody can prevent her. She can press charges
against you, and they will take you to the court, in this case you might need legal help, this
might have serious consequences.

 On the other hand, if she decides not to take any measure, may be you should try to improve
the situation by taking steps to decrease the stress in your life, and you can consider reducing
your alcohol drinking. Drinking alcohol might leads to what we call disinhibition in which
one might lose control on his reactions and usually this leads to violent and serious
consequences.
 I can help you by referring you to attend:
o Alcohol rehabilitation programs
o Stress management and anger control programs

 I recommend also that you consider attending family marital counselling; they have good
experience in dealing with couples going through difficult times.

 Finally, I can help you to contact the social services. They might be able to help; you can
speak with them and see what they might be able to do! Is that ok with you?

OSCE-guide-III.doc Page 229 of 255


Psychiatry

Depression
Screen: MI PASS ECG
Organic:
 Illness: hypothyroid, anemia / pernicious anemia, M.S, cancer / cancer pancreas
 Medication B Blockers, Anti-parkinsonian
 SAD

Depression management / counselling


7. Inform the patient
a. Based on what you have told me, the most likely explanation for your condition
is a medical condition called depression, what do you know about depression?
Did you read anything about it? It is the most common mood disorder, in which
you feel low, upset and lack of energy. It is a common problem, and it is
treatable.
b. Explain the pathophysiology: it is related to imbalance of the chemicals in our
brain, most likely related to decreased serotonin
c. Consequences / complications of the condition: it affects functionality, leads to
decreased concentration and ability to work, and in severe cases in susceptible
persons, it might lead them to suicide
8. Treatment (outpatient):
a. Talk therapy (psychologist / psychiatrist)
b. Medications: SSRIs (which are very effective medications) take effect after few
weeks  Cipralex 10 mg PO od x 3 weeks (side effects include: 1. weight gain,
2. GIT symptoms, 3. sexual dysfunction). These side effects improve by time and
to reduce it, we start increasing the dose gradually (start low, go slow).
 Because depression has serious (fatal) consequences, if you do not take this
medication, you will be compromising your safety.
 DO NOT stop it on your own; we can start to gradually decrease the dose
after the proper period of treatment. You need to continue on it for at least 6
months after symptoms improve.
c. Follow up visit after 2-3 weeks
d. Contract: sometimes when the anti-depressant starts to work, the energy level
improves while the mood is still low, that is why sometimes there is increase in
suicidal ideation. Usually happens 2-3 weeks, you need to promise me that if this
happens with you, call 911 or call me immediately and come to see me
9. Offer more info: brochures / web sites
10. Break every 30-60 seconds and ask the patient: does that make sense? Is this acceptable?
Reasonable? Is it clear?

11. Treatment (hospitalization): for suicidal patients


a. From what you have told me, you are meeting the criteria of what we call
and I have concerns about your safety, because you have more than THREE risk
factors for suicide as per the screening test. Do you mind to stay with us in the
hospital for few days, so we can do the required investigations and start the
medications, until you feel ok, what do you think about that?
No doctor, I am not staying in the hospital!!!
Actually, Mr as I told you, I have concerns about your safety, we can not
compromise your safety. And by allowing you to leave today, we will be
compromising your safety.

OSCE-guide-III.doc Page 230 of 255


Psychiatry

Dysthymia
COMMON CASE IN THE EXAM

 Depression presentations:
o Sad (low mode), weight loss, insomnia, tired
 Scale the sadness 0 10
o Indecisiveness: difficulty making decisions
o Low self esteem  how do you feel about yourself?
o If good days: ask for periods (check for gaps 2 months)

 How does it affect your life?


o Then assess functionality; what do you do?!

 Screen MI:
o If positive  MI PASS ECG
 If positive  assess SAD PERSONS

 Any relation to menstrual periods  pre-menstrual dysphoric disorder

 Counselling: similar to depression, but mention that Dysthymia is a milder form of


depression, with longer duration, and does not interfere with life functionality.

OSCE-guide-III.doc Page 231 of 255


Psychiatry

Premenstrual Dysphoric Disorder (PMDD)

DSM-IV-TR Diagnostic Criteria for Premenstrual Dysphoric Disorder


A. In most menstrual cycles during the past year, five (or more) of the following symptoms were
present for most of the time during the last week of the luteal phase, began to remit within a
few days after the onset of the follicular phase, and were absent in the week post-menses,
with at least one of the symptoms being one of the first four listed
1. Markedly depressed mood. Feelings of hopelessness, or self-deprecating thoughts
2. Marked anxiety, tension, feeling of being "keyed up" or "on edge
3. Marked affective lability
4. Persistent and marked anger, irritability, or increased interpersonal conflicts
5. Decreased interest in usual activities
6. Difficulty concentrating
7. Lethargy, easily fatigued, lack of energy
8. Change in appetite overeating or specific food cravings
9. Hypersomnia or insomnia
10. A sense of being overwhelmed or out of control
11. Physical symptoms breast tenderness or swelling, headaches, joint or muscle pain,
sensation of bloating or weight gain

B. The disturbance markedly interferes with work, school, social activities or relationships with
others
C. The disturbance is not merely an exacerbation of the symptoms of another disorder such as
Major Depressive Disorder, Panic Disorder, Dysthymic Disorder or Personality Disorder
D. Criteria A, B and C must be confirmed by prospective daily recordings and/or ratings during
at least two consecutive symptomatic cycles (how to diagnose)

Treatment
 1st line: SSRIs highly effective in treating PMDD
o Fluoxetine (20 mg od) and sertraline (50 mg od) most studied
o Can be used intermittently in luteal phase (mid cycle  onset of menstruation
pre-menstrual) for 14 days
 2nd line
o Alpraxolam (Xanax) for anxiety symptoms
 3rd line
o OCP containing progesterone drospirenone (e.g. Yasmin)
o GnRH agonists (e.g. leuprolide)
o If GnRH agonist completely relieves symptoms, may consider definitive surgery
(i.e. Total abdominal hysterectomy+ bilateral salpingo-oophorectomy)

OSCE-guide-III.doc Page 232 of 255


Psychiatry

Abdominal Pain / Headache


Abdominal pain for few weeks, and was seen by a surgeon last week, comes to your office ( to
have MRI OR to renew medication).
Headache for 7 months, young man, comes to renew Tylenol 3

Chief complain a request (investigations OR medication renewal)

HPI Analyze  Os Cf D / PQRST / / 1st time


the CC  When did the headache (pain) start?
 Did you seek medical attention? What was the diagnosis? Did you take
any medication? When did you start Tylenol 3? Why?
 Analyze previous visits: is the pain different from before? How?
AS  Other pains / headache
 GIT / liver
 Genito / urinary
Impact  How does this affect your life? How are you coping?
Red flags  Constitutional symptoms
 Screen red flags for headache:
 Trauma
 Worse at night
 Nausea / vomiting
 Bothered by light /+/ Neck pain / stiffness
 Weakness / numbness / tingling in body / seizures
 Are you under stress?
 Support systems
DD
Medical problem Somatisation
MOAPS screening (screen for depression)
PMH PMH: HEAD SSS
FH FH of psychiatric disease
SH SH
Physical examination Counselling
Notes:
- Somatisation disorder: (4 pains / 2 GIT / 1 neuro / 1 sexual) complains
- If the pain is only during the day, and not nights  mostly non-organic

+ If requesting MRI  NO MRI


+ Actions for Tylenol 3:
- If using it for few weeks  stop it / do not worry about withdrawal symptoms
- If using it for long time  counsel / renew / promise to cut down gradually
- If using it for depression  start SSRIs / taper Tylenol 3 (decrease gradually) / then brief
counselling for BOTH Tylenol 3 & depression
- If drug seeker  DO NOT give any narcotics / rehabilitation
+ Always renew the medication, except for drug seeker (very anxious to renew the narcotic / will
not accept another alternative / making stories to rash you to prescribe it).
+ If not drug seeker: reassure the patient that you will prescribe her pain medication before the
end of the session, but in order to prescribe the proper medication, you need to ask some
questions, can you bear with me for few minutes?

OSCE-guide-III.doc Page 233 of 255


Psychiatry

Somatoform disorders DD

General Characteristics:
 Physical signs and symptoms lacking a known medical basis in the presence of psychological factors
 Cause significant distress or impairment in functioning
 Symptoms are produced unconsciously
 Symptoms are not the result of malingering or factitious disorder which are under conscious control
 Primary gain: somatic symptom represents a symbolic resolution of an unconscious psychological
conflict; serves to reduce anxiety and conflict; no external incentive
 Secondary gain: the sick role; external benefits obtained or unpleasant duties avoided (e.g. work)
Management of Somatoform Disorders:
 Brief frequent visits
 Limit number of physicians involved in care
 Focus on psychosocial not physical symptoms
 Minimize medical investigations; co-ordinate necessary investigations
 Biofeedback
 Psychotherapy: conflict resolution
 Minimize psychotropic drugs: anxiolytics in short term only, antidepressants for depressive symptoms

Somatization disorder Lots of symptoms: 8 physical symptoms that have no organic


pathology: 4 pain + 2 GIT + 1 sexual + 1 pseudo-neurology
Conversion disorder One or more symptoms or deficits affecting voluntary motor or
sensory function that mimic a neurological or general medical
condition (e.g. impaired co-ordination, local paralysis, double vision,
seizures or convulsions
La belle indifference
Pain disorder Pain is primary symptom and is of sufficient severity to warrant
medical attention
Post-traumatic / post-surgical
Exacerbated by psychic factors
Hypochondriasis Preoccupation with fear of having, or the idea that one has, a serious
disease (e.g. brain tumour) based on a misinterpretation of one or
more bodily signs or symptoms

Fibromyalgia Onset often after car accident


Wake from sleep feeling un-refreshed
Wide spread pain, above and below waist, both sides of the body
Characteristic reproducible tender points: occiput, low cervical C5-
C7, lateral border of the sternum, post neck, lateral epicondyle 2cm
below that point, lat thigh, med knee
Treatment:
 Patient education
 Exercise program (walking, aquatic exercises), physical therapy
(good posture, stretching, muscle strengthening, massage)
 Stress reduction, CBT
 Amitriptyline 10 25 mg qhs
 Gabapentin 300 mg tid
Chronic fatigue Similar to fibromyalgia but FATIGUE is the predominant feature
syndrome Associated with sleep apnea / irritable bowel syndrome
Factitious disorder /
malingering

OSCE-guide-III.doc Page 234 of 255


Psychiatry

Counselling for somatisation disorder:


- I understand that you are here because of , and to (renew medication / do MRI / )
and we will discuss that, but before discussing this, I would like to explain the findings in
your case.
- Based on the symptoms ( and the surgeries you had) the most likely explanation to your
pain (headache) is a medical condition called somatisation disorder.
- What do you know about somatisation disorder? Would like me to explain?
- It is not uncommon condition, and we do not know the exact explanation for it, but we
believe that because some patients are more sensitive to pain than others, or may be due
to patients difficulty in handling stresses in their lives, these stresses may manifest as
painful experiences (symptoms).
- Do you have a family doctor?
 YES
o I will explain some points for you now, and then you can arrange a meeting with
your family physician and discuss the follow up with him, in these situations, it is
important to have only one physician dealing with all the investigations so that he
can get better understanding of the whole situation.
 NO, I do not have a family physician!
o I can be your family physician, if you would like to. That means we will set a
follow-up visits every 4 weeks, during which we will review underlying
symptoms, to make sure we are not missing any serious condition.
o We will review the stress in your life and see how we can help you with it:
 I can refer you to psychiatrist to help you deal better with any stress /
conflict in your life
 And we can consider some medications (Amitriptyline 25 mg PO qhs),
it belongs to a family of medications called TCA (tri-cyclic anti-
depressants) but we use it for pain control

Counselling if the patient is depressed:


- I understand that you are here because of , and to renew medication and we will
discuss that, but before that, I would like to explain the findings in your case.
- Based on what you have told me, the most likely explanation to your symptoms is a
medical condition called depression.
- We need to treat the depression with [talk therapy, behavioural modifications, and
medications (SSRIs, TCA)]  depression counselling
- For Tylenol 3 we will not stop it suddenly, I will renew it for you, but we will agree that
you will gradually cut it down, over the next few days, till the other medication (SSRIs)
kicks in.

If there was a suicidal attempt:


- However, because of the suicidal attempt 2 days ago, we would like you to stay with us
in the hospital for few days so that we can start the treatment

OSCE-guide-III.doc Page 235 of 255


Psychiatry

Drug seeker

If you find a man searching in the drawers of the hospital, firmly ask him to stop, tell him this is private
property and he is not allowed to go through this medical stuff

I wish it could be that simple, but I need more information and physical exam before I can write any
prescriptions to you, as I am a little bit concerned about the amount you have been taking, which might
have been harmful to you

Introduction  Why are you taking it? What was the diagnosis?
HPI Analyze  Os Cf D / PQRST / / 1st time
the CC  When did the headache (pain) start?
 Did you seek medical attention? What was the diagnosis? Did you take
any medication? When did you start Tylenol 3? Why?
 Analyze previous visits: is the pain different from before? How?
AS  Other pains / headache
 GIT / liver
 Genito / urinary
Impact  How does this headache affect your life? How are you coping?
 Have the medications been impacting your life?
 Relationship with family
 Education, Employment
 Legal problems, police involvement?
Red flags  Constitutional symptoms
 Screen red flags for headache:
 Trauma
 Worse at night
 Nausea / vomiting
 Bothered by light /+/ Neck pain / stiffness
 Weakness / numbness / tingling in body / seizures
 Are you under stress?
 Support systems
Analyze Tylenol 3
Other medications In addition to Tylenol 3, do you take any other meds? Sleeping pills?
MOAPS screening
PMH: HEAD SSS
FH of psychiatric disease
SH
Counselling
Analyze Tylenol 3
- So you told me you are taking it for
- Who prescribed it to you?
- Who renewed it to you? Why?
- When was the last renewal? Can you show me your last bottle?
- How many tablets do you use now? And before? When did you start to the use?
- When you take it, beside for the headache relief, how do you feel? What if you do not take it, how do
you feel? Shaking? Heart racing? You feel you are on the edge?
- Do you renew it from the same doctor or different doctors? Why you did not go to him this time? Is it
ok that I contact him?
- Do you renew it from the same pharmacy or different pharmacies? Is it ok that I contact the pharmacy?
- Did you ever obtain the medication from the street?

OSCE-guide-III.doc Page 236 of 255


Psychiatry

Given the benign history with no suspicion of ICP or focal deficits, and description of headache
consistent with the common tension headache, full neurological examination is not indicated, I
would like to perform a brief neuro screening exam  move on.

Counselling:
- I understand that you are here to renew your Tylenol 3; we will discuss that, but before
that let me ask you: what is your understanding of Tylenol 3?
- Tylenol 3 is a good medication when it is used for particular indication. Do you know
what does it contain? It contains 2 medications:
o One of them is the regular Tylenol as you buy it from the pharmacy
o The other one is codeine
- Tylenol itself is a safe and effective drug, and can be used for long time, however, if there
is no strong indication to use it, it is better to it as it might cause liver and kidney injury.
- On the other hand, the other medication codeine it is a drug belongs to the family we
call narcotics which is similar to morphine. It is an excellent pain killer if used for short
term, but, if it is used for long term, this is concerning for us, do you know why?
o First of all, people need to keep increasing the dose in order to obtain the same
effect; we call that tolerance.
o Also, if you stop using it suddenly, you will have withdrawal symptoms,
similar to that you have now; running nose, tearing, N/V, diarrhoea, drowsiness,
muscle aches, sweats, shaking, and heart racing.
- For these reasons, people get easily hooked on Tylenol 3, and can not stop it. Not only
that, they will need to keep increasing the dose. We call that a habit forming
medication.
- If I renew your medication, I will not be helping you, it will be like a vicious circle, and
the more I renew your medication; the more dependent you will be on it; the more you
will need it. For that reason it is not the right step to renew it.
o Can you give me just few pills; I have a very important interview?
o Even if I give you few pills, this is not the solution, this will be temporarily, and
the problem will keep increasing. We must stop the drug
o I can help you with sick note
o I can give you another non-narcotic medication that can help you with your pain
- I appreciate your trust to give me all the information, but based on what you described,
you are having dependence on narcotics.
- It sounds like you have been going through a lot of stress in your life. I am wondering
that if you would be interested in talking to one of our social works here, who is expert to
find out the community resources for you.
There are also some numbers you can call; they are professionals to help people deal with
medications or drugs. Or if you like, I can refer you to a detoxification center, where they
will help you to quit.

The seeker may be seeking Fiorinal


- Fiorinal is a combination preparation of (barbiturate / caffeine / ASA) properly used only for the
relief of occasional tension headaches.
- It is a habit-forming medication; that can precipitate withdrawal symptoms: agitation, delirium
and seizures.
- The fact that patient consumes a lot suggests overuse due to dependence.
- The patient may also develop analgesic headache syndrome, in which inappropriately used
analgesics actually cause headache.
- Suggest a drug holiday, with weaning from caffeine and alcohol, proper sleep hygiene, diet
control, exercise, and stress management.

OSCE-guide-III.doc Page 237 of 255


Psychiatry

Lithium discontinuity

Introduction  Have been diagnosed with bipolar 3 years ago, and would like to
discontinue your medication.
 What is the medication you want to stop?
 Why would you like to stop your medication?
 I am glad you came here to discuss it, any other concerns
Mania History  When were you diagnosed? How?
 Any serious consequences? Were you hospitalized? For how
long?
 Were you seen by a psychiatrist? Regular f/u?
Today Do you feel: DIG FAST (distractibility, impulsiveness, grandiosity,
flight of ideas, activity, sleep, talkative)
Scan for MI PASS ECG
depression
Lithium History  Do you renew your medications on regular basis?
 How much lithium do you take? From the beginning?
 Are you taking it regularly?
 Do you measure lithium level? On regular basis? When was the
last time? What was it? What is your target?
 Are you still taking it? Did you stop?
 How do you feel about lithium?
Side effects  Hypothyroidism: do you have your thyroid hormone measured?
Do you feel cold? Dry skin? Constipation?  give thyroxin
 Diabetes insipidus: do you feel thirsty? Drink more? Pee more?
Got your urine checked? ttt: thiazides
 Abdominal pain? Nausea / vomiting?
 Neuro shaking/tremors: -blockers
 Neuro ataxia/balance/seizure: stop it
MOAPS I know that you have been asked all these questions before, let me ask
it for another time!

Do you feel: DIG FAST (distractibility, impulsiveness (with painful consequences), grandiosity,
flight of ideas, activity, sleep, talkative)
D Do you have a lot of projects? Were you able to finish it to the end? Can you focus on
multiple projects?
I  Are you spending more money than before? Are you borrowing money that you can not
pay back? Are you over-using your credit cards?
 With whom do you live? Many sexual partners?
 SAD: what started 1st; feeling high or talking drugs?
 Have you had problems with the law? Fighting? Arrest? Speeding tickets?
G Do you feel very special? Have special mission?
F Do you feel a lot of thoughts? Ideas?
A How much time do you spend on your projects?
S How many hours do you sleep? Any changes?
T Did anybody mention that you are talking fast?

OSCE-guide-III.doc Page 238 of 255


Psychiatry

Side effects of any medication:


- Liver toxicity
- Nausea, vomiting, diarrhea
- Headache
- Insomnia, irritability
- Agranulocytosis (Carbamazepine): check CBC every week, then every 2 weeks, then
every month

Counselling:
- I understand you are here because you would like to discontinue the lithium, however
before we discuss that; I would like to know your understanding about mania and mood
disorders!
- Mood disorders are common, and the most common of them is depression where people
feel low and do not concentrate and its treatment include the talk therapy and medications
that could be used for 6-12 months and could be stopped if the condition improved and in
some times we need to give the treatment for longer periods of time.
- This is not the case for mania/bipolar. We can treat and control it, but we can not cure it,
may be one day in the future we will be able to do this.
- Your chance of relapse if you stop it is 60% and after the second time this goes up to
80%, and after the third time it goes higher to 90%. You can see it is increasing.
- Based on your lithium level, which is within therapeutic target (0.5 1.2), we can
measure it today and we can try to decrease it gradually to check if you are feeling good
and closely monitor you. But you have to promise me that at anytime you feel high mood,
start to spend too much, talk fast or start not to sleep well, you have to come to see me or
go to the nearest ER and inform them.
- Regarding your inability to write, this is not related to lithium, thought block is not a side
effect of lithium. You may try some relaxation techniques to help you concentrate more.

OSCE-guide-III.doc Page 239 of 255


Psychiatry

Manic patient

If the patient is psychotic:


First step is to detect early what is his mood?
- If high mood: manic attack, with psychotic feature
- If no high mood: brief psychotic disorder / schizophreniform

Usually patient brought by police or family member or asked to come by family members
 Patient is talking fast and a lot, laughing, moving around
 Ask whether the patient has been on medication before or not, e.g.: Lithium
 Ask about any side effects of lithium medication N/V / Diarrhea / tremors / polyuria
 Obtain history in the usual format

Introduction
Ask about the Mood
Assure the patient Assure the patient: you are safe here, you are in the hospital and no
one will hurt you
Red flags Fever / headache / nausea & vomiting / head injury
HPI  OCD
 Mania (DIG FAST)
 Depression (MI PASS ECG)
 Suicide (SAD PERSONS)
 If you leave what will happen? What would you like to do?
MOAPS Screen
 SAD: alcohol / substance abuse / amphetamine
 Medical conditions; hyperthyroidism: history of thyroid
problems, symptoms (heart racing, sweating, heat intolerance,
neck swelling, visual field changes)
PMH / FH Psychiatric disease
SH

N.B. if any patient has mood disorder; go through DIG FAST and MI PASS ECG

Management:
 Explain that the patient has recurrence of his mania or bipolar. This is because he stopped
taking the Lithium.
 Will examine and do some tests.
 Will start medication. If Lithium is causing some troubles, we can start another medication.
 Usually you need to admit the patient to control the symptoms of mania (from what you have
told me, you are meeting the criteria of what we call manic episode and I have concerns
about your safety).

OSCE-guide-III.doc Page 240 of 255


Psychiatry

Suicidal attempt
LOTS OF EMPATHY
Introduction - And to see what should be the next step, first, I would like
to know how you feel about being saved.
o If happy, I am glad for that
o No!
Analyze the event - Assure confidentiality
- Can you tell me more about what happened?
- What is the name of the medication? How many tablets? Any
alcohol with it?
- Why did you do that?
- Is it the first time?
- Who saw you and brought you to the hospital?
Before - Assess the plan here, was it organized? Or it was an impulse?
Did you leave a note? Recently, have you been giving your
belongings away?
After - What is going in your mind now?
- If you leave the hospital, what are your plans? Where do you
want to go? What do you want to do?
- If another crisis may happen, are you going to hurt yourself?
Psychiatric assessment - Were you seen by a psychiatrist? Were you given a diagnosis?
Do you see your psychiatrist? Take meds?
Risk - Assess the risk factors: Analyze SAD PERSONS
MOAPS - Screen for anxiety
- Screen for psychosis
- Screen for suicidal / homicidal ideation / self care
- Past medical history / allergy / medications /
Decision
Conclusion / Counselling
SAD PERSONS
S A D P E R S O N S
Sex Ag Depressio Previou Ethano Rationa Suicid Organize NO Seriou
Mal e n s l l e in d plan suppor s
e > attempts thinkin the t illness
65 g lost family
SAD HEAD PMH
SSS
3-4 Release if enough support
>5 Hospitalize
E - SAD
R - What did you think will achieve by ending your life?
- Sometimes people hear voices asking them to end their life, did you hear this?
N - HEAD SSS
- H: With whom do you live? Anybody else? Anybody else? If there is a step-
parent in the image, ask about the relations with him and with other parents. Do you
feel safe at home? Then ask gradually, if there is a chance that this parent might get
angry when he drinks? May shout, may swear at, may push, and may hurt?
S - Past medical history

OSCE-guide-III.doc Page 241 of 255


Psychiatry

Decision:
- If still depressed and/or SAD PERSONS (>3-5)  admit
- It she is ok, regrets the accidents, no SAD PERSONS  release

Conclusion / Counselling:

HOSPITALIZE
- Based on our interview, I have concerns about your safety, because you have more than
THREE risk factors for suicide as per the screening test. Do you mind to stay with us in
the hospital for few days, so we can do the required investigations and start the
medications, until you feel ok, what do you think about that?

RELEASE
- Based on our interview, it is ok if you would like to leave, but you have to arrange a
follow up meeting with your family doctor within 3 days.
- However, I would like you to know that life sometimes could be challenging, and you
may face challenges in the future. It is important that you learn how to deal with
challenges. If you feel over whelmed, talk to somebody, and ask for help
- I can arrange a meeting with a social worker, a psychiatrist!
- I would also like you to promise me that if at any time you want to harm yourself or end
your life, you will seek medical help immediately; you can come to my office or call 911.

If multiple suicidal attempts  borderline personality disorder  do NOT admit

Notes:
- If no eye contact, wasting time, no pt interaction  assure confidentiality
- Whenever you hear car accident  show empathy / did you hurt yourself / ask about
who was in the car / was any one injured?
- If the person driving was < 18 and was driving alone  be curious  this must be an
important meeting / person that you really did not want to miss!
- The girl asks you to tell her mother that she crashed mothers new car! She does not want
to directly (herself) inform the mother!
o I can not do this.
o Why do you think this would help? She will not be angry
I see, however, life is full of challenges, it is better that you try to learn how to
deal with challenges yourself.
o We can help you to tell your mother by yourself, we can arrange a meeting with
your mother, I can be present, or we can ask a nurse or a social worker to be
there.
- The girl does not want to inform her parents that she did attempt suicide!
o You assess her and if she is to be released, e.g. she regrets what happened, she is
happy to be saved, no SAD PERSONS risk factors  she is competent 
respect her wishes.

OSCE-guide-III.doc Page 242 of 255


Psychiatry

Eating disorder

Young female, her parents brought her because they have concerns about her weight

Anorexia nervosa Bulimia nervosa


Restrictive Binge-purge
Under weight Average low weight
Distorted self image
Amenorrhea
Disturbed perception  loss of insight  Binge-purge (> 3 times / week) > 3 months
incompetence  inform parents and admit Lose control  over-eat  react (purge)
involuntarily

Introduction Your parents brought you . How do you feel about that?
I am glad that you came:
- To figure this out (if she is ok)
- To assure your parents (if she is not ok)
Weight analysis
Diet
Exercise
Extra measures
Impact
MOAPS - Mood: scan for depression
- Organic: DM / hyperthyroidism / constitutional symptoms (cancer)
- AP: screen for anxiety / psychosis
- S: HEAD SSS
FH Eating disorder / psychiatric illness / suicide

Weight analysis:
- What is your weight today?
- When did you start to lose weight? What was your weight at that time? How much did
you lose? What was your highest weight? What is your target weight?
- Why are you losing weight?
- Are you losing weight alone? Or someone else is encouraging you?
- When do you look at yourself in the mirror, how do you perceive yourself? How do you
perceive your weight?
- Do you like to dress in baggie?
- It looks like you lost a lot of weight in short period of time; I would like to know how did
you achieve that?

Diet:
Let us talk about your diet;
- How many meals do you eat per day? How about snacks?
o What do you eat in breakfast? How about the amount?
- Do you calculate calories? How much calories do you eat per day?
- Do you eat alone or with other people?
- Do you like to collect recipes? To cook?

OSCE-guide-III.doc Page 243 of 255


Psychiatry

Exercise
- How about exercise? Do you exercise?
- How many times a week?
- Do you dance? Practice any sports?
Extra measures:
- Do you take anything else to help you to lose weight?
- Do you take stool softeners? Do you take water pills?
- Did you try before to induce vomiting?
- Do you sometimes exceed the amount of food you intended to eat? How many times a
week?
- How do you feel after that? How do you compensate?
Impact / consequences:
Because you have lost a lot of weight, I would like to know the impact of this on you!
- Do you have amenorrhea? When was the LMP? Regular?
- Do you feel cold / tired / swelling in your legs?
- Pigmentation on your skin? Fine hair growth? Skin changes?
- Any bony pains? Fractures?
- Muscle cramps? Calf pain?
- Heart racing? Light headedness, dizziness, fainting?
Conclusion:
- I am concerned that you have a condition called Anorexia Nervosa (explain)
- It is affecting your body, without treatment it could be fatal
- The treatment is to start eating and to gain weight. It is a tough task but I will refer you to
a multi disciplinary team to start treatment
- Would you like to discuss that with your parents

Management of anorexia nervosa:


- Anorexia patient is to be admitted to hospital if:
o <65% of standard body weight (<85% of standard body weight for adolescents),
o Hypovolemia requiring intravenous fluid,
o Heart rate <40 bpm
o Abnormal serum chemistry or if
o Actively suicidal
- Agree on target weight on admission and reassure this weight will not be surpassed
- Psychotherapy (individual/group/family): addressing food and body perception, coping
mechanisms, health effects
- Monitor for complications of AN
- Monitor for re-feeding syndrome: a potentially life-threatening metabolic response to re-
feeding in severely malnourished patients resulting in severe shifts in fluid and electrolyte
Bulimia nervosa:
- Criteria for admission: significant electrolyte abnormalities
- Treatment: biological (treatment of starvation effects, SSRIs), psychological (cognitive
behavioural therapy, family therapy, recognition of health risks)
Notes: So doctor do you agree with me that I am overweight? Or do you see me like my parents I am not
good?
- I will share your parents concern, it looks like you lost significant weight in short period of time,
and this is concerning.
- If the patient lost interest  slow down  summarize and start again slowly

OSCE-guide-III.doc Page 244 of 255


Psychiatry

Mini-mental status exam:

Introduction: Mr Now, we will do a mental exercise, in which I am going to ask you some
questions. Some of these questions are easy, and some questions are difficult, please do as much
as you can!
 Prepare this list before you go to the room in cases of delirium / dementia / post-concussion.
Then you can mark the correct or the wrong ones
1 2 3 4 5 5 Orientation to place
6 7 8 9 10 5 Orientation to time

11 12 13 3 3 words recall immediate


14 d 15 l 16 r 17 o 18 w 5 world backwards
19 20 21 3 3 words recall delayed

22 23 24 3 3 steps command
25 26 2 Aphasia (pen / watch)
27 Close your eyes! 1 Read / execute
28 1 Write
29 1 Copy
30 No ifs, ands or buts 1 Repeat

1-5 / Orientation to place: do you know which country we are in? Province? City? Hospital (or
street) name? Which floor (or suit number)?

6-10 / Orientation to time: do you know which year we are in? Season? Month? Day of the
month? Day of the week?

11-13 / 3 words recall immediate: I am going to tell you 3 objects, and I would like you to
repeat after me and memorize it, and I will ask you about it later! (penny/ tree/ car)

14-18 / Concentration: can you spell the word world backwards? He gets -1 for each non-
matching letter (first check if he can spell it correctly forward)

19-21 / 3 words recall delayed: can you tell me the 3 words that I told you before

22-24 / 3 steps command: give all the instructions at once; are you left or right handed? Can you
please take this paper by the hand / fold it into halves / give it back to me?

25-26 / Aphasia (pen / watch): what is the name of this? What is this?

27 / Read and execute: can you read this sentence and do what is written in it!
28 / Write: can you write a sentence for me!
29 / Copy: can you copy these two shapes!
30 / Repeat: can you repeat after me; no ifs, ands, or buts!

MMS score < 24  incompetent

OSCE-guide-III.doc Page 245 of 255


Psychiatry

Dementia

Difficulty with memory for 6 months

Introduction I would like to ask some questions; then we will do a mental exercise
Analysis of CC Memory assessment
Behavioural  Did anybody tell you that you have changes in your personality? Being short
changes temper? More arguments?
 If there is a fire in this building; what are you going to do?
 How about your sleep? (dementia: fragmented sleep /+/ delirium: reversed
sleep cycle; sleep at day, awake at night)
MMS
Let us take a day of your life; I would like to see how did it affect your life?
DEATH Activities of daily living (ADL)
SHAFT Instrumental Activities of Daily Living (IADL)
MOAPS Organic in details and screen the rest (especially mood for pseudo-dementia)

Memory assessment: Can you tell me more about this difficulty! OCD +
- Any fluctuations in memory level?
- This deterioration is gradual slowly progressive, or is it you feel ok for a while then you
have attack then you are fine then you have another attack? (step ladder)
- Are you having difficulty memorizing numbers?
- Do you have difficulty finding words?
- Do you have difficulty reading? Writing? Calculating?
- Do you lose your stuff?
- Do you make lists to remind you to do things you used to do on regular basis? Do you
have difficulty organizing your schedule?
- Do you have difficulty doing tasks you used to do before; like tying a tie?
- Do you feel difficulty for new events, or old events?
o Recent: What did you have for breakfast? Confirm from partner!
o Remote: Who was the USA president during WWII? (Roosevelt)

ADL DEATH:
- Dressing: difficulty dressing and undressing yourself?
- Eating: do you remember to get all your meals? Or do you skip meals?
- Ambulatory: do you have difficulty moving around?
- Toileting: how about urination? Have you ever lost control or wet yourself?
- Hygiene: any difficulty having showers?

IADL SHAFT:
- Shopping: who is responsible for shopping? You or your wife?
- House keeping: how about house keeping, are you able to help your wife?
- Accounting: who is responsible for banking at home?
Did you ever give cheque without balance?
- Food: do you cook? Did you ever forget the stove on?
- Traffic: do you drive? Difficulty driving? Have you ever lost your way?

OSCE-guide-III.doc Page 246 of 255


Psychiatry

MOAPS screening:
Mood:
- Depression pseudo-dementia?
Organic:
- Do you have nay long term disease? Kidney? Lung? Heart?
- SAD
 History of stroke? Difficult with vision / hearing? Weakness / numbness? Loss of
balance? Urinary incontinence?
 Head trauma? Injury?
 Brain tumour / infection
- Medications? OTC? Sleeping pills?
- Any history of thyroid disease? Symptoms of hypothyroidism?
 Hx of surgeries? In stomach?
 Are you vegetarian? For how long? Do you take supplements?  pernicious anemia

Anxiety
Psychosis
Self care / suicide

Dementia cases:
- 69 years old man comes to your clinic because he is keeping forgetting for the last few
months. In the next 5 minutes; take history and assess (this is too long for 5 minutes, but
during taking history, and if you mention: I would like to do the MMS exam, the
examiner will give you the score)  Alzheimer.
- 55 years old patient comes to your clinic because he has difficulty in memory. His MMS
score is 21. In the next 5 minutes, take history  thyroid.
- 67 years old man, comes to your clinic complaining of difficulty with memory. In the
next 10 minutes take history and assess (make MMS exam)  Dementia.

The cases could be:


- Thyroid disease (especially if pt is younger than 60 years)
- Alzheimer disease
- Dementia
- Depression pseudo-dementia
- HIV
- Pernicious anemia
- NPH (normal pressure hydrocephalus): if the patient has difficulty in AT of the
DEATH; i.e. falls due to ataxia and urinary incontinence

OSCE-guide-III.doc Page 247 of 255


Psychiatry

Delirium
Delirium cases:
- A middle aged gentleman comes to your clinic because his dad is not himself for the last
3 days. Take history by proxy
- A middle aged gentleman comes to your clinic because his mom is in seniors home; they
gave her 15 units of insulin instead of 5 units, and she is not herself. Counsel him!
(insulin induced hypoglycemia  stressful event  decompensate a border line
delirium)
- Patient has surgery 3 days ago, not feeling himself. Patient will be aggressive.
- Patient has surgery 3 days ago, not feeling himself. Patient will keep repeating: I do not
know!  mini-mental status exam

Case 1: Dad has not been himself / not sleeping well


Introduction I will ask some questions in order to reach a working plan
Analysis of the CC  How old is he? What are your concerns?
 Tell me more! Any recent stress? OCD
 Did you notice if your dad is angry / aggressive?
 Does it look like your dad is seeing things do not exist? Hearing
voices? Complaining of insects crawling on his skin?
 Does he sleep during night? What a bout during the day?
 Is he eating? Taking care of himself?
 With whom does he live? How is he capable of keeping life? How
does this affect his / their life?
 Is it first time?
Causes  Constitutional symptoms
DD  Any headache / vomiting / neck pain / skin rash / red eyes / any ear
discharge / runny nose / teeth pain / diff swallowing / SOB / cough /
Infection urine changes / abd pain / calf pain / swelling
Trauma  Head trauma? Injury?
Surgery  Recent surgeries? Pain at site of injection? Dressing change?
SAD  SAD
Medications  What about medications, do you have a list with medications? Go one
by one!
- Is he hypertensive? Controlled? Regular measurements?
- I can see that he is diabetic; for how long? Controlled? Regular
f/u and measurements? HbA1c?
- Cholesterol / Water pills / Anti-depressants
- Sleeping pills; if more than 1; ask if it was prescribed by the
same doctor
- Erythromycin!!! Why was he taking it? Pneumonia!
Conclusion It looks like your dad has a medical condition called delirium it is a
serious condition. Your dad needs to be seen by a doctor ASAP, can you
bring him to see me. If he is too far, he needs to be taken to the nearest
ER; we will need to decrease or stop some of his medications, and restart
them gradually.

Theophylline (for asthma): stop and take beta 2 agonist instead


Erythromycin (for pneumonia): change the antibiotic
Lorazepam: discontinue

OSCE-guide-III.doc Page 248 of 255


Psychiatry

Case 2: DT
Patient is agitated, delirious and uncooperative
Introduction I can assure you that are safe here, you are in the hospital and no one will
hurt you, we would like to help you
 I can see that you are looking to the wall, do you see anything? Do
you see anything else? Do you hear voices?
 Doctor, do you see the spiders I see? For me, it does not look like
spiders, however, I understand that you can see them at the moment,
but I can assure you that nothing will hurt you!
Analysis of CC  I can see you are scratching; do you feel anything? Do you hear / see
anything?
 Do you think any one would like to hurt you? Assure safety!
 When did that start? OCD?
 How was your sleep?
Full MMS exam
Causes  Constitutional symptoms
DD  Any headache / vomiting / neck pain / skin rash / red eyes / any ear
discharge / runny nose / teeth pain / diff swallowing / SOB / cough /
Infection urine changes / abd pain / calf pain / swelling
Trauma  Head trauma? Injury?
Surgery  Recent surgeries? Pain at site of injection? Dressing change?
SAD  SAD: any shaking / sweating
Medications  What about medications, do you have a list with medications? Any
sleeping pills?
 Do you have nay long term disease? Kidney? Lung? Heart?
Conclusion It looks like you have a medical condition called delirium it is a serious
condition. It is reversible, fluctuating, impairment of LOC. It affects 25%
of Hospitalized people.
 Will give medication to help you calm down
 Will have a nurse close by if you need any thing
 Will keep the room quiet and well lit
 Will come back again to see you
Notes:
- It the patient is not cooperative, keeps repeating I do not know; start to ask the questions of the
MMS exam, they will go with you. After you finish, you can continue the rest of your exam
- If the patient is starring at the wall; ask him: I can see that you are looking to the wall, do you see
anything there?
- Mental status exam = psychiatric interview
- For delirium; we do the MMS exam daily until he improves
- For dementia; we do the MMS exam every 3-6 months; for follow-up
If confused patient (long case examination)
 GCS: only if the patient is poorly responsive
 MMS
 Cranial nerves
 Body:
- Pronator drift
- Hoffmans reflex thumb flexion  UMNL
- Cerebellar tests: finger to nose, rapid alternating movements
- Power / sensation / reflexes
 Patient standing: gait, Romberg test, planter flexion power
 Patient supine: tone

OSCE-guide-III.doc Page 249 of 255


Psychiatry

Smoking Cessation counselling

1- Congratulations, We will speak in details about how we can work together to achieve this
healthy goal, but first let me ask you some questions, I need to have the bigger picture about your
smoking, and this will help us to figure out the best plan to achieve our goal
2- Smoking history:
 When did you start smoking? For how many years?
 How many cigarettes per day?
3- Reasons (motivations): to seek smoking cessation
4- Previous attempts: How many times? Why did you fail? When was the last time?
EMPATHY: failure is a normal part of trying to stop
5- Is there any other smoker in your home? Is she/he willing to quit? It will be a great idea if
both of you tried to quit at the same time, this will increase the success rate of your trial.
If she/he would like to know more information or need help, I will be more than happy to
meet her/him, we can arrange a meeting
6- Impact (complications of smoking):
 Cancer (lung hemoptysis, tongue, nasopharynx, urinary bladder, other cancers)
 Cardio vascular hazards (myocardial ischemia)
7- Red flags:
 Constitutional symptoms
 Risk factors (personal history or family history) of:
 Heart disease / attack / HTN
 Diabetes mellitus / hyper-cholesterolemia
8- Plan:
 STAR:
i. Set a quit date, print papers with this date and stick it under your vision
so that you see it frequently during the day
ii. Tell your family, friends, they will be your support
iii. Anticipate the challenges you will face (nicotine-withdrawal effects:
headache, nausea and a craving for tobacco, insomnia, irritability,
anxiety, and weight gain)
iv. Remove cigarettes and other tobacco products (e.g. ashtrays) from your
home, car, and work
 Nicotine Replacement Therapy:
i. Nicotine patch [21 mg (if smoking > 25 cig/day), 14 mg, 7 mg]
ii. Nicotine gums
iii. Nicotine inhaler
 Psychological support for smoking cessation (to the craving):
i. Zyban (Bupropion):
+ used with tapering smoking for 2 weeks, then stop smoking
+ 150 mg qAM x 3 days then 150 mg bid x 3 months
+ Contra-indications: epilepsy, seizure disorder, eating disorders, patients
undergoing abrupt discontinuation of ethanol or sedatives
ii. Champix (Varenicline): urge to smoke and withdrawal symptoms
+ 0.5 mg qAM x 3 d then 0.5 mg bid x 4 d then 1 mg bid x 3 months
 Investigations:
i. CBC / urinalysis / lipid profile
ii. If there is risk factors for heart diseases: stress ECG test
iii. If patient is worried, or if there is hemoptysis: chest x-ray

OSCE-guide-III.doc Page 250 of 255


Psychiatry

Refusal to treatment counselling


Mrs 56 yrs old, was recently diagnosed with lung cancer, counsel her.
Introduction - I understand you were diagnosed recently with lung cancer
- How are you coping?
History Brief history - SAD
- Family history of lung cancers
Support - With whom do you live?
- Any family support?
- How do you support yourself financially?
Lung cancer - What is your understanding of (lung cancer)?
- Do you know which type you have?
- Available treatment
- Now, I would like to explain the treatment options we have,
- Once diagnosed, usually surgery is late to be done, so we have radio / chemo
- Based on your condition and stage of cancer, the surgeon thinks that chemo
and/or radio therapy are the best line of treatment for you, this is based on
the many clinical trials and evidence-based medicine.
Treatment - No doctor, I do not want to contaminate my body with chemicals, I am going
refusal for spiritual therapy! OR whatever cause!
Ask for reason: Why? How long have you been thinking that way? Did you talk
about this with your family?
Assess competency; rule out depression:
- When we talk about decision-making, we need to rule out certain conditions
and to know more about your health. Can you please tell me how has your
mood been lately? Do you find yourself cry easily? Interests? Suicidal
ideation? Any major event, death, accident in the family recently? MI PASS
ECG
Counsel If depressed  assess suicide and psychiatric consult
- You sound depressed to me, I would love to respect your wish, but I would be
more comfortable to follow that after you talk to psychiatrist, do you want to
do arrange that?
- Give it more thoughts; I will arrange the family meeting, social worker, and
psychiatrist. I will come back to talk to you again tomorrow
In not depressed  she is competent, she can refuse treatment
- After all this is your decision; I just want to make sure you know the available
treatments that were proved to be beneficial in treating lung cancer.
- Why not to try both? We start the medical therapy that we are sure it works,
and you go for spiritual therapy!
- Explain the condition, the available treatments
- The side effects of treatments and the complications of not getting treatment
(terminal illness)
- What about arranging a meeting with some one who has had the same medical
condition, and speak with him/her. You will get better insight into the disease
and you will see the results of treatment.
- How about arranging a meeting for you and your family members (if you wish)
with the surgeon, so that he can explain condition in more details?
- A 71 years old with cancer colon, patient concerns: afraid of living with colostomy / concerned about
being a burden to the family / afraid of complications of the surgery (a friend died in similar surgery)
- A 70 years old lady with lung cancer, refusing the surgery because her husband died 30 years ago in a
surgery (she does not know why) / she thinks that people die from the anesthesia

OSCE-guide-III.doc Page 251 of 255


Psychiatry

Truth telling

Usually a son or daughter asking you not to inform the patient (parent / grandparent) about his
terminal illness or advanced condition

Introduction:
- Well, it is not unusual for families to have that request!
- Why you do not what her/him to know? (cant handle the bad news, fragile personality,
depression, )
- Does the patient have advanced directive? Will? Have discussed this before?

Explain patient must be told:


- Well, you know your loved one the best, given her/his previous reaction, it is reasonable
to have that concern.
- On the other hand, the patient has the right to know, we can not hold this right, besides,
usually; people will go through different stages when they react to bad news, we have
specialists here who can really help the patient and family to go through these stages.

Explain the reasons to tell:


- Patient has the right to know
- Patient will have suspicion about his own condition
- We need to discuss the treatment options, and it is the decision on the patient
- Patient may need to start some arrangements

Explain the implications not to tell:


- It is difficult to hide, it is a team work, will eventually know or find out
- Patient will lose the trust to doctors in general

Decision:
- Will talk to the patient to see if she/he wants to know all the details or not?!!
o If yes, we have to tell her/him
o If no, we will ask if she/he would like us to inform someone else
- In all cases, if the patient asks, we have to tell her/him

Conclusion:
- I can tell that the patient has a very caring family, it must be very hard on the family as
well, if they need someone to talk to cope, I can arrange that if they want
- I can give the family a little bit more time to think and we will talk again, patient will
eventually need to know the truth.

OSCE-guide-III.doc Page 252 of 255


Psychiatry

Organ Donation

Explain resuscitation effort


- What has been done to the patient resuscitation
- When he came in, he was not breathing, we put down a tube, his heart was not beating,
we did the compression
- What is the outcome, patient is not responding to treatment due to the severe trauma to
the head.

Explain the condition: brain death


- Patient is in a state called the brain death, (check with the relatives if they know this
terminology)
o Irreversible brain damage, no functioning at all
o Not responding to light, to pain
o No spontaneous breathing, will never gain consciousness, when we stop the
machine, he does not breathe
o Legal term for death.
- Confirmed by two nerve specialists: neurologist / neurosurgeon:
- Show empathy: sorry for the loss, patient was young / healthy / family needs him /
Give time to family to recall how active and how nice he was

Bring the issue of organ donation:


- He was healthy, good candidate for organ donation
- If he had advanced will, driver license
- What familys view about this
- It is familys decision now, the decision is a life gift

Explain how to do it
- We have a team to do that, they will respond very quickly
- Many organs can be used
- There is time limit; decision should be made within the next 24 hrs
- You will be notified which organ used and where to go, but you wont get the
individuals name

Explain funeral
- It wont affect the arrangement for funeral
- Still can have the open casket, wont affect the face

Address any questions or concerns

OSCE-guide-III.doc Page 253 of 255


Psychiatry

OCD

- Address body language


- Why do you wear gloves and hat? And refuse hand shaking?
- Start with an open ended question; OCD / trigger /

 Obsessions:
- Type of obsession: dirt and contamination, orderliness, religious, checking and
rechecking?
- Do you feel that these obsessions are not real?
- Do you want to get rid of them?
- What do you do to overcome the stress created by these ideas?
- How many times do you wash your hands? How long do you take in a shower?
- Impact on life, work,

 MOAPS:
- Screen for mood disorders
- Screen for organic causes
- Screen for other types of anxiety disorder,
- Screen for psychosis
- Screen for suicide, homicide, self care

 Past medical history / medication & allergies


 Family history
 Social Hx: smoking, alcohol, drugs, work, home, support

OSCE-guide-III.doc Page 254 of 255


Psychiatry

NOTES

OSCE-guide-III.doc Page 255 of 255

Вам также может понравиться